You are on page 1of 110

2017-18 100 &

op kers
Class 12 T
By E ran culty
-JE Fa r
IIT enior emie .
S fP r es
o titut
Ins

PHYSICS
FOR JEE MAIN & ADVANCED
SECOND
EDITION

Exhaustive Theory
(Now Revised)

Formula Sheet
9000+ Problems
based on latest JEE pattern

2500 + 1000 (New) Problems


of previous 35 years of
AIEEE (JEE Main) and IIT-JEE (JEE Adv)

5000+Illustrations and Solved Examples


Detailed Solutions
of all problems available

Topic Covered Plancess Concepts


Tips & Tricks, Facts, Notes, Misconceptions,
Electric Potential Key Take Aways, Problem Solving Tactics
and Capacitance
PlancEssential
Questions recommended for revision
19. ELECTRIC
POTENTIAL AND
C A PA C I TA N C E
1. INTRODUCTION
Associating a potential energy with a force allows us to apply the principle of the conservation of mechanical
energy to closed systems involving the force. The principle allows us to calculate the results of experiments for
which force calculations alone would be very difficult. In this chapter, we first define this type of potential energy
and then put it to use. We will also discuss the behavior of conductors which can store the charges in it, in the
presence of the electric field.

2. ELECTRIC POTENTIAL
An electric field at any point can be defined in two different ways:

(i) by the field strength E , and
(ii) by the electric potential V at the point under consideration.

Both E and V are functions of position and there is a fixed relationship between these two. Of these the field
strength E is a vector quantity while the electric potential V is a scalar quantity. The electric potential at any point
in an electric field is defined as the potential energy per unit charge, and similarly, the field strength is defined as
the force per unit charge. Thus,
U
V= or U = q0 V
q0
The SI unit of potential is volt (V) which is equal to joule per coulomb. So,
1V 1volt
= = 1J=/ C 1 joule / coulomb
According to the definition of potential energy, the work done by the electrostatic force in displacing a test charge
q0 from point a to point b in an electric field is defined as the negative of change in potential energy between them,
or ∆U =−Wa−b ∴ Ub − Ua = −Wa−b
Dividing this equation by qο ,

Ub Ua W W U
− − a−b or Va − Vb =a−b since V =
=
qο qο qο qο qο

Thus, the work done per unit charge by the electric force when a charge body moves from point a to point b is equal
to the potential at point a minus the potential at point b. We sometimes abbreviate this difference as Vab
= Va − Vb .
1 9 . 2 | Electric Potential and Capacitance

Another way to interpret the potential difference Vab is that it is , equal to the work that must be done by an external
force to move a unit positive charge from point b to point a against the electric force. Thus,
( Wb−a )externalforce
Va − Vb =
qο

PLANCESS CONCEPTS

1 1 q dq
In the equation V =
4 πεο
∑ ri
4 πε ∫ r
or V =
, if the whole charge is at equal distance ro from the
i i ο
1 qnet
point where V is to be evaluated, then we can write, V = . , where qnet is the algebraic sum of
4 πεο rο
all the charges of which the system is made.
Vaibhav Gupta (JEE 2009 AIR 54)

Illustration 1: Consider a non-conducting rod of length  having a uniform  y


charge density λ . Find the electric potential at P at a perpendicular distance
P
y above the midpoint of the rod  (JEE MAIN)

Sol: The potential due to the small length element dx of the rod at a point p ’

dq 1
at distance r apart is given by=
dV × where dq is the charge on dx. y
4 πεo r r’
∂V
The electric field due to the rod is given by E = − .
∂r x’
x
Consider a differential element of length dx’ which carries a charge dq = λdx' O dx’
. As shown in Fig.19.1, the source element is located at (x’, 0), while the field L
point P is located on the y-axis at (0,y).
Figure 19.1

(x )
1/2
'2 2
The distance from dx’ to P =
is r +y .
1 dq 1 λdx'
Its contribution to the potential is given=
by dV =
4 πεο r 4 πεο
(x )
1/2
'2
+ y2

Taking V to be zero at infinity, the total potential due to the entire rod is
 /2 V(y)/VO
λ  /2 dx' λ
=V ∫− /2 = ln  x'+ x' 2 + y 2  
4 πεο '2
x +y 2 4 πε ο   −  /2 4

3
 
 (  / 2) + (  / 2) + y
2 2
λ 
= ln   2
4 πεο 
 (
−  / 2) + (  / 2) + y2
2

 1

Where we have used the integration formula -4 -2 2 4


y/t

dx'
∫ = In  x' + x' 2 + y 2  Figure 19.2
'2
x +y 2  

A plot of V ( y ) / Vο , where Vο = λ / 4 πεο , as a function of y /  is shown in Fig. 19.2. (Electric potential along the axis
that passes through the midpoint of a non-conducting rod.)
P hysi cs | 19.3

In the limit   y , the potential becomes


 2   
 (  / 2 ) +  / 2 1 + ( 2y /  )   1 + 1 + ( 2y /  )
2
λ λ 
V = ln  4 πε ln 
4 πεο  
 (
−  / 2 ) +  / 2 1 + ( 2y /  )   −1 + 1 + ( 2y /  )
2 2
ο 
  

λ  2  λ  2  λ 
≈ ln   = ln   = ln  
4 πεο  2y 2 / 2  4 πεο  y 2  2πεο  y 

∂V λ /2
The corresponding electric field can be obtained as Ey =
− =
∂y 2πεο y
(  / 2)
2
+ y2

Illustration 2: Consider a uniformly charged ring of radius R and charge 


density λ . What is the electric potential at a distance z from the central Z
axis?  (JEE MAIN)
r
The Fig. 19.3 shows a non-conducting ring of radius R with uniform
charge density λ Z
y

Sol: The point lies along the axis of the ring hence, the potential at the R
dq dp
point due to the charge on the ring is given by= ∫=
dV ∫ where
V x
4 πε r o Figure 19.3
r is the distance of point from ring. As the point lies on the z axis, the
∂V
field due to ring is given by EZ = − .
∂z
 Rdφ ' on the ring. The element carries a charge dq =
Consider a small differential element d= λ d =λRdφ ' , and its
1 dq 1 λRdφ '
contribution to the electric potential at =
P is dV =
4 πεο r 4 πεο R 2 + z2
1 λR 1 2πλR 1 Q
The electric potential at P due to the entire ring is =
V ∫ dV
= ∫ d=
φ' =
4 πεο 2
R +Z 2 4 πεο 2
R +z 2 4 πε ο R + z2
2

Where we have substituted Q = 2πRλ for the total charge on the ring. In the limit z  R,
1 Q
The potential approaches its “point-charge” limit: V≈
4 πεο z
∂V ∂  1 Q  1 Qz
The z-component of the electric field may be obtained as E2 =
− − 
= =
∂z  4 πεο 2 
∂z 4
( )
2 πε 3/2
R +z  ο R 2 + z2

Illustration 3: Consider a uniformly charged disk of radius R and charge density σ z

lying in the xy-plane. What is the electric potential at a distance z from the P
central axis?  (JEE ADVANCED)
dq 
r z
Sol: The disc can be assumed to be composed of many co-centric rings. Thus
dq
the potential due to small ring is dV = where r is the distance from the R
r’
y
4 πεor dr’
surface of ring. As the point lie on the z axis the field due to ring is given by
∂V
EZ = − . x
∂z Figure 19.4
1 9 . 4 | Electric Potential and Capacitance

Consider a small differential circular ring element of radius r’ and width dr’. The charge on the ring is
dq' =σdA' =σ ( 2πr 'dr ' ) . The field at point P located along the z-axis a distance z from the plane of the disk is to
(r )
1/2
'2
be calculated. From the Fig. 19.4, we also see that the distance from a point on the ring to P is=r + z2 .

1 dq 1 σ ( 2πr 'dr ' )


Therefore, the contribution to the electric potential at =
P is dV =
4 πεο r 4 πεο r' 2 + z 2

By summing over all the rings that make up the disk, we have
R
σ R 2πr 'dr ' σ  '2 2 σ  2
=V ∫ο = r + z= R + z2 − z 
4 πεο '2 2 2ε 
  ο 2ε 
 
r +z ο ο

1/2
2 2
 R2   R2 
In the limit z  R , R +Z = z 1 +  = z 1 + + ....  ,
 z 2   2z 2 
  
And the potential simplifies to the point-charge limit:

V≈
σ R2
. =
1 σ πR
=
2
( )
1 Q
2εο 2 z 4 πεο z 4 πεο z

As expected, at large distance, the potential due to a non-conducting charged disk is the same as that of a point
charge Q. A comparison of the electric potentials of the disk and a point charge is shown in Fig. 19.5.

V/VO
6
5
4
3 Point charge
2 Disk
1
Z/R’
-3 -2.5 -2 -1.5 -1 -0.5 0.5 1 1.5 2 2.5 3

Figure 19.5

The electric potential is measured in terms of=


Vο Q / 4 πεοR .
Note that the electric potential at the center of the disk (z=0) is finite, and its value is
σR Q R 1 2Q
Vc
= = .= = 2Vο
2εο πR 2εο 4 πεο R
2

This is the amount of work that needs to be done to bring a unit charge from infinity and place it at the center of
the disk.
The corresponding electric field at P can be obtained as:

∂V σ z z 
Ez =
− =  − 
∂Z 2εο  z R 2 + z2 

In the limit R  z, the above equation becomes E2 =
σ / 2εο , which is the electric field for an infinitely large non-
conducting sheet.

Illustration 4: Consider a metallic spherical shell of radius ‘a’ and charge Q, as shown in Fig. 19.6.
(a) Find the electric potential everywhere.
(b) Calculate the potential energy of the system. (JEE MAIN)
P hysi cs | 19.5

Sol: The sphere is symmetrical body. Thus the potential at the surface is constant while  ++++++
the potential changes outside of the sphere at a distance a < r < ∞ . ++ +
+  +
 Q + E=0 +
  r̂, r > a + +
E =  4 πεοr 2 + +
 + +
 0, r<a + +
+ +
B 
++ +
The electric potential may be calculated by V − V =− ∫ E.ds ++ ++
B A A ++++
r Q 1 Q Q Figure 19.6
For r>a, we have V (r ) − V ( ∞ ) =− ∫ dr ' = =k e

4 πεοr '2 4 πεο r r

We have chosen V ( ∞ ) =0 as our reference point. On the other hand, for r<a,  V
the potential becomes KeQ
KeQ r
a r
V (r ) − V ( ∞ ) =− ∫ drE (r > a) − ∫ drE(r < a)
a
∞ a

a Q 1 Q Q r
− ∫ dr
= = ke
= a

4 πεοr 2 4 πεο a a

A plot of the electric potential is shown in Fig. 19.7. Note that potential V is Figure 19.7
constant inside a conductor.

Illustration 5: An insulated solid sphere of radius a has a uniform charge density ρ . Compute the electric potential
everywhere.  (JEE MAIN)
Sol:
 Q
r̂, r > a a
  4 πε r 2
E= ο
 P2
 Qr r̂, r < a
 4 πε r 3 r
 ο
P1
For r > a,
Figure 19.8
r Q 1 Q Q
V1 (r ) − V ( ∞ ) =− ∫ dr ' = =k e

4 πεοr '2 4 πεο r r

On the other hand, the electric potential at P2 inside the sphere is given by V
a r a Q r Qr 3keQ
V2 (r ) − V ( ∞ ) =− ∫ drE (r > a) − ∫ E (r < a) =− ∫ dr − ∫ dr ' r' 2a
∞ a ∞ 2 a 3
4 πεοr 4 πεο a
 ke Q
1 Q r  2 a
=
1 Q

1 Q1 2
4 πεο a 4 πεο a 2
3
2
r − a= ( 
8πεο a 
)
3 − 2 
a 
a r
Q r2  Figure 19.9
= ke  3 − 
2a  a2 
A plot of electric potential as a function of r is given in Fig. 19.9.
1 9 . 6 | Electric Potential and Capacitance

2.1 Deriving Electric Field from the Electric Potential



In previous equations,
 we established the relation between E and V. If we consider two points which are separated
by a small distance ds , the following differential form is obtained:
 
dV = −E.ds
 
In Cartesian coordinates, E = Ex ˆi + Ey ˆj + E2kˆ and ds = dxiˆ + dyjˆ + dzkˆ , we have
dV = (E ˆi + E ˆj + E kˆ ) . (dxiˆ + dyjˆ + dzkˆ ) =
x y 2 Ex dx + Ey dy + Ez dz
Which implies

∂V ∂V ∂V
Ex =
− ,E =
− ,E =

∂x y ∂y z ∂z

By introducing a differential quantity called the “del (gradient) operator”


∂ ˆ ∂ ˆ ∂ ˆ
=
∇ i+ j+ k
∂x ∂y ∂z
The electric field can be written as
  ∂V ˆ ∂V ˆ ∂V ˆ   ∂ ∂ ∂ 
E = Ex ˆi + Ey ˆj + E2kˆ = −  i+ j+ k  = −  ˆi + ˆj+ kˆ  V = −∇V
 ∂x ∂y ∂z   ∂x ∂y ∂z 

E = −∇V

3. EQUIPOTENTIAL SURFACES
The equipotential surfaces in an electric field have the same basic idea as topographic maps used by civil engineers
or mountain climbers. On a topographic map, contour lines are drawn passing through the points having the
same elevation. The potential energy of a mass m does not change along a contour line as the elevation is same
everywhere.
By analogy to contour lines on a topographic map, an equipotential surface is a three dimensional surface on which
the electric potential V is the same at every point on it. An equipotential surface has the following characteristics.
(a) Potential difference between any two points in an equipotential surface is zero.
(b) If a test charge q0 is moved from one point to the other on such a surface, the electric potential energy q0V
remains constant.
(c) No work is done by the electric force when the test charge is moved along this surface.
(d) Two equipotential surfaces can never intersect each other because otherwise the point of intersection will
have two potentials which is of course not acceptable.
(e) As the work done by electric force is zero when a test charge is moved along the equipotential surface, it

follows that E must be perpendicular to the surface at every point so that the electric force qο E will always
be perpendicular to the displacement of a charge moving on the surface, causing the work done to be 0.
Thus, field lines and equipotential surfaces are always mutually perpendicular. Some equipotential surfaces
are shown in Fig. 19.10.
The equipotential surfaces are a family of concentric spheres for a point charge or a sphere of charge and are a
family of concentric cylinders for a line of charge or cylinder of charge. For a special case of a uniform field, where
the field lines are straight, parallel and equally spaced the equipotential are parallel planes perpendicular to the
field lines.
P hysi cs | 19.7

10V 10V
20V 20V
30V 30V
40V 40V 
E
+ -

40V 30V 20V

Figure 19.10

PLANCESS CONCEPTS

While drawing the equipotential surfaces we should keep in mind the two main points.
• These are perpendicular to field lines at all places.
• Field lines always flow from higher potential to lower potential.
Anurag Saraf (JEE 2011 AIR 226)

Illustration 6: Suppose the electric potential due to a certain charge distribution can be written in Cartesian
Coordinates as V ( x,=
y, z ) Ax2 y 2 + Bxyz where A,B and C are constants. What is the associated electric field?
 (JEE MAIN)
∂V
Sol: The field is given by Er = − where r is the respective Cartesian co-ordinate.
∂r
∂V ∂V ∂V
Ex = − = −2Axy 2 − Byz ; Ey = − =−2Ax2 y − Bxz ; Ez = − = −Bxy
∂x ∂y ∂z

Therefore, the electric field is E = ( ) ( )
−2Axy 2 − Byz ˆi − 2Ax2 y + Bxz ˆj − Bxykˆ

4. ELECTRIC POTENTIAL ENERGY


The electric force between two charges is directed along the line of the charges and is proportional to the inverse
square of their separation, the same as the gravitational force between two masses. Like the gravitational force, the
electric force is conservative, so there is a potential energy function U associated with it.
When a charged particle moves in an electric field, the field exerts a force that can do work on the particle. This
work can always be expressed in terms of electric potential energy. Just as gravitational potential energy depends
on the height of a mass above the earth’s surface,
 electric potential energy depends on the position of the charged
particle in the electric field. When a force F acts on a particle that moves from point a to point b, the work Wa→b
b  b
done by the force is given by, = Wa→b ∫= F.ds ∫ F cos θ ds
a a

Here Wa→b is the work done in displacing the particle from a to b by the conservative force (here electrostatic)
not by us. Moreover we can see from Eq. (i) that if Wa→b is positive, the change in potential energy ∆U is negative
and the potential energy decreases. So, whenever the work done by a conservative force is positive, the potential
energy of the system decreases and vice-versa. That’s what happens when a particle is thrown upwards, the work
done by gravity is negative, and the potential energy increases.
1 9 . 8 | Electric Potential and Capacitance

4.1 Potential Energy in a System of Charges


If a system of charges is assembled by an external agent, then ∆U =−W =+ Wext .

P
That is, the change in potential energy of the system is the work that must be put
in by an external agent to assemble the configuration. A simple example is lifting a
mass m through a height h. The work done by an external agent you, is +mgh (The q2
gravitational field does work -mgh). The charges are brought in from infinity without r12
acceleration i.e. they are at rest at the end of the process. Let’s start with just two
charges q1 and q2. Let the potential due to q1 at a point P be V1 (See Fig. 19.11).

The work W2 done by an agent in bringing the second charge q2 from infinity to P q1
is then W2 = q2 V1 . (No work is required to set up the first charge and W1 = 0). Since
V1 q1 / 4 πε0r12 , where q1 and r12 is the distance measured from q1 to P, we have
= Figure 19.11

1 q1q2
U=
12 W
=2
4 πε0 r12
If q1 and q2 have the same sign, positive work must be done to overcome P
the electrostatic repulsion and the potential energy of the system is
positive, U12 > 0. On the other hand, if the signs are opposite, then r23
U12 < 0 due to the attractive force between the charges. To add a q2
r12
third charge q 3 to the system, the work required is
q3  q1 q2 
W3= q3 ( V1 + V2 )=
q2
 + 
4 πεο  r13 r23 
r13
The potential energy of this configuration is then q1

1  q1q2 q1q3 q2q3  Figure 19.12


U = W2 + W3 =  + +  = U12 + U13 + U23
4 πεο  r12 r13 r23 

The equation shows that the total potential energy is simply the sum of the contributions from distinct pairs.
1 N N qiq j
Generalizing to a system of N charges, we have U = ∑∑
4 πεο =i 1=j 1 rij
j >i
Where the constraint j>i is placed to avoid double counting each pair. Alternatively, one may count each pair twice
and divide the result by 2. This leads to
 
1 N N qiq j 1 N  1 N q j  1 N
=U = ∑∑
8πεο =i 1=j 1 rij
∑ q=  ∑  ∑ q V (r )i
2=i 1 i  4 πεο =j 1 rij  2=i 1 i
 
j≠i  j≠i 

Where V (ri ) , the quantity in the parenthesis is the potential at ri (location of qi) due to all the other charges.

4.2 Continuous Charge Distribution



q
If the charge distribution is continuous, the potential at a point P can be found by summing 
>

r
over the contributions from individual differential elements of charge dq.

Consider the charge distribution shown in Fig. 19.13. Taking infinity as our reference point with
1 dq r
zero potential, the electric potential at P due to dq is dV =
4 πεο r
P
1 dq 
Summing over contributions from all differential elements, we have V = ∫ E
4 πεο r
Figure 19.13
P hysi cs | 19.9

Illustration 7: Find the potential due to a uniformly charged sphere of radius R and charge 
per unit volume ρ at different points in space.  (JEE MAIN) 

R
q ρR3
Sol: The field due to uniformly charged solid sphere
= is E = . The potential
4 πεor 2 3εor 2
inside the sphere is always constant while outside sphere at a distance R < r < ∞ is given
r r Figure 19.14
by  V1  =
R ∫R Edl
For a point outside the sphere, we can integrate the electric field outside to obtain an expression for the electric
field outside.
r  
3
r ρR ρR 3
∫ V= − ∫ E.dl =

−∫

3εο x2
dx =
3εοr

The dots represent a spherically symmetric distribution of charge of radius R, whose 


volume charge density ρ is a constant.
For finding the potential at an inside point, we integrate as before. Keep in mind
that the lower limit in this integration cannot be infinity, as infinity does not lie v3kQ
inside the sphere. But lower limit can be assumed to be R where potential is not 2R
zero but a known value.

Vin r   r ρx (
ρ R2 − r2 ) ;V = ρR 2 (
ρ 3R 2 − r 2)
∫VR
dV − ∫ E.dl =
=
R
−∫
R 3ε
dx =
6ε ο R
3εο
⇒ Vr =
6ε ο
.
r
ο -R R
This potential is plotted in Fig. 19.15. Figure 19.15

Illustration 8: A non-conducting disc of radius a and uniform positive surface charge density σ is placed on the
ground with its axis vertical. A particle of mass m and positive charge q is dropped, along the axis of the disc from
a height H with zero initial velocity. The particle has q / m =ε
4 οg / σ
(a) Find the value of H if the particle just reaches the disc.
(b) Sketch the potential energy of the particle as a function of its height and find its equilibrium position. 
 (JEE ADVANCED)

Sol: It is known that, for non conducting disc the potential at a point situated at height H above  P qm
σ  2
the disc is given by=
Vp a + H2 − H where a is the radius of the disc. When the charge of
2εο   H
the particle of mass m falls along the axis of the disc, the change in the gravitational portential
energy is equal to gain in its electric potential energy. As we required minimum H, the kinetic a O
energy, will be zero.
σ  2 Figure 19.16
As we have derived in the theory,=
Vp a + H2 − H
2εο  
σa
Potential at centre, (O) will be Vο = (H=0)
2εο
(a) Particle is released from P and it just reaches point O. Therefore, from observation of mechanical energy
0 because K=
Decrease in gravitational potential energy = increase in electrostatic potential energy ( ∆KE = i K=
f 0)

 q  σ  2 2 
or gH   
∴ mgH = q  Vο − Vρ  =  a − a + H + H  … (i)

 m   2εο   
1 9 . 1 0 | Electric Potential and Capacitance

q 4εο g qσ
= ∴ 2g
=
m σ 2εοm
Substituting in Eq. (i), we get

H H
= 2g a + H − a2 + H2  or = ( a + H) − a2 + H2 or
gH a2 + H2 =a +
  2 2
H2 3 4
or a2 + H2 = a2 +
4
+ aH or H2 = aH or H = a and H = 0 ∴ H =
4 3
( 4 / 3) a
(b) Potential energy of the particle at height H = Electrostatic potential energy + gravitational potential energy
U qV + mgH
=
Here V=potential at height H
σq  2
=U a + H2 − H + mgH  … (ii)
2εο  

−dU
At equilibrium position,
= F = 0
dH
Differentiating E.q. (ii) w.r.t.H

σq  1  1   σq 
or mg +   ( 2H) − 1 =0  = 2mg
2εο  2  a2 + H2   2εο 

 H  2H
∴ mg + 2mg  − 1 =0 or 1 + −2 =0
 a2 + H2  a2 + H2

2H H2 1 a
= 1 or = or 3H2 = a2 or H =
2
a +H 2 2
a +H 2 4 3
U
From Eq. (ii), we can see that,
a
U = 2mga at H = 0 and
= U U=
min 3mga at H = 2mga
3
Therefore, U-H graph will be as shown. 3mga
a
Note that at H = , U is minimum.
3 O aA 3 H
a
Therefore, H = is stable equilibrium position. Figure 19.17
3

5. ELECTRIC DIPOLE
An electric dipole is a system of equal and opposite charges separated
 by a fixed distance. Every electric dipole is
characterized by its electric dipole moment which is a vector P directed from the negative to the positive charge.
The magnitude of dipole moment is, P=(2a)q; Here, 2a is the distance between the two charges.

5.1 Electric Potential and Field Due to an Electric Dipole



Consider an electric dipole lying along positive y-direction with its center at origin. -q P
+q

P = 2aqjˆ 2a
Figure 19.18
P hysi cs | 19.11

The electric potential due to this dipole at point A(x, y, z) as shown y


is simply the sum of the potentials due to the two charges. Thus, 2 2
x + z + (y-a)
2 A (x, y, z)
 
1  q q 
V  − 
+q
4 πεο  2 +
x + ( y − a) + z2 x2 + ( y + a) + z2
2 2 2 2 2
 x + z + (y+a)
  a

By differentiating this function, we obtain the electric field of the a


dipole. -q -
  z
 
∂V q  x x  Figure 19.19
Ex
= =  −
∂x 4 πεο   2 3/2 3/2 
2  x2 + y + a 2 + z2  
  x + ( y − a) + z  ( )
2
  

 
 
∂V q  y − a y + a 
E= =  −
y
∂y 4 πεο   2 3/2 3/2 
2  x2 + y + a + z2  
  x + ( y − a) + z  ( )
2 2
  

 
 
∂V q  z x 
Ez
= =  − 
∂z 4 πεο   2 2
3/2
 2
3/2

  x + ( y − a) + z  x + ( y + a) + z
2 2 2
  
Special Cases
(i) On the axis of the dipole (say, along y-axis) X=0, z=0

q  1 1  2aq
∴V
=  =
− 
4 πεο  y − a y + a  4 πεο y 2 − a2 ( )
p
or V= (as 2aq = p) i.e., at a distance r from the Centre of the dipole(y=r)
4 πεο y 2 − a2( )
p p
V≈ or Vaxis = (for r  a )
( 2
4 πεο r − a 2
) 4 πεοr 2

V is positive when the point under consideration is towards positive charge and negative if it is towards negative charge.
Moreover the components of electric field are as under,

Ex 0,E
= = z 0 (as x = 0, z = 0)

 
q  1 1  4ayq 1 2py
and Ey
= − = or Ey =
4 πεο  y − a
 ( ) ( y + a)  4πεο y 2 − a2 ( ) 4 πεο
(y )
2 2 2 2
2
− a2

Note that Ey is along positive y-direction or parallel to P . Further, at a distance r from the centre of the dipole (y=r).

1 2pr 1 2p
Ey = or Eaxis ≈ . for r  a
4 πεο
(r ) 4 πεο r3
2
2
− a2

(ii) On the perpendicular bisector of dipole


Say along x-axis (it may be along z-axis also). y = 0, z = 0
1 9 . 1 2 | Electric Potential and Capacitance

1  q q 
=∴V  =
−  0 or V⊥ bi sector = 0
4 πεο  x2 + a2 x2 + a2 

Moreover the components of electric field are as under,=


Ex 0=
Ez 0
 
q  −a a  −2aq 1 P
and
= Ey =  − = ; Ey = .
3/2 
4 πεο  2
( ) ( ) ( ) 4 πεο
(x )
3/2 3/2 3/2
x + a2 x2 + a2  4 πεο x2 + a2 2
+ a2
 

Here, negative sign implies that the electric field is along negative y-direction or antiparallel to P . Further, at a
distance r from the Centre of dipole (x = r), the magnitude of electric field is,

1 P 1 P
E= or E⊥ bisector ≈ . (for r  a )
4 πεο
(r ) 4 πεο r3
3/2
2
+ a2

5.2 Force on Dipole



Suppose an electric dipole of dipole moment P = 2aq is  
 E
placed in a uniform electric field E at an angle θ . Here, θ +q  
    F1 P
is the angle between P and E . A force F1 = qE will act on a
A
positive charge and
    O
F2 = −qE on negative charge. Since, F1 andF2 are equal in a

   -q

magnitude but opposite in direction. Hence, F1 + F2 = 0 or F2
E

Fnet = 0 B

Thus, net force on a dipole in uniform electric field is zero. Figure 19.20
While in non-uniform electric field it may or may not be zero.

5.3 Torque on Dipole


     
The torque of F1 about O, τ1= OA × F1= q OA × E ( )
       
And torque of F2 about O is, τ 2 =OB × F 2 =−q OB = ×E q BO × E ( ) ( )
The net torque acting on the dipole is,
              
( ) (
τ = τ1 + τ2 = q OA × E + q BO × E = q OA × BO = ) (
× E q BA × E ) ( ) or τ =P xE

Thus, the magnitude of torque= is τ PEsin θ . The direction of torque is perpendicular to the plane of
 paper inwards.
Further this torque is zero at θ =0 or θ =180 , i.e., when the dipole is parallel or antiparallel to E and maximum
ο ο

at θ =90ο .

5.4 Potential Energy of Dipole


   
When an electric dipole is placed in an electric field E , a torque τ = P × E acts on it.  +q
If we rotate the dipole through a small angle dθ , the work done by the torque is, o
90
dW =τdθ ; dW =−PEsin θdθ
The work is negative as the rotation dθ is opposite to the torque. The change in
electric potential energy of the dipole is therefore. -q
dU =−dW =PEsin θdθ Figure 19.21
P hysi cs | 19.13

Now, at angle θ = 90°, the electric potential energy of the dipole may be assumed to be zero as net work done by
the electric forces in bringing the dipole from infinity to this position will be zero.
Integrating, =
dU PEsin θdθ

From 900 to θ , we have =


θ
∫ ο dU
90
θ
∫90ο PEsin θdθ ( )
or U ( θ ) − U 90ο = PE  − cos θ  ο
θ
90
 
∴ U ( θ ) = −PEcos θ = −P ⋅ E
If the dipole is rotated from an angle θ1 to θ2 then

Work done by external forces = U ( θ2 ) − U ( θ1 ) or Wext.forces =


−PEcos θ2 − ( −PEcos θ1 )

= PE ( cos θ1 − cos θ2 ) and work done by electric forces, Welectric force =− Wext.force =PE ( cos θ2 − cos θ1 )
Or Wext.forces

Illustration 9: Figure 19.22 is a graph of Ex, the x component of the electric field,  EX(V/m)
versus position along the x axis. Find and graph V(x). Assume V = 0; V at x = 0m.  2000
(JEE MAIN)
1000 V=-Area
Sol: The potential V(x) is given by V=E•dx=Area of the shaded region of the graph.
The potential difference is the negative of the area under the curve. 0 x(m)
 0 1 x
Ex is positive throughout this region of space, meaning that E points in the positive
x direction. Figure 19.22

If we integrate from x-0, then Vi = V(x − 0) − 0 . The potential for x>0 is the negative of the triangular area under the
Ex curve. We can see that Ex = 1000xV / m , where x is in meters(m). Thus,
Vt= V ( x )= 0 − (Area under the Ex curve) v(v)
x
1 1
− ( x )(1000x ) =
1 2
× base × height = −500x2 V.
2 2
Figure 19.23 shows that the electric potential in this region of space is parabolic, 1000
decreasing from 0 V at x=0 m to -2000V at x=2m. v(x) = -500x V
2

2000
The electric field points in the direction in which V is decreasing. We’ll soon see that
as this is a general rule.
Figure 19.23

Illustration 10: The electric potential at any point on the central axis of a uniformly charged disk is given by
σ  2 2 
= V  z + R − z .
2εο  
Starting with this expression, derive an expression for the electric field at any point on the axis of the disk. 
 (JEE MAIN)
∂V
Sol: As the point lie on the z axis the field due to ring is given by EZ = − .
∂z

Conceptualize/Classify: We want  the electric field E as a function of distance z along the axis of the disk. For
any value of z, the direction of E must be along that axis because the disk has circular symmetry about that axis.

Thus, we want the component Ez of E in the direction of z. This component is the negative of the rate at which the
electric potential changes with distance z.
Compute: Thus, from the previous equations, we can write

∂V σ d 2 σ  z 
Ez =
− =
− 2
 z +R =− z  ; 1 − .
∂z 2εο dz   
2εο  
z + R2
2

1 9 . 1 4 | Electric Potential and Capacitance

6. CAPACITOR
A capacitor is a combination of two conductors placed close to each other. It is used to store energy electrostatically
in an electric field. The ‘non-conducting’ dielectric acts to increase the capacitor’s charge capacity. Capacitors are
widely used as parts of electrical circuits in many common electrical devices. Unlike a resistor, a capacitor does not
dissipate energy. Instead, a capacitor stores energy in the form of an electrostatic field between its plates.
The physics of capacitors can be utilized to any scenario involving electric fields. For example, Earth’s atmospheric
electric field is analyzed by meterologists as being produced by a huge spherical capacitor that partially discharges
via lightning. The charge that is collected as they slide along snow can be modeled as being stored in a capacitor
that frequently discharges as sparks.

7. CAPACITANCE
There are 2 conductors in a capacitor. One conductor has positive charge (positive plate)
and the other has an equal and opposite negative charge (negative plate). The charge
on the positive plate is called the charge on the capacitor and the potential difference
between the plates is called the potential of the capacitor. For a given capacitor, the
charge Q on the capacitor is proportional to the potential difference V between the Figure 19.24
plates Thus, Q∝V or, Q=CV.
The proportionality constant C is called the capacitance of the capacitor. It depends on the shape, size and
geometrical placing of the conductors and the medium between them.
The SI unit of capacitance is coulomb per volt which is written as Farad. The symbol F is used for it.

Illustration 12: A Capacitor gets a charge of 60µC when it is connected to a battery of emf 12V. Calculate the
capacitance of the capacitor. (JEE MAIN)
Q
Sol: The capacitance is given by C =
V
The potential difference between the plates is the same as the emf of the battery which is 12V.
Q 60µC
Thus, the capacitance is C = = = 5µF.
V 12V

8. CALCULATING CAPACITANCE
To calcutate the capacitance of a capacitor once we know its geometry:-
(a) Assume a charges q and –q on the plates ;
(b) Calculate the electric field E between the plates in terms of this charge, using Gauss’ law;
(c) Knowing E , calculate the potential difference V between the plates.
(d) Calculate C.

9. TYPES OF CAPACITORS

9.1 Parallel Plate Capacitor


d
A parallel-plate capacitor contains two large plane plates parallel to each other  +Q
with a small separation between them. Suppose, the area of each of the surfaces -Q
is A and the separation between the two plates is d. Also, assume that vacuum
fills the space between the plates. Figure 19.25
P hysi cs | 19.15

Q
The magnitude of charge density on each of these surfaces is given by σ =
A
B
Let us draw a small area A parallel to the plates and in between them,  V+ A’ +Q
draw a cylinder with A as a cross-section and terminate it by another + + + + + + + +
symmetrically situated area A’ inside the positive plate. The flux through d A
A’ and through the curved part inside the plate is zero as the electric E
- - - - - - - -
field is zero inside a conductor. The flux through the curved part outside V- A -Q
the plates is also zero as the direction of the field E is parallel to this
surface. Figure 19.26

  Q
The flux through A is φ = E. ∆ A = E∆A . The only charge inside the Gaussian surface is ∆Q = σ∆A = ∆A.
A
  Q Q
From Gauss’s law, ∫ E.dS
= Qin / ε0 or, E∆A= ∆A ; or, E = .
ε0 A ε0 A
B 
The potential difference between the plates is V = V+ − V− =− ∫ E.dr.
A
  B
Qd
E.dr = ∫ E dr =
−Edr and V = Ed =
ε0 A
A
Q Qε0 A ε0 A
The capacitance of the parallel-plate capacitor is =
C = = .
V Qd d

Illustration 13: Calculate the capacitance of a parallel-plate capacitor having 20cm x 20cm square plates separated
by a distance of 1.0mm.  (JEE MAIN)
ε0 A
Sol: The capacitance is given by C = .
d
ε A 8.85 × 10−12 Fm−1 × 400 × 10−4 m2
The capacitance is C =0 = 3.54 × 10−10 F =
= 350pF.
d 1 × 10−3 m

9.2 Cylindrical Capacitor V-


A cylindrical capacitor consists of a solid or a hollow cylindrical conductor  V+
surrounded by another coaxial hollow cylindrical conductor. Let l be the length of
the cylinders and R1 and R2 be the radii of the inner and outer cylinders respectively.
If the cylinders are long as compared to the separation between them, the electric
field at a point between the cylinders will be radial and its magnitude will depend x
P
L
R1
only on the distance of the point from the axis. E r R2

To calculate the electric field at the point P, at a distance r, draw a coaxial cylinder
Q
of length x through the point. A Gaussian surface is made by the cylinder and its
two cross sections. The flux through the cross sections is zero as the electric field
is radial wherever it exists and hence is parallel to the cross sections. -Q
 
The flux through the curved part is φ= E∫ dS= E2πrx = ⇒ φ ∫= E .dS ∫ EdS . Figure 19.27

Q Q  Q
The charge enclosed by the Gaussian surface is =
Qin x. , E2=
πrx  x  / ε0 or,
= E
l  l  2 πε 0rl

B  R1
Q Q R
The potential difference between the cylinders is V = − ∫ E.dr =
V+ − V− = − ∫ dr = ln 2 .
A R2
2πε0rl 2πε0l R1
Q 2πε0l
The capacitance is =
C =
V ln R / R
2 ( 1
)
1 9 . 1 6 | Electric Potential and Capacitance

9.3 Spherical Capacitor


A Spherical Capacitor consists of two concentric spherical shells, of radii a and b. As a Gaussian surface we draw a

sphere of radius r concentric with the two shells q = ε0EA = ( )


ε0E 4 πr 2 , in which the area of the spherical Gaussian
1
surface is given by 4 πr 2 . We solve equation for E, obtaining E = , which we recognize as the expression for
4 πε0r 2
the electric field due to a uniform spherical charge distribution
+
q a dr q 1 1  q b −a
∫ Eds =
V= − ∫b r2 =  − =   , Substituted – dr for ds

4 πε0 4 πε0  a b  4 πε0  ab 

ab
C= 4 πε0 (Spherical capacitor).
b−a

Isolated Sphere
We can assign a capacitance to a single isolated spherical conductor of radius R by assuming that the “missing
a
plate” is a conducting sphere of infinite radius. C= 4 πε0 .
1−a/b
If we then let b ∞ and substitute R for a, we find C= 4 πε0R (isolated sphere)

PLANCESS CONCEPTS

• Earth can be considered as a capacitor with infinite capacitance.


• When a conductor is grounded and charge flows into the earth, the ground is still at potential zero.
GV Abhinav (JEE 2012 AIR 329)

Illustration 14: Three identical metallic plates are kept parallel to one another at a separation of a and b. The outer
plates are connected by a thin conducting wire and a charge Q is placed on the central plate. Find final charges on
all the six surfaces. (JEE ADVANCED)
Sol: The charge on any plate is computed using q = CV where C is the capacitance  1 2 3 4 5 6

of the plate and V is the potential difference applied on it. The chrge induced in
plates A and C are such that total q(A) + q(C) =0. Plates A and C are at the same
potential.
Let the charge distribution in all the six faces be as shown in Fig. 19.28. While a b
distributing the charge on different faces, we have used the fact that opposite faces
have equal and opposite charges on them.
Figure 19.28
Net charge on plates A and C is zero.
0 Or q2 + q3 =
Hence, q2 − q1 + q3 + q1 − Q = Q ….. (i) 
(Q-q1)
q2 -q2 -q1 q3
Further A anc C at same potentials. Hence, VB − VA = VB − VC or E1a = E2b
(q-Q1)
q1 Q − q1 Qb
.a = .b (A= Area of plates); q1a = ( Q − q1 ) b ∴ q1 = ….. (ii)
Aε0 Aε0 a+b
E1
Electric field inside any conducting plate (say inside C) is zero. Therefore, E2
q2 q1 q1 Q − q1 q1 − Q q3 a b
− + + + − 0
=
2Aε0 2Aε0 2Aε0 2Aε0 2Aε0 2Aε0
Figure 19.29
P hysi cs | 19.17

Qb Q
Solving these three equations, we get q=
1 , q=
2 q=
3
a+b 2

10. COMBINATION OF CAPACITORS


+C- +C- +C- +C-n
The desired value of capacitance can be obtained by combining more than one  Q Q Q Q
capacitor. We shall discuss two ways in which more then one capacitor can be V1 V2 V3 Vn
connected.

10.1 Series Combination of capacitors:


+ -
There is equal amount of charge Q deposited on each capacitor; but the potential V
difference between their plates is different (if the capacitance of each of the Figure 19.30
capacitors is different)
Q Q Q Q Q
V = V1 + V2 + V3 + .......... + Vn = + + + ..... + (from C = ) ;
C1 C2 C3 Cn V
V 1 1 1 1
∴ = + + + ..... +
Q C1 C2 C3 Cn

If C is the resultant capacitance of C1, C2……, Cn which are connected

V 1 1 1 1 1 1
In series then = ; ∴ = + + + ..... +
Q C C C1 C2 C3 Cn

PLANCESS CONCEPTS

The value of C is smaller than the smallest of the values of the capacitors which are connected in series.
B Rajiv Reddy (JEE 2012 AIR 111)

10.2 Parallel Combination of Capacitors


In this arrangement of the capacitors the charge accumulated on each of the capacitors is different while the
potential difference between them is the same and is equal to potential difference between the common points A
and B of such a connection.
Now,
= Q1 C=
1 V,Q 2 C=
2 V,Q3 C3 V
The total electric charge, Q = Q1 + Q2 + Q3 = ( C1 + C2 + C3 ) V
Q
In general, C = = C1 + C2 + C3 + .....,
V
Hence, by connecting capacitors in a parallel combination, a resultant capacitance can be obtained, whose value is
equal to the sum of all capacitances connected in parallel.
•• When a potential difference V is applied across several capacitors connected in parallel, it is applied across each
capacitor. The total charge q stored on the capacitors is the sum of the charges stored on all the capacitors.
•• Capacitors connected in parallel can be replaced with an equivalent capacitor that has the same total charge q
and the same potential difference V as the actual capacitors.
•• When a potential difference V is applied across serveral capacitors connected in series, the capacitors have
identical charge q. The sum of the potential differences across all the capacitors is equal to the applied potential
difference V.
1 9 . 1 8 | Electric Potential and Capacitance

Illustration 15: Three capacitors each of capacitance 9pF are connected in series.
(a) What is the total capacitance of the combination?
(b) What is the potential difference across each capacitor if the combination is connected to a 120V supply?
 (JEE MAIN)
−1
 1 1 1 
Sol: The equivalent capacitance in series combination is given by Ceq = 
C
+ +  .
 1 C2 C3 
q
The potential difference applied across each capacitor is given by V = where C is the capacitance of the capacitor.
C
1 1 1 1 1 1 1 1
(a) Here, = + + = + + = 3x
Ceq C1 C2 C3 C C C C
1 1 1 −12
i.e. = 3x = =
or Ceq 3x10
= F 3pF.
Ceq 9x10 −12 3x10 −12

q q q  q q
(b) Here V1 + V2 + V3 =
120 i.e. + += 120 ∴C
= or
= V 
C1 C2 C3  V C
3q 3xq
i.e. = 120 i.e. = 120 or q = 360x10−12 F
C 9x10 −12

q 360x10−12
∴ P.D. across a capacitor= = = 40V
C 9x10−12
Short cut
120
Since three equal capacitors are across 120V, so p.d. across each capacitor
= = 40V
3

10 F 20 F
Illustration 16: Calculate the charge on each capacitor shown in Fig. 19.31. (JEE MAIN)
C1C2
Sol: The equivalent capccitance is given by C =
C1 + C2
The two capacitors are joined in series. Their equivalent capacitance

1
Is given by =
1
+
1
or, =
C
C1C2
=
(10µF )( 20µF=) 20
µF.
30 V
Figure 19.31
C C1 C2 C1 + C2 30µF 3

The charge supplied by the battery is Q = CV


In series combination, each capacitor has equal charge and this charge equals the charge supplied by the battery.
 20 
 µF  ( 30V ) =µ
Thus, each capacitor has a charge of 200 µC . = 200 F
 3 

Illustration 17: Find the equivalent capacitance of the combination  P


shown in Fig. 19.32 between the points P and N. (JEE MAIN)
10 F 20 F
Sol: The circuit is the combination of series and parallel connection of
capacitors. The equivalent capacitance of series combination is given Q 30 F
CeqC2 N
by C = whereas the equivalent capacitance of the parallel
Ceq + C2 Figure 19.32

connection is given by Ceq


= C1 + C3 .
P hysi cs | 19.19

The 10µF and 20µF capacitors are connected in parallel. Their equivalent capacitance is 10µF +20µF = 30µF.
We can replace the 10µF and the 20µF capacitors by a single capacitor of capacitance 30µF between P and Q.

This is connected in series with the given 30µF capacitor. The equivalent capacitance C of this combination is given
1 1 1
by = + or,C = 15µF .
C 30µF 30µF

11. ENERGY STORED IN AN ELECTRIC FIELD OF A CAPACITOR


The work that needs to be done by an external force to charge a capacitor is visualized as electric potential energy
U stored in the electric field between the plates. This energy can be recovered by discharging the capacitor in a
circuit.
Suppose that, at a given instant, a charge q’ has been transferred from one plate of a capacitor to the other. The
potential difference V’ between the plates at that instant is q’/C. The work required to increase the charge by dq’ i
q
dW V'dq'
= = dq'
C
1 q q2
The work required to bring the total capacitor charge up to a final value q is
= w ∫= dW ∫ q'dq'
= .
C 0 2C

q2
This work is stored as potential energy U in the capacitor, so that we can also write this as U = (Potential energy).
2C
•• The potential energy of a charged capacitor may be viewed as being stored in the electrc field between its
plates.

Illustration 18: A parallel plate air capacitor is made using two plates 
+ - + -
0.2 m square, spaced 1 cm apart. It is connected to a 50V battery.
 (JEE ADVANCED)
+ - + -
(a) What is the capacitance? E E
(b) What is the charge on each plate? + - + -
d1 d2
(c) What is the energy stored in the capacitor?
(d) What is the electric field between the plates? Figure 19.33

(e) If the battery is disconnected and then the plates are pulled apart to a separation of 2cm, what are the
answers to the above parts?

Sol: The capacitance, charge on capacitor, energy stored in capacitor, the electric field between plates of capacitor,
εo A 1 2 V
are given=by C = ; Q CV;
= U CV and = E respectively.
d 2 d
ε0 A 8.85X10−12 X0.2X0.2
(a) =
C0 = = ; C0 3.54X10−5 µF
d0 0.01

(b) Q=
0 C0 V
=0 (3.54X10 −5 X50)µ=
C 1.77X10 −3 µC

1
(c) U0
= = C V 2 1 / 2(3.54X10−11 )(50)2 ; U0 = 4.42X10−8 J.
2 0 0
V0 50
(d) E=
0 = = 5000V / m.
d0 0.01
1 9 . 2 0 | Electric Potential and Capacitance

(e) If the battery is disconnected, the charge on the capacitor plates remain constant while the potential difference
between plates can change.
Aε0 Q Q0
C
= = : 1.77X10−5 µF ; =
Q Q= 1.77X10 −3 µC ; V
= = = 2V= 100Volts. ;
2d0 0
C C0 / 2 0

2
1 Q2 1 Q0 2V0
U
= = = 2U = 8.84X10−8 J , =
E = E= 5000V / m.
2 C 2 ( C0 / 2 ) 0
2d0 0

Work has to be against the attraction of plates when they are separated. This gets stored in the energy of the
capacitor.

Illustration 19: Find the energy stored in a capacitor of capacitance 100 µF when it is charged to a potential
difference of 20V.  (JEE MAIN)
1 2
Sol: The energy stored in the capacitor is given by U = CV .
2
1 1
The energy stored in the capacitor is U =CV 2 =(100µF )( 20V ) =
2
0.02J
2 2

C C
Illustration 20: Prove that in charging a capacitor half of the  + -
S S
energy supplied by the battery is dissipated in the form of q

heat.  (JEE ADVANCED)
V
Sol: When the swich S is closed, q=CV charge is stored in the
V +
capacitor. Charge transferred from the battery is also q. Hence, q
Figure 19.34
energy supplied by the battery = qV = (CV)(V) = CV 2 Half, of
1 1
its Energy, i.e., CV 2 is stored in the capacitor and the remaining 50% or CV 2 is dissipated as heat.
2 2

11.1 Energy Density


Neglecting fringing, the electric field in a parallel-plate capacitor has same value at all points between the plates.
The energy density u, which is the potential energy per unit volume between the plates should also be uniform. We
U CV 2
can find u by dividing the total potential energy by the volume Ad of the spacebetween
betweenthe
thePlat
plates
es =
u = ,.
2
Ad 2Ad
1 V
C = ε0A/d, this result becomes u= ε  
2 0d
1
V/d equals the electric field magnitude E; so u= ε E2 (Energy density )
2 0
Although we derived this result for the special case of a parallel-plate capacitor, it holds generally, whatever may
be the source of the electric field.
P hysi cs | 19.21

12. FORCE BETWEEN THE PLATES OF A CAPACITOR


q -q
Consider a parallel plate capacitor with plate area A. Suppose a positive charge q is given to  + -
one plate and a negative charge – q to the other plate. The electric field on the negative plate + -
due to positive charge is + -
σ q q
+ -
=E =  σ= + -
2ε0 2Aε0 A
+ -
q2 -
The magnitude of force on the charge in negative plate is =
F qE
= +
2Aε0 Figure 19.35
q2
This is the force with both the plates attract each other. Thus, F =
2Aε0

13. DIELECTRICS
If the medium between the plates of a capacitor is filled with an insulating substance (dielectric), the electric field
due to the charges plates induces a net dipole moment in 
the dielectric. This effect called polarization, gives rise to + -
+ - +- + -
a field in the opposite direction. Due to this, the potential +
+
-
-
+ -
+- + -
difference between the plates is reduced. Consequently, the +
+
-
-
+ -
+- + -
capacitance C increases from its value C0 when there is no + - +- + -
+ - + -
medium (vacuum). The dielectric constant K is defined by + - +- + -
+ - + -
Cdielectric + - +- + -
C + -
the relation
= K = + -
+ -
+- + -
C vacuum C0 + -
(a) (b)
Where C vacuum = capacity of a capacitor when there is Electric field lines with The induced charges on
vacuum between the plates. Thus, the dielectric constant of vacuum between the the faces of the dielectric
a substance is the factor by which the capacitance increases plates reduce the electric field

from its vacuum value, when dielectric is inserted fully Figure 19.36
between the plates.

13.1 Dielectrics at an Atomic View


What happens, in atomic and molecular terms, when we put a dielectric in an electric field? There are two possibilities,
depending on the type of molecule:-
(a) Polar Dielectrics: The molecules of some dielectrics, like water, have permanent electric dipole moments.
In such cases, the electric dipoles tend to line up with an external electric field because the molecules are
continuously jostling each other as a result of their random thermal motion. This alignment becomes more
complete as the magnitude of the applied field is increased (or as the temperature, and thus the jostling, are
decreased). The alignment of the electric dipoles produces an electric field that is directed opposite to the
applied field and is smaller in magnitude.

(b) Nonpolar moments: Even nonpolar molecules acquire dipole moments by induction when placed in an
external electric field. This occurs because the external field tends to “stretch” the molecules, slightly separating
the centres of negative and positive charge.
1 9 . 2 2 | Electric Potential and Capacitance

+
+

+
+
+

+
+ +

+
+

+
+

+
+

+
+

+
+

+
+

(a) (b)
Figure 19.37

Molecules with a permanent electric dipole An electric field is applied producing partial
moment, showing their random orientation in alignment of the dipoles. Thermal agitation
the absence of an external electric field. prevents complete alignment

+ - + -
+ -+ -+ -+ -+ - + + -
+ - + -
+ - +   -
-+ -+ -+ -+ E0 E +
 + - + -
E0=0 +  - + -
E0 
+ -+ -+ -+ -+ - + E0 + -
+ - + -
(a) (b) (c)

Figure 19.38

(i) A nonpolar dielectric slab. The circles represent the electrically neutral atoms within the slab.
(ii) An electric field is applied via charged capacitor plates; the field slightly stretches the atoms, separating the
centres of positive and negative charge.

(iii) The separation produces surface charges on the slab faces. These charges set up a field E ’, which opposes
   
the applied field E0 . The resultant field E inside the dielectric (the vector sum of E0 and E ’) has the same

direction as E0 but a smaller magnitude.

PLANCESS CONCEPTS

Real Capacitors
• Real capacitors used in circuits are parallel plate capacitors with a dielectric material inserted in
between the plates and rolled into a cylinder.
• This model reduces the size to get maximum capacitance.
Nitin Chandrol (JEE 2012 AIR 134)
P hysi cs | 19.23

Illustration 21: A parallel plate capacitor is to be designed with a voltage rating 1kV, using a material of dielectric
constant 3 and dielectric strength about 107 Vm-1. (Dielectric strength is the maximum electric field a material
can tolerate without breakdown, i.e. without starting to conduct electricity through partial ionization.) For safety,
we should like the field never to exceed, say 10% of the dielectric strength. What minimum area of the plates is
required to have a capacitance of 50pF?  (JEE ADVANCED)

C ⋅D
Sol: The area of the capacitor is given by A = where ε = ε0 ⋅ εr lim is the permittivity of the medium.
ε x →∞

10% of the given field i.e. 107 Vm-1. Given E=0.1x107 Cm-1.

−dV V V 1000
Using E = ; i.e. E = , weget r =
we get; = = 10−3 m
dr r E 0.1x107

Using C =
ε0 εr A
, we get;
= A =
Cd Cr
=
( 450x10 )
−12
(10 =)
−13

19cm2
d ε0 εr ε0 εr 8.854x10 −3
×3

Illustration 22: A parallel plate capacitor with air between the plates has a capacitance of 8pF (1pF = 10−12 F.) What
will be the capacitance if the distance between the plates is reduced by half, and the space between them is filled
with a substance of dielectric constant 6? (JEE MAIN)
kεA
Sol: The capacitance is C = where k is the dielectric constant. If d reduces to ½ and k changes to 6 then new
d
capacitance is 12 times the original capacitance.
ε0 εr A
Using C’ = C = , C' ∈r C = 12(8 × 10‒12) = 96 × 10‒12F = 96pF
d

Illustration 23: Two parallel-plate capacitors, each of capacitance 40 µF , are connected in series. The space
between the plates of one capacitor is filled with a dielectric material of dielectric constant K=4. Find the equivalent
capacitance of the system. (JEE ADVANCED)
C1C2
Sol: The equivalent capacitance of series connection is Ceq = .
C1 + C2
The capacitance of the capacitor with the dielectric is C1 = KC0 = 4 × 40 µF = 160 µF.
The other capacitor has capacitance C2 = 40 µF.

As they are connected in series, the equivalent capacitance is C=


C1C2
=
(160µF )( 40µF=) 32µF
C1 + C2 200µF

13.4 Dielectrics and Gauss law


 
From parallel plate capacitor with a dielectric equation we can write Gauss’ law in the form ε0 ∫ K E.dA = q (Gauss’
law with dielectric)
This equation, although derived for a parallel-plate capacitor, is true generally and is the most general form in
which Gauss’ law can be written.
Some more information about capacitors
(a) If charge is held constant, i.e. battery disconnected and dielectric is inserted between plates.

(i) Charge remains unchanged, i.e., q = q0, as in an isolated system, charge is conserved.
(ii)
Capacity increases, i.e. C = KC0, as by presence of a dielectric capacity becomes K times.
q q0
(iii)
P.D. between the plate decreases, i.e., V=(V0/ K) V= = as q c KC0  as,
= q0 and =
C KC
1 9 . 2 4 | Electric Potential and Capacitance

q q0
V= = as q
= q0 and =
c KC0 
C KC

V V E  V V0 
E between the plates decreases, i.e., E = (E0 / K ) , as, =
(iv) E = 0= 0 = 0 and E=
as V 0 
d Kd K  K d
Energy stored in the capacitor decreases, i.e. U = (U0 / K ) .
(v)
2
q2 q 0 U0
=U = = as=
q q0 and =
C KC0 
2C Kd K

q0 q
+++++ +++++ +++++ +++++

C0 , V0 , E0 , U0 C, V, E, U K

- - - - - - - - - - - - - - - - - - - -

(a) (b)
Figure 19.39

(b) If potential is held constant, i.e., battery remains attached and dielectric in inserted between plates.

(i) PD remains constant, i.e. V = V0, as battery is a source of constant potential difference.

(ii)
Capacity increases, i.e., C = KC0, as by presence of a dielectric capacity becomes K times.

(iii)
Charge on capacitor increases, i.e., q = Kq0 as=
q CV
= (KC0 )=
V Kq0 as q
=0 C0 V 

 V  V0  V0 
(iv)
Electric field remains unchanged, i.e., E = E0,=
E =  = E0 as =
V V0 and = E0 
d d  d 
(v)
Energy stored in the capacitor increases, i.e., U = KU0

1 2 1
((KC0 )( V0 ) ; 1=
2
As,
= U = CV  1 
2 2 = KU as C KC
= 0 and U0 C V 2
2 0 2 0 0 

Illustration 24: A parallel-plate capacitor has plate area A and  b


plate separation d. The space between the plates is filled up to
a thickness x(<d) with a dielectric of dielectric constant K. d c
Calculate the capacitance of the system.  (JEE ADVANCED) x

a
Figure 19.40

Sol: As the distance between the plates is filled with dielectric material upto a distance x<d, this system represent
CC
two capacitors connected in series. Thus equivalent capacitance is Ceq = 1 2 where each capacitance depends
C1 + C2
on the dielectric constant K and distance from the plate of capacitor.
The situation is shown in Fig. 19.40. The given system is equivalent to the series combination of two capacitors, one
between a and c and the other between c and b.
Here c represents the upper surface of the dielectric. This is because the potential at the upper surface of the
dielectric is constant and we can imagine a thin metal plate being placed there.
P hysi cs | 19.25

Kε0 A ε0 A
The capacitance of the capacitor between a and c is C1 = and that between c and b is C2 = .
x d−x
C1C2 Kε0 A
The equivalent capacitance
= is C =
C1 + C2 Kd − x (K − 1 )

Illustration 25: In the situation shown in Fig. 19.41 the area of the 
plates is A. The dielectric slab is released from rest. Prove that the
slab will execute periodic motion and find its time period. Mass of
the slab is m.  (JEE ADVANCED) V K d

Sol: As the slab is moving in constant electric field applied x


between the plates of capacitors, the force acting on it is constant.
F L
The acceleration is obtained by a = .When the slab moves
m Figure 19.41
completly out side the plates the electric force pulls the slab slides
2s
inside the plates of capacitor. The time period of oscillation is given by t = where s is the distance travelled by
a
dU ε0bV 2 (K − 1 )
the slab. After using, F = − , where U→ Potential
Constantenergy.
force,Constant force, F =
dx 2d
A
Here, b width
= = of plate
L
F ε 0 ( A / l ) V 2 (K − 1 )
∴ Acceleration of slab a = or a=
m 2md
The equilibrium position of the slab is, at the instant when the slab is fully inside the plates. So, the slab will execute
oscillations in the phases as shown in Fig. 19.42.
x x
(a) (b) (c)

x x
(a) (b) (c)

x
(d) (e)
Figure 19.42
T
= time taken to reach from position (a) to position (b) = t (say) ; using Sx = 1 at2
using
4 (d) 2(e)
1
using S = at2 2s
2
2 (l − x )
we ha ve t =
4 (l − x ) mdl a
; (S = l − x )
2s
Wehahave
we ve t = ; = (S = l − x ) = 2 (l − x ) 4 (l − x ) mdl
a
ε (A/l)V2 (K − 1)
ε0 AV 2 (K − 1 )
= =
2 (l − x ) 4 (l − x ) mdl ε0 AV 2 (K − 1 )
0
ε
2md 0( A/l) V2 (K −1 )
= =
ε
0( A/l) V2 (K −1 )
ε0 AV (K − 1 )
2
2md

TheThe desired
desired time
time
2md
period
period is, is, =
T 4t
= 8
(L − x ) mdl
ε0 AV 2 (K − 1 )
1 9 . 2 6 | Electric Potential and Capacitance

Illustration 26: Three concentric conducting shells A, B and C of radii a, b and c are  C
B
a shown in Fig. 19.43. A dielectric of dielectric constant K is filled between A and B,
find the capacitance between A and C. (JEE ADVANCED) A

Sol: The potential difference in the region dr between the shells is given by a
∆V = ∫ E ⋅ dr where E is the electric field. Capacitance is the ratio of charge and
b
potential difference. Let the sphere A have a charge q. When the dielectric is filled
between A and B, the electric field will change in this region. Therefore the potential c
difference and hence the capacitance of the system will change. So, first find the
electric field E(r) in the region a ≤ r ≤ c . Then find the potential difference (V) Figure 19.43
q q
between A and C and finally the capacitance of the system is C == , E(r) for a≤r ≤b
  V 4 πε r 2
q q 0
) forfor =
a≤r ≤b for b ≤ r ≤ c . Using, dv = − ∫ E.dr
2
4 πε0r 4 πε0r 2
b q b q
Here potential difference between A and C is, V = VA − VC = ∫ .dr − ∫ dr
2
a
4 πε0Kr a
4 πε0r 2
q  1  1 1   1 1  q  (b − a ) ( c − b )  q
=   − + = −   =
+  c (b − a) + Ka ( c − b ) 
4 πε0  K  a b   b c   4 πε0  Kab bc  4 πε0kabc  

q 4 πε0Kabc
The desired capacitance is C
= =
V Ka ( c − b ) + c (b − a)

14. R-C CIRCUITS


To understand the charging of a capacitor in C-R circuit, let us first consider the charging of a capacitor without
resistance. q0=CV
C
+ -

S V V

Figure 19.44

Consider a capacitor connected to a battery of emf V through a switch S. When we close the  q
switch, the capacitor gets charged immediately. Charging takes no time. A charge q0 = CV
q0
comes in the capacitor as soon as switch is closed and the q-t graph in this case is a straight
line parallel to t-axis as shown.

Figure 19.45

But if there is some resistance in the circuit charging takes some time. Because resistance  C
opposes the charging (or current flow in the circuit).
Final charge (called steady state charge) is still q0 but it is acquired after a long period of
time.

( )
S V
− t/ τc
The q-t equation in this case is,=
q q0 l − e
Figure 19.46
P hysi cs | 19.27

Here, q0 = CV and τc = CR = time constant. q


q0
q-t graph is an exponentially increasing graph. The charge q increases 
0.632 q0
exponentially from 0 to q0.
From the graph and equation we see that at t = 0, q = 0 and at t = ∞, q = q0
t=C t
Figure 19.47

14.1 Charging
When a capacitor C is connected to a battery through a resistance R, the plates of a capacitor will acquire equal and
opposite charge and the potential difference across it becomes equal to the emf of the battery. The process (called
charging) takes sometime and during this time there is an electric current through the resistance. If at any time t,
I is the current through the resistance R and q is the charge on capacitor C, the equation of emf for the circuit will
be VC= + VR E, i.e., V=+ IR E

But I = (dq/ dt) and q = CV

dq q q dq t dt
so, R + =E or ∫0 ( CE − q) = − ∫0 CR
dt C

q q0 l − e− t/CR
Which on solving for q gives= ( ) with q 0 = CE ( for t = ∞ ) …..(i)

This is the required result and from this it is clear that:


(a) During charging, charge on the capacitor increases from 0 to q0 (=CE) non-linearly.
(b) The density CR is called capacitive time constant τ of the circuit [as it has dimensions of time] and physically
represents the time in which charge on the capacitor reaches 0.632 times of its maximum value during
charging.
(c) During charging current at any time t in the circuit will be


dq d 
I == 
dt dt 
(
q0 l − e− t/CR  =
 ) E
I0 e− t/CR with I0 =
R
( at t =
0)

i.e., initially it acts as short circuit or as a simple conducting wire. If t → ∞ , I → 0 , i.e., it acts as open circuit or
as a broken wire.

14.2 Discharging
If a charged capacitor C having charge q0 is discharged through a resistance R, then at any time t,
V = IR ( −dq / dt ) and q =
but as I = CV

dq q q dq t dt
R + =
dt C
0 i,e., ∫q0 dt = −∫
0 CR
or q = q0 e− t/CR

This is the required result and from this it is clear that


(a) During discharging, the charge on capacitor decreases exponentially from q0 to 0
(b) The capacitive time constant τ =CR is the time in which charge becomes (l/e), i.e., 0.368 times of its initial
value (q0)
(c) During discharging current at any time t in the circuit:


I I=
dq
dq
−− =
=
dt
dt
=
dd
dt
dt
((
−− qq00ee−−t/CR
t/CR
=
= ))
I0I0ee−−t/CR
t/CR
with
EE
= and its direction is opposite to that of charging.
withI0I0 =
RR
1 9 . 2 8 | Electric Potential and Capacitance

(d) As in discharging of a capacitor through a resistance q = q0 e− t/CR i.e., R = t


Cloge (q0 / q)
So resistance R can be determined from the value of t and ( q / q0 ) , i.e. ( V0 / V ) . Using this concept in
laboratory we determine the value of high resistances (  MΩ ) by the so called ‘Leakage method’.

Definition of τc At t = τc , q = q0 (1-e-1) = 0.632 q0


Hence, τc can be defined as the time in which 63.2% charging is over in a C-R circuit. Note that τc is the time.
time  or CR  = M0L0 T  .
Hence,  τc  =
 

15. VAN DE GRAAFF GENERATOR


+ + + ++ ++
Van de Graaff generator is a device that can generate a potential  Pulley

+
Metal brush

++

++
difference of a few million volts. The highly intense electric field

++ +++

+ ++++
produced in this matching is used to accelerate charged particles, Insulating belt
which can then be used to study the composition of matter at the
+
+ to carry and
+
microscopic level. + +

+
+ deliver charge

+
+
+ Insulating
Principle: let us presume (See Fig. 19.48) that charge Q is +
+ supporting
+
residing on an isolated conducting shell, having radius R. + column
+
+ Motor driven
+
There is another conducting sphere having charge q and radius Metal brush
+++ pulley
equal to r at the centre of the above mentioned spherical shell (r<R)
delivering Grounded
charge from metal base
The electric potential on the spherical shell of radius R is equal to source Principle of construction of
Van de Graaff generation
kQ kq
V
=R +
R R Figure 19.48
kQ kq
The electric potential on the surface of the sphere of radius r is, =
Vr +
R r
kQ kq kQ kq 1 1 
Therefore, the potential difference between the surfaces of the two sphere is, Vr − VR =+ − − = kq  − 
R r R R r R
The above equation shows that the smaller sphere is at a higher electric potential in comparison to the larger
spherical shell. If the smaller sphere is brought in contact with the larger spherical shell then electric charge will
flow from the smaller sphere towards the larger spherical shell. Note if charge can be continuously transferred to
the smaller sphere in some way, it will keep getting accumulated on the larger shell, thereby increasing its electric
potential to a very high value.
Construction: S is a large spherical conducting shell with a radius of a few meters. This is erected to suitable
height over the insulating pillars, C1 and C2. A long narrow belt of insulating material moves continuously between
2 pulleys P1 and P2 as shown in the Fig. 19.48. B1 and B2 are two sharply pointed brushes fixed near pulleys, P1 and
P2 respectively, such that B2 touches the belt near the pulley P2. B1 is called the spray brush and B2 is called the
collector brush.
Working: The spray brush is given a positive potential w.r.t. the earth by high tension source known as E.H.T.
around the sharp points of B1, the high potential causes the air to ionize and this sprays positive charges on the
belt. As the belt moves, and reaches the sphere S, the collecting brush B2, which touches the belt near pulley P2,
collects the positive charge, which spreads on the outer surface of S. the uncharged belt returns down and again
collects the positive charge from B1. As the belt moves continuously between P1 and P2 the positive charge starts
accumulating on sphere S and the charge due to ionization is minimized by enclosing the metallic shell in an earth
connected steel tank filled with air at high pressure. The charge particles may accelerate in this large potential to
high kinetic energies of the order of more than 2 MeV.
P hysi cs | 19.29

PROBLEM-SOLVING TACTICS

Below we illustrate how the above methodologies can be employed to compute the electric potential for a line of
charge, a ring of charge and a uniformly charged disk.

Charged Rod Charged Ring Charged disk


Figure y z z
P P P
r dq 
’ z
Z r
y y
r’  R y
R r’
dr’
x’ x dp
x
O dx’
L x

Figure 19.49 Figure 19.50 Figure 19.51

(2) Express dq in dq = λdl dq = σdA


dq = λdx'
terms of charge
density
(3) Substitute dq into
λdx' dV = k e
λdl
dV = k e
σdA
expression for dV dV = k e
r r r

(4) Rewrite r and dl


= Rdφ
dx' dA = 2πr'dr'
the differential
element in terms =r x' 2 + y 2 =r R 2 + z2 =r r' 2 + z 2
of the appropriate
coordinates
(5) Rewrite dV
λdx' λRdφ' 2πσr'dr'
dV = k e dV = k e dV = k e
(x' 2 + y 2 )1/2 (R 2 + z 2 )1/2 (r' 2 + z 2 )1/2

(6) Integrate to get V


λ dx1
 /2 Rλ ' R r'dr'
V=
4 πε0 ∫− /2= V ke
(R + z 2 )1/2
2 ∫ dφ V k e 2πσ ∫
=
'2
x +y 2
(r' 2 + z 2 )1/2 0

(2πRλ )
=
λ
ln = ke = 2k e πσ  z 2 + R 2 − z 
4 πε0 R 2 + z2  
 ( / 2) + ( / 2)2 + y 2  Q 2k eQ  2 2 
= ke =  z +R − z 
  R2  
 −( / 2) + ( / 2)2 + y 2  R + z2
2
 

Derive E from V ∂V k Q ∂V
∂V
Ey = − Ez =
− = e Z Ez = −
∂y ∂z
∂z
( )
3/2
R + z2
2
λ /2 2k eQ  z z 
= =  − 
2πε0 y R 2  z 
(  / 2)
2
+ y2 z + R2
2

Point- charge limit k eQ k eQ k eQ


for E Ey ≈ y >>  Ez ≈ z >> R Ez ≈ z >> R
2
y z 2
z2
1 9 . 3 0 | Electric Potential and Capacitance

For any given combination, one may proceed as follows:


Step 1: Identify the two points between which the equivalent capacitance is to be calculated. Call any one of them
as P and the other as N.
Step 2: Connect (mentally) a battery between P and N with the positive terminal connected to P and the negative
terminal to N. Send a charge +Q from the positive terminal of the battery.
Step 3: Write the charges appearing on each of the plates of the capacitors. The charge conservation principle
may be used. The facing surfaces of a capacitor will always have equal and opposite charges. Assume variables
Q1, Q2 …, etc., for charges wherever needed.
Step 4: Take the potential of the negative terminal N to be zero and that of the positive terminal P to be V. Write
the potential of each of the plates. If necessary, assume variables V1, V2….
Step 5: Write the capacitor equation Q = CV for each capacitor. Eliminate Q1, Q2 …and V1, V2…., etc., to obtain the
equivalent capacitance C=Q/V.

FORMULAE SHEET

S. No FORMULA

1. q = CV
 
2. ε0 ∫ E.dA =
q.

f 
3. ∫ E.dS.
V1 − V2 =
i

+ d
4. =V ∫=
− ∫0 ds
Eds E= Ed

5. q = ε0EA.

ε0 A
6. C= (parallel-plate capacitor)
d

L
7. C= 2πε0 (cylindrical capacitor)
ln (b / a)

8.
ab
C= 4 πε0 (spherical capacitor)
b−a

9. C= 4 πε0R (isolated sphere)

n
10. Ceq = ∑ C J (n capacitors in parallel)
j=1

n
1 1
11. =∑ (n capacitors in series)
Ceq j=1 C J
P hysi cs | 19.31

S. No FORMULA

12. 1 2 (potential energy)


U= CV
2

13. q2 (potential energy)


U=
2C

1
14. u= ε E2 (energy density)
2 0

q
15. E=
4 πKε0r 2

 
16. ε0 ∫ KE.dA =
q (Gauss’ law with dielectric).

ε0bV 2 (K − 1 )
17. Force on a Dielectric Slab inside a Capacitor F =
2d

Electric Potential Formulae

S. No Term Description
1 Electric Potential ∆U = -W Where ∆U = Change in potential energy and W= Work done by the electric lines of
energy force.

For a system of two particles U(r) = q1q2 / 4πεr

Where r is the separation between the charges.

We assume U to be zero at infinity.

Similarly for a system of n charges

U = Sum of potential energy of all the distinct pairs in the system

For example for three charges

U = (1 / 4 πε )(q1q2 / r12 + q2q3 / r23 + q1q3 / r13 )

2 Electric PE of a = qV where V is the potential.


charge
3 Electric Potential Like Electric field intensity is used to define the electric field; we can also use Electric Potential
to define the field. Potential at any point P is equal to the work done per unit test charge by
the external agent in moving the test charge from the reference point (without Change in KE)

Vp = Wext / q. So for a point charge V0 = Q / 4πεr

Where r is the distance of the point from charge.


1 9 . 3 2 | Electric Potential and Capacitance

S. No Term Description
4. Some points 1. It is scalar quantity
about Electric
2. Potential at a point due to system of charges will be obtained by the summation of potential
potential
of each charge at that point

V = V 1 + V2 + V3 + V4

3. Electric forces are conservative forcew so work done by the electric force between two points
is independent of the path taken

4. V2 − V1 =
− E.dr ∫
5. In Cartesian coordinates system

dV =
−E.dr; dV =
−(Ex dx + Ey dy + Ez dz)
So Ex =
∂V / ∂x, Ey =
∂V / ∂y and Ez =
∂V / ∂z,

( ) ( ) (
Also E = ∂V / ∂x i + ∂V / ∂y i + ∂V / ∂z k  ) 
6. Surface where electric potential is same everywhere is called equipotential surface.

Electric field components parallel to equipotential surface are always zero.


5 Electric dipole A combination of two charges +q and –q separated by a distance d has a dipole moment
p = qd, where d is the vector joining negative to positive charge.
6 Electric potential
=V (1 / 4 πε) × (pcos θ / r 2 )
due to dipole
Where r is the distance from the center and θ is angle made by the line from the axis of dipole.
7 Electric field due
Eθ (1 / 4 πε )(psin θ / r3 );=
= Er (1 / 4 πε ) × (2pcos θ / r3 )
to dipole
Total
= E Eθ2 +=
Er 2 (p/ 4 πε r3 )( (3cos2 θ + 1)
Torque on dipole = p × E

Potential Energy U = ‒p.E


8 Few more points
1. ∫ E.dl over closed path is zero

2. Electric potential in the spherical charge conductor is Q / 4πεR where R is the radius of the
shell and the potential is same everywhere in the conductor.

3. Conductor surface is a equipotential surface

Electric potential due to various charge distributions

Name/Type Formula Note Graph


Point Charge •• q is source charge
Kq
r •• r is the distance of the point from the V
point charge.
r

Ring (uniform/non •• Q is source charge V


KQ
uniform charge At centre,
R •• x is distance of the point from centre.
distribution)
KQ
At axis,
R 2 + x2
P hysi cs | 19.33

Name/Type Formula Note Graph


Uniformly charged •• R is radius of sphere V
KQ
hollow conducting/ For r ≥ R, V =
R •• r is distance of the point from centre
non - conducting KQ/R
KQ of the sphere
sphere or solid For r ≤ R, V = 2
conducting sphere R •• Q is total charge ( ( =σ 4 πR )
r
R

Uniformly charged KQ •• R is radius of sphere V


solid non - conducting For r ≥ R, V =
r •• r is distance of point from centre of 3KQ/2R
sphere (insulating
For r ≤ R, the sphere. KQ/R
material)
4 3
•• Q is total charge ( =ρ πR )
3 3
KQ(3R 2 − r 2 ) ••   Vcentre = V
r
V= R
2R3 2 surface
ρ •• Inside sphere potential varies
= (3R 2 − r 2 ) parabolically.
6ε 0
•• Outside potential varies
hyperbolically.

Line charge Not defined •• Absolute potential is not defined

•• Potential difference between


two points is given by formula
VB − VA =−2Kλ ln(rB / rA ) , where
lambda is the charge per unit length

Infinite nonconducting Not defined •• Absolute potential Is not defined


thin sheet
•• Potential difference between
two points is given by formula
σ
VB − VA =
− (r − r ) , where
2ε0 B A
sigma is the charge density

Infinite charged Not defined •• Absolute potential is not defined


conducting thin sheet
•• Potential difference between
two point is given by formula
σ
VB − VA =
− (rB − rA ) , where
ε0
sigma is the charge density

 d
Electric dipole moment: p = qdzˆ , where two charges of charge ±q are placed along the z axis at z = ±
2
 1 p
Electric dipole field: Along the z axis (z>>d): E = zˆ , in the +z direction.
2πε0 z 3

 1 p
Along the x axis(x>>d): E = xˆ in the +x direction.
2πε0 x 3

 1 p
Along the x axis(y>>d): E = yˆ in the +y direction.
2πε0 y 3
1 9 . 3 4 | Electric Potential and Capacitance

Solved Examples

JEE Main/Boards Potential difference across C4 and 200 PF is in the ratio


2:1 i.e. 200 V across C4
Example 1: Obtain the equivalent capacitance of the ∴ Charge on C4 = C4 V4; =100 × 200 × 1012 = 2 × 10‒8 C
network in figure. For a 300V supply, determine the
Potential difference across C1=100V
charge and voltage across each capacitor.
100pF Charge on C1= C1 × V1 = 100 × 100 × 10‒12 =1 × 10‒8 C
Potential difference across C2 and C3 is 50V each
C1 ∴ Charge on C2 or C3= C2V2 = 200 × 50 × 1012 = 10‒8 C.

200pF 200pF
Example 2: The connection shown in figure are
+ established with the switch S open. How much charge
C2 100pF C3 300V will flow through the switch if it is closed?
-

C4 1 2

Sol: The circuit is made up of series and parallel + 24V 1F S 2F
combinations of the capacitors. The charge on the -
capacitor is given by q = CV. 2F 1F

100pF
4 3

100pF C1 (a)

C4 C2 C3
-Q1 Q1 Q2 -Q2
200pF 200pF 1 2
1F 2F
+ 24V S
300 V
- 0
2F V0 1F
4 3


Q2 Q2
The equivalent circuit is as shown below: (b)
100pF
Sol: Find the initial charge on capacitors. After the
100pF switch is closed, 1 and 2 become parallel and 3 and 4
become parallel.
C4
When the switch is open, capacitors (2) and (3) are in
100pF 2
µF. The charge appering on
series. Their equivalent capacitance is
3
300 V The charge appering on each of these capacitors is,
2


therefore, 24V × µF = 16µC


3
The equivalent capacitance of (1) and (4), which are
100pF 200pF 2
also connected in series, is also µF and the charge
3
300 V on each on each of these capacitors is also 16 µC . The
total charge on the two plates of (1) and (4) connected
to the switch is, therefore, zero.
P hysi cs | 19.35

The situation when the switch S is closed is shown in The equivalent capacitance between A being equivalent
figure. Let the charges be distributed as shown in the to the original ladder, the equivalent capacitance is
figure. Q1 and Q2 are arbitrarily chosen for the positive also C
plate of (1) and (2). CC1
Thus, C1= C + or C1C + C12 = C2 + 2CC1
Take the potential at the negative terminal to the zero C + C1
and at the switch to be V0
Or C12 − CC1 − C2 =
0,
Writing equations for the capacitors (i), (ii), (iii) and (iv).
Q1 = (24V ‒ V0) × 1µF … (i) C + C2 + 4C2 1 + 5
Giving C1
= = C
Q2 = (24V ‒ V0) × 2µF  … (ii) 2 2
Q1 = V0 × 1µF  ... (iii) Negative value of C1 is rejected.

Q2 = V0 × 2µF  … (iv)
Example 4: A parallel-plate capacitor has plates of area
From (i) and (iii), V0 =12V, Thus, from (iii) and (iv), 200 cm2 and separation between the plates 1.00 mm.
What potential difference will be developed if a charge
Q1= 12µC and Q= 24µC .
2 of 1.00 nC (i.e., 1.00 × 10‒9C) is given to the capacitor? If
The charge on the two plates of (1) and (4) which are the plate separation is now increased to 2.00 mm, what
connected to the switch is, therefore Q2 − Q1 =12µC . will be the new potential difference?
When the switch was open, this charge was zero. Thus, ε A
12µC of charge has passed through the switch after it Sol: Capacitance is given by C = 0 and the potential
d
was closed. Q
difference is given by V =
C
Example 3: Find the capacitance of the infinite ladder The capacitance of the capacitor is
shown in figure.
ε0 A 200x10−4 m2
C P C C C= = 8.85x10−12 Fm−1 x
A d 1x10−3 m
−9
= 0.177x10
= F 0.177nF.
C C C C The potential difference between the plates is
B
Q Q 1nC
V
= = = 5.65 Volts.
C 0.177nF
Sol: Capacitance between points P and Q is same as If the separation is increased from 1.00 mm to 2.00 mm,
that between A and B. The equivalent capacitance of the capacitance is decreased by a factor of 2. Thus, the
the circuit is deduced by reducing it to simple network. new potential difference will be 5.65 volts × 2 = 11.3 volts.
The simplified circuit is a parallel combination of the
capacitors whose capacitance is given by
Example 5: An isolated sphere has a capacitance of 50pF.
CC1
Ceq= C + . (a) Calculate its radius.
C + C1
(b) How much charge should be placed on it to raise its
As the ladder is infinitely long, the capacitance of the potential to 104 V?
ladder to the right of the points P, Q is the same as
that of the ladder to the right of the points A, B. If the Sol: For sphere, radius R is given by C= 4 πε0R . To
equivalent capacitance of the ladder is C, the given raise the potential to 104 V, the charge to be placed on
ladder may be replaced by the connections shown in sphere is given by Q = CV.
figure.
C P (a) The capacitance of an isolated sphere is
A
R
C= 4 πε0R. Thus, 50x10−12 F =
9x109 mF−1
−12
C C1 =or R 50x10
= x9x109 m 45cm.
B
Q (b) Q = 50X10−12 FX10 4 =
= CV V 0.5µC
1 9 . 3 6 | Electric Potential and Capacitance

Example 6: A parallel-plate capacitor of capacitance of charge induced in other plates. The capacitance is
100 µF if connected to a power supply of 200V. A Aε
dielectric slab of dielectric constant 5 is now inserted between any two surfaces is given by C = 0 .
d
into the gap between the plates. Suppose the negative terminal of the battery gives a
(a) Find the extra charge flown through the power charge – Q to the plate B. As the situation is symmetric
supply and the work done by the supply. on the two sides of B, the two faces of the plate B
will share equal charge – Q/2 each. From Gauss’s law,
(b) Find the charge in the electrostatic energy of the
the facing surfaces will have charge Q/2 each. As the
electric field in the capacitor.
positive terminal of the battery has supplied just this
much charge (+Q) to A and C, the outer surfaces of A
Sol: After inserting the dielectric the capacitance is
and C will have no charge. The distribution will be as
changed, thus the charge stored in the capacitor is
shown in figure. The capacitance between the plates A
given by Q = CV and work done by supply is CV2 = qV.
and B is
1 2
The energy stored in the capacitor is given by U = CV Aε0 200 × 10−4 m−2
2 C ==× 8.85 10−12 F / m×
d 2 × 10−4 m
(a) The original capacitance was 100µF. The charge on
= 8.85 = X10−10 F 0.885nF
the capacitor before the insertion of the dielectric was,
therefore, Q1 = 1000F × 200V = 20mC Thus, Q 8.85nF
= = x 20V 17.7nC
After the dielectric slab is introduced, the capacitance is The distribution of charge on various surfaces may be
increased to 500µF. The new charge on the capacitor is, written from figure.
100mC .
Therefore, 500µ F x 200v = Q
The equivalent capacitance is = 1.77nF
The charge flown through the power supply is, 20V
80 mC .
therefore, 100 mC − 20 mC =
Example 8: The emf of the cell in the circuit is 12 volts and
The work done by the power supply is 200V × 80mC = 16J. the capacitors are: C1 = 1µF, C2 = 3µF, C3 = 2µF, C4 = 4µF
(b) the electrostatic field energy of the capacitor without Calculate the charge on each capacitor and the total
the dielectric slab is charge drawn from the cell when
1 2 1
U1 =
2
CV =
2
(100µF ) X ( 200V ) =
2J (a) The switch s is closed
(b) The switch s is open.
And that after the slab is inserted is
1 C1 C2
(500µF ) X ( 200V ) =
2
U2 = 10J thus, the energy is
2
increased by 8J.

C3 C4
Example 7: Each of the three plates shown in figure has
an area of 200cm2 on one side and the gap between
the adjacent plates is 0.2mm. The emf of the battery is Cell
+ -
20V. Find the distribution of charge on various surfaces
of the plates. What is the equivalent capacitance of the 12 V
system between the terminal points?
Sol: When the switch is closed the equivalent
+
-
+
- 20V
capacitance is given by C=
( C1 + C3 )( C2 + C4 )
( C1 + C3 ) + ( C2 + C4 )
and the charge stored in the circuit is given by Q=CV.
Q/2

Q/2
-Q/2

-Q/2

When the switch is open the capacitance is given


A B C A B C
C1C2 C3C 4
by C
= + . The charge stored in the
(a) (b) C1 + C2 C3 + C 4
Sol: As the charge distribute symmetrically around capacitor is given by
the central plate. Due to this there is equal amount
(a) Switch S is closed:
P hysi cs | 19.37

C
=
( C1 + C3 )( C2 + C4 ) ; C
=
3×7
= 2.1µF ;
C1= charge on C2

( C1 + C3 ) + ( C2 + C4 ) 3+7  CC
= 3 4
 8
C +C  V = × 12 =16µC
 3  6
Total charge drawn from the cell is: 4

Q = CV = 2.1µF × 12 volts = 25.2µC C3 & C4 are in series and the potential difference across
combination is 12 volts charge on
C1 , C3 are in parallel and C2, C4 are in parallel.
C3= charge on C4
C1 C2
Q  CC  8
= 3 4  V = × 12 =16µC
C +C 6
 3 4 
C3 C4
Example 9: Two capacitors A and B with capacities 3 µF
and 2 µF are changed to a potential difference of 100V
+ -12V and 180V respectively. The plates of the capacitors are
connected as shown in the figure with one wire of each
Charge on C1 capacitor free. The upper plate of A is positive and that
of B is negative. An uncharged 2 µF capacitor C with
C1 1
Q1 = Q= × 25.2µC = 8.4µC lead wires falls on the free ends to complete the circuit.
C1 + C3 1+2 Calculate:
Charge on C3 q2
B + - C
C3 2 3 4
Q3= Q= × 25.2µC= 16.8µC
C1 + C3 1+2
+ 2 5 -
3F q1 2F
Charge on C2 -
1 6 q +
3

C2 3
Q2= Q= × 25.2µC= 10.8µC
C2 + C 4 3+ 4 A D

Charge on C4 (i) The final charge on the three capacitors, and

C4 4 (ii) The amount of electrostatic energy stored in the


Q 4= Q= × 25.2µC= 14.4µC system before and after the completion of the circuit.
C2 + C 4 3+ 4
(c) switch S is open : Sol: The charge stored in each capacitor when they
C1C2 C3C 4 1X3 2X4 25 are not connected to each other is given by q = CV.
=C + C= + = µF When the capacitors are connected to each other then
C1 + C2 C3 + C 4 1 + 3 2 + 4 12
the charge stored in each the capacitor is obtained by
Total charge drawn from battery is: applying Kirchoff’s 2nd law to the circuit. The energy
25 1
Q =CV = × 12 =25µC stored in the capacitor is given by U = CV 2
12 2

C1 C2 (i) charge on capacitor A, before joining with an


uncharged capacitor,
qA= CV= (100) + 3µc= 300µc
C3 C4 Similarly charge on capacitor B,
qB= 180 × 2µc= 360µc
12V
+ - Let q1, q2 and q3 be the charges on the three capacitors
after joining them as shown.
C1 and C2 are in series and the potential difference From conservation of charge,
across combination is 12 volts charge on
1 9 . 3 8 | Electric Potential and Capacitance

Net charge on plates 2 and 3 before joining (c) The total electrical energy stored after joining
= Net charge after joining 200V
+ -
∴300 =q1 + q2  …… (i)
6F
Similarly, net charge on plates 4 and 5 before
S
Joining = Net charge after joining
100V
−360 =−q2 − q3 ; 360
= q2 + q3  ...…(ii) + -

Applying Kirchoff’s 2nd law in loop ABCDA, 4F


q1 q2 q3 How do you account for the difference in energies in
− + 0
= (b) and (c) ?
3 2 2
0
2q1 − 3q2 + 3q3 = ..… (iii) Sol: When the capacitors are connected to each other
From equations (i), (ii) and (iii), then the total charge stored in the circuit is given by
q1 =90µC,q2 =90µC and q3 =150µC Q = Q1 + Q2 = C1 V1 + C2 V2 . The energy stored in each
1 2
(iii) (a) Electrostatic energy stored before completing capacitor is given by U = CV .
2
the circuit,
V1 = 100V,C1 = 4µF , Q1 = 4 × 10 −6 × 100
C1 V1 =
 1 2
 U = CV  = 4.74x10 J = 47.4mJ.
-2
2 = 4 × 10−4 C ; V2 = 200V,C2 = 6µF
 
2F Q2 =C2 V2 =6X10 −6 × 200 =12 × 10 −4 C
C When the plates with like charges are connected
together, both capacitors have the same potential after
+ - redistribution of charge.
3F 2F
A (a) Total charge = =
Q Q1 + Q2
B
=( 4 + 12 ) × 10−41.6 × 10−3 C
Capacitance of the combination in parallel
(b) Electrostatic energy stored after completing the = C = C1 + C2 = ( 4 + 6 ) µF = 10µF
circuit,
Potential across each capacitor after joining
1
( ) 1
2
Uf = 90X10−6 q 1.6 × 10−3
2 3x10−6 = V= = = 160V
C 10 × 10−6
1
( ) 1
2
+ 90x10−6 (b) Electrical energies U1 and U2 before joining are
2 2x10−6
given as :
1
( ) 1  1 q
2
150x10−6
+
2 2x10 −6
;  U =


2 c2 
1
U1 =
2
1
C1 V12 =
2
(
× 4X10 −6 × (100 ) =
2
)
2X10−2 J

= 90x10−4 J = 90mJ 1
U2 =
2
1
C2 V22 =
2
(
× 6X10−6 × ( 200 ) =
2
)
12X10−2 J

Example 10: A capacitor of capacitance 4 µ F is Energy before joining


charged to a potential difference of 100V and another = U=
i U1 + U2 ; = (12 + 2) × 10−2 = 0.14J
of capacitance 6 µ F is charged to a potential of 200V.
These capacitances are now joined with plates of like (c) Electrical energy after joining of capacitors
charges connected together. Calculate:
1
=Uf = × 10 × 0 −6 × (160)2 =0.128J
(a) The total potential across each after joining 2
(b) The total electrical energy stored before joining The stored electrical energy after joining is less by
(0.14-0.128) J i.e. 0.012J. the energy is dissipated as heat
energy through the connecting wires as current flows.
P hysi cs | 19.39

Example 11: What charges will flow thorugh A, B and = q1 (C)(P.D)


= (2)(30)µC
C in the directions shown in Fig. 19.69 when switch S is
Similarly, P.D. across 3 µF capacitor is same as that
closed?
between 60V battery. Hence, q =2 = 180µC. Now
(3)(60)
A let qA charge goes to the upper plate of 2 µF capacitor.
30V 2F Initially it had a charge +q and finally charge on it is
B + q1 . Hence, q1= q + qA or qA = q1 − q = 60 − 180
-S
=−48µC
3F
60V 48F
C -
48C
Sol: The charges stored in capacitor is given by Q = CV. - 30V
- 2F
CC
The equivalent capacitance is Ceq = 1 2 . To find the 48C +
C1 + C2
charges flowing in each capacitor we use the Kirchoff’s 72C
- 60V
2nd law. - 3F
Let us draw two figures as shown in figure and find the 72C C
charge on both the capacitors before closing the switch
and after closing the switch. Refer figure (a) when
Similary, charge qB goes to the upper plate of 3 µF
switch is open: both capacitors are in series. Hence,
capacitor and lower plate of 2 µF capacitor. Initially
their equivalent capacitance is,
both the plates had a charge +q, -q or zero. And finally
C1C2 (2)(3) 6 they have a charge (q2 − q1 ) .
Ceq= = = µF
C1 + C2 2+3 5
Hence, q2 − q1 = qB + 0
Therefore, charge on both capacitors will be same.
∴qB = q2 − q1
Hence, using q = CV, we get
= 180 − 60

30V
+ A = 120µC Initially it had a charge-q and finally −q2 .
q 2F 30V 2F
- Hence, qc =q − q2 =108 − 180 =−72µC
B
+ B
q 3F 3F Example 12: If 100 volts of potential difference is
60V - 60V
C applied between a and b in the circuit of figure, find the
potential difference between c & d.

(30 + 60 )  56  µC=
 
q= 108µC Sol: The charge stored in each capacitor is given by
  Q = CV. To find the potential difference at point’s c and
Refer figure (b), when switch is closed: let q1 and q2 be d we apply Kirchoff’s law to the circuit.
the charges (in µC ) on two capacitors. Then applying The charge distribution on different plates is shown in
Kirchoff’s 2nd law in upper and lower loop, we have figure. Suppose charge Q1 + Q2 is given by the positive
q1 terminal of the battery, out of which Q1 resides on the
30 − 0 Or q1 = 60µC;
=
2 positive plate of capacitor (1) and Q2 on that of (2). The
q2 remaining plates will have charges as shown in the
60 − 0 Or q2 = 180µC
= figure. Take the potential at the point b to be zero. The
3
potential at a will be 100V. Let the potentials at points
Charges q1 and q2 can be calculated alternatively by c and d be Vc and Vd respectively. Writing the equation
seeing that upper plate of 2 µF capacitor is connected Q=CV for the four capacitors, we get,
with positive terminal of 30V battery. Therefore, they
are at the same potential. Similarly, the lower plate of Q1 = 6µF × 100V = 600µC  …..(i)
this capacitor is at the same potential as that of the Q2 = 6µF × (100V − Vc )  …..(ii)
negative terminal of 30V battery. So, we can say that
Q2 = 6µF × (Vc − Vd )  …..(iii)
P.D. across 2 µF capacitor is also 30V.
Q2 = 6µF × Vd−  …..(iv)
1 9 . 4 0 | Electric Potential and Capacitance

From( ii(ii)
From ) andand((iii)
iii) 100V − Vc = Vc − Vd JEE Advanced/Boards
or 2Vc − Vd =
100V  (v)
…..…..(v)
Example 1: Two capacitors C1 = 1 µF and C2 = 4µF
And from ( iii ) and ( iv ) Vc − Vd =
Vd
are charged to a potential difference of 100 volts and
or Vc = 2Vd−  .. ( vi )
……..(vi) 200 volts respectively. The charged capacitors are now
connected to each other with terminals of opposite
From (v) and (vi)
sign connected together. What is the
100 200
 Fro m ( v(v)
From )= and ((vi)
and vi) Vd = V and Vc V V
3 3 V1
100
So that
So that Vc − Vd = V
3
+ + -- -- --
C1 C2 C1 C2
Example.13 A regular hexagon of side 10 cm has + + + + + + + +
a charge 5 µ C at each of its vertices. Calculate the
potential at the center of the hexagon.
q q V1

(a) Final charge on each capacitor in steady state?


q q (b) Decrease in the energy of the system?
O
Sol: The capacitors are connected parallel to each other.
q 10 cm q Thus the equivalent capacitance is given by C = C1+C2.
The charges stored in capacitor is given by Q = CV. The
Sol: The potential at the center of the hexagon is
1
due to 6 charges placed at its vertices. It is given by energy stored in each capacitor is given by U = CV 2 .
2
 1 q initial charge on C
= C V
= 100 µ F ;
V= 6 ×  1 1 1
 4 πε r 
 0  Initial charge on C
= 2 V2 800µF ; C1 V1 < C2 V2
C2 =
6F
When the terminals of opposite polarity are connected
a c
together, the magnitude of net charge finally is equal
6F 6F to the difference of magnitude of charges before
connection.
b d (Charge on C2 )i - (charge on C1 ) i
6F
(a) = (charge on C2 )f - (charge on C1 ) f
Let V be the final common potential difference across
Q2 -Q2 each.
a c
The charges will be redistributed and the system attains
Q 2 Q2 a steady state when potential difference across each
100V 1 3
-Q1 -Q2 capacitor becomes same.
b d
-Q2 Q2 C2 V2 − C1 V1 =C2 V + C1 V
(b) C1 V2 − C1 V2 800 − 100
=V = = 140 volts
Total potential at O is given by, C2 + C1 5

 1 q 5 × 10−6 Note that because C1 V1 < C2 V2 , the final charge


V =6×  = 6 × (9 × 109 ) × = 2.7 × 106 volt.
 4 πε r  0.1
 0  polarities are same as that of C2 before connection.
Final charge on C
=1 C1=
V 140µC
Final charge on C
= 2 C2=
V 560µC
Loss of energy= Ui − Uf
P hysi cs | 19.41

Loss of energy
Example 3: A 10 µF condenser C is charged through
1 1 1 1 resistance R of 0.1 MΩ from a battery of 1.5V. Find the
= C1 V12 + C2 V22 − C1 V 2 − C2 V 2
2 2 2 2 time required for the capacitor to get charged upto
0.75V for the circuits shown below.
1 1 1
1 (100 ) + 4 ( 200 ) − (1 + 4 )(140 )
2 2 2
=
2 2 2 R S
= 36000
=µJ 0.036J
E C C R
Note: the energy is lost as heat in the connected wires S
due to the temporary currents that flow while the
charge is being redistributed.
Sol: For RC circuit, the charge stored in the capacitor is
Example 2: Four identical metal plates are located in
given by
= q q0 1 − e− t/RC  . Taking log on both sides
the air at equal separation d as shown. The area of each  
plate is a. Calculate the effective capacitance of the we can get the value of time t.
arrangement across A and B.
(a) In the case of charging of a capacitor C through the
A
resistance R,
q
=q q0 1 − e− t/RC  ; e− t/RC= 1 −
  q0
B
q V 0.75 1
For a capacitor, q = CV = = =
q0 V0 1.5 2
Sol: The plates are connected in parallel combination
with each other. The equivalent capacitance is given =
by t RClog
= e2 (0.1 x106 )x(10x10−6 )loge 2
C = C1 +C2 +C3. The capacitance between A and B= is t RClog
= e 2 0.693 second.
2C
given by CAB = (b) In the case of the capacitor C being connected
3
directly to the battery initially, it acts like short circuit.
A
The capacitor will get charged instantaneously at t = 0
1 secs.
2
P
3 Hence, time cannot be calculated as per the requirement
4 of the question.
5 B
6 Example 4: Twelve capacitors, each having a capacitance
C, are connected to form a cube. Find the equivalent
capacitance between the diagonally opposite corners
such as A and B.
2 1 P 5 6 f
B B
A B
e
d
3 4 h Q/3
c b
F

Let us call the isolated plate as P.A capacitor is formed


Q/6
A g
A Q/3 E
by a pair of parallel plates facing each other. Hence we a

have three capacitor formed by the pairs (1,2), (3,4) and Sol: Applying the Kirchoff’s second law across diagonal
(5,6). The surface 2 and 3 are at same potential as that points A and B to find the charge distribution across
of A. The arrangement can be redrawn as a network of the branch of the circuit, and find the equivalent
three capacitors. capacitance.
2C 2ε A0 Suppose the points A and B are connected to a battey.
C=
AB =
3 3 d The charges appearing on some of the capacitors are
1 9 . 4 2 | Electric Potential and Capacitance

shown in figure suppose the positive terminal of the Let length and breadth of the capacitor be l and b
battery supplies a charge +Q through the point A. respectively and d be the distance between the plates
This charge is divided on the three plates connected as shown in figure then consider a strip at a distance x
to A. Looking from A, the three sides of the cube of width dx.
have identical properties and hence, the charge will
Now QR=x tan θ
be equally distributed on the three plates. Each of the
capacitors a, b and c will receive a charge Q/3. The And PQ = d-x tan θ ; where
negative terminal of the battey supplies a charge –Q tan θ =d/l,Capacitance of PQ
through the point B. This is again divided equally on
L
the three plates connected to B. Each of the capacitors
d, e and f gets equal charge Q/3.
A b
Now consider the capacitors g and h. As the three
plates connected to the point E form an isolated
system, their total charge must be zero. The negative
plate of the capacitor has a charge -Q/3. The two plates k1 ε0 (b dx) k1 ε0 (b dx)
= dC1 =
of g and h connected to E should have a total charge d − x tan θ xd
d−
Q/3. By symmetry, these two plates should have equal l
charges and hence each of these has a charge Q/6. The k1 ε0b / dx k1 ε0 A
capacitors a, g and d have charges Q/3, Q/6 and Q/3 = dC1 =
d (l − x ) d (l − x )
respectively.
k 2 ε0b ( dx )
And dC2 = capacitance of QR dC2 = ;
We have, VA − VB = ( VA − VE ) + ( VE − VF ) + ( VF − VB ) d tan θ
k 2 ε0 A ( dx )  d
Q / 3 Q / 6 Q / 3 5Q dC2 = … ∴tan θ = 
= + + = Xd  l
C C C 6C
Q 6 Now dC1 and dC2 are in series. Therefore, their
Ceq =
= C.
VA − VB 5 resulatant capacity dC will be given by

1 1 1 1 d (l − x ) X.d
Example 5: The Capacitance of a parallel plate = + , =then +
1 0 ( ) 1 0 A ( dx )
capacitor with plate area A and separation d is C. The dc dC 1 dC 2 dC K ε A dx K ε
space between the plates is filled with two wedges of
dielectric constants K1 and K 2 respectively (figure).=
Find 1 d  l − x X  d K 2 (l − x ) + K1 X 
 +  =
the capacitance of the resulting capacitor. dc ε0 A ( dx )  K1 K 2  ε0 AK1K 2 ( dx )

A
ε0 AK1K 2 ε0 AK1K 2
dC = dx , dC = dx
d K 2 (l − x ) + K1 X  d K 2l + (K1 − K1 ) X 
K2

d All such elemental capacitor representing DC are


connected in parallel.
K1

Now the capacitance of the given parallel plate capacitor


is obtained by adding such infinitesimal capacitors
Sol: As represented in the figure the dielectric material parallel from
are connected in series with each other across diagonal. x =L
X = 0 to X = L. i.e. C = ∫ dC ;
Consider one thin strip of the capacitor. The equivalent x =0
1 1 1 P
capacitance is given by = + . dc1
dC dC1 dC2
K1
These strips are connected to each other parallely such Q
that the equivalent capacitance of the capacitor is given dc2
d
by K2
x =L
C= ∫ dC
R

x =0 dx x
L
P hysi cs | 19.43

L
ε0 AK1K 2 KK11KK22εε00AA KK connected to a battery which supplies a positive charge
= ∫ d K l + (K dx ; CC == In 22
In
ln of magnitude Q to the upper plate. Find the rise in the
0  2 1 − K 2 ) X  (KK11 −−KK22) dd KK11 level of the liquid in the space between the plates.

Example 6: Figure (a) shows a parallel-plate capacitor Sol: The dielectric liquid experiences a electric force due
having square plates of edge a and plate-separtion d. to which it rises to the height h. This force is balanced
The gap between the plates is filled with a dielectric by weight of liquid.
medium of dielectric constant K which varies parallel to
an edge as =
K K 0 + ax  1
The situation is shown in figure. A charge −Q  1 −  is
 K
a a
 1
induced on the upper surface of the liquid and Q  1 − 
 K 
d K
at the surface in contact with the lower plate. The net
 1 Q
(x)
dx charge on the lower plate is −Q + Q  1 −  =− .
(a)
 K K
Consider the equilibrium of the liquid in the volume
(b)

Where K and α are constants and x is the distance from


the left end. Calculate the capacitance. ABCD. The forces on this liquid are
(a) The force due to the electric field at CD,
Sol: As the medium varies, consider a small strip of
the dielectric medium such that its capacitance is (b) The weight of the liquid,

dC =
(K0 + αx ) ε0adx . These strips are connected
(c) The force due to atmospheric pressure and
d (d) The force due to the pressure of the liquid below AB.
in parallel so the equivalent capacitance is given by As AB is in the same horizontal level as the outside
a surface, the pressure here is the same as the atmospheric
C = ∫ dC where a is the length of plates. pressure. The forces in (c) and (d), therefore, balance
0
each other. Hence, for equilibrium, the forces in (a) and
Consider a small strip of width dx at a separation x from
(b) should balance each other.
the left end. This strip forms a small capacitor of plate
Q
area adx. Its capacitance is dC =
(K0 + αx ) ε0adx The electric field at CD due to the charge Q is E1 =
2Aε0
d
The given capacitor may be divided into such strips with In the downward direction.
x varying from 0 to A. All these strips are connected in Q
parallel. The capacitance of the given capacitor is, The field at CD due to the charge –Q/K is E2 =
2Aε0K
Q Also in the downward direction.
D+ - + + + + + + +
- - -Q(1-1/K) (K + 1 ) Q .
The net field at CD is E1 + E2 =
h
A B 2Aε0K

 1
The force on the charge −Q  1 −  at CD is
-Q/K
 K

=
 1  (K + 1 ) Q
F Q 1 −  ; =
(K 2
)
− 1 Q2
a
(K0 + αx ) ε0adx ; 2
ε0 a  aα   K  2Aε0K 2Aε0K 2
C= ∫ d
= K +
d  0 2 

0 In the upward direction. The weight of the liquid
considered is hAρg.
Example 7: A parallel-plate capacitor is placed in such
a way that its plates are horizontal and the lower plate
thus, hAρg =
(K 2
)
− 1 Q2
; or h=
(K 2
)
− 1 Q2
is dipped into a liquid of dielectric constant K and 2 2 2
2Aε0K 2A K ε0 ρg
density ρ . Each plate has an area A. the plates are now
1 9 . 4 4 | Electric Potential and Capacitance

JEE Main/Boards

Exercise 1 Q.7 What capacitance is required to store an energy of


100kW h at a potential difference of 104 V?
Q.1 Calculate the equivalent capacitance in the
following circuit Q.8 A capacitor is filled with two dielectrics of the
same dimensions but of dielectric constants 2 and 3
4.5F respectively. Find the ratio of capacitances in the two
3F 3F
arrangements shown in figure A and B.

4.5F A B A B

Q.2 Find the potential of a sphere having charge of + - + - + -


5 µC and capacitance of 1nF.

d
Q. 3 Find the equivalent capacitance between A and B
in the given figure. Take C2= 10µF and B in the given (A) d/4 d/2 d/4
(B)
figure. Take C2= 10µF and C1 ,C3 ,C 4 ,C5 each equal to
4µF
C4 Q.9 Two metal plates separated by a distance d
constitute a parallel plate capacitor. A metal slab of
thickness (d/2) and same area as the plate is inserted
A B
between the plates. What is the ratio of the capacitances
C1 C2 C3 in the two cases?

C5
K1 C1 d/2
d
K2 C2 d/2
Q.4 Find the value of C if the equivalent capacitance
between the points A and B in the given figure is 1µF
(A)
1F
A C
A/2 A/2
4F
6F d K1 K2
8F
C3 C4
12F

2F (B)
2F Q.10 Keeping the voltage of the charging source
B
constant, what would be the percentage change in
the energy stored in a parallel plate capacitor if the
separation between its plates were to be decreased by
Q.5 A 4x10-6F capacitor is charged by a 200V supply. It 10%?
is then disconnected from the supply and is connected
across another uncharged 2x10-6F capacitor. How much
Q.11 A parallel plate air capacitor with its plates spaced
energy of the first capacitor is lost?
2cm apart is charged to a potential of 300 volts. What
will be the electric field intensity inside the capacitor,
Q.6 The plates of a parallel plate capacitor of area if the plates are moved apart to a distance of 5cm
100cm2 each and are separated by 2.0mm. The capacitor without disconnecting the power source? Calculate the
is charged by a 100V supply. Find energy store by the change in energy of the capacitor. Area of the plates is
capacitor. equal to A = 100cm2. Also solve the problem assuming
P hysi cs | 19.45

the entire operation was done after disconnecting the Q.16 Two charge particulars one is electron and other is
power source. Account for the change in energy in both negatively charged ion have a velocity directed parallel

(
the cases. ∈0 =9x10−12 SI units ) to the plates. They are sent separately into the field.
Both the electron and the ion have reveived their initial
kinetic energy by passing the same potential difference.
Q.12 Find the charge on the capacitor C in the following Which of the two particles will travel a greater distance
circuit in steady state. (parallel to plates shown by ‘I’) before hitting the
12V 2 positively charged plate, if both fly into the capacitor
at a point that is exactly in the middle of the distance
4
between the plates?
2C
6
L
+
V
Q.13 Three uncharged capacitors of capacitance C1,C2
and C3 are connected as shown in figure to one another _
and to points A, B and D potentials (φA), (φB) and (φD).
Determine the potential (φO) at point O. Q.17 A 3 µF capacitor is charged to a potential of 300
volt and 2 µF is charged to 200 volt. The capacitor
A are connected so that the plates of same polarity
C1 are connected together. What is the final potential
O difference between the plates of the capacitor after they
C2 C3 are connected? If instead of this the plates of opposite
B D polarity were joined together, what amount of charge
will flow and from which capacitor does it come?

Q.14 Determine the current through the battery in the


Q.18 In the following circuit, internal resistance of the
circuit shown.
battery r = 1Ω .
E K

C1 C1 R=2 C3
R1 C2 C2 C4
A B
R2 R=3
R3 E
R=1
(i) Immediately after the key K is closed and
(ii) In a long time interval, assuming that the parameters E = 4V, C1 = 8µF , C2 = 2µF , C3 = 6µF , C 4 = 4µF .
of the circuit are known.
Find the charge on plate of each capacitor.

Q.15 The lower plate of a parallel plate capacitor lies


Q.19 In the network given in the figure a potential
on an insulating plane. The upper plate is suspended
difference of 10 volts is applied across the two points
from one end of a balance. The two plates are joined
A and B find;
together by a thin wire and subsequently disconnected.
The balance is then counterpoised. A voltage V=5000 5F 4F
volt is applied between the plates. What additional A
8F 2F
mass should be placed to maintain balance? The 3F
distance between the plates is d=5mm and the area of 8F
6F 6F
each plate is A=100cm2? [all the elements other than B
plates are massless and nonconducting] (i) The potential difference across the capacitor of 2 µF .
(ii) The charge in both of 6 µF capacitors.

Rigid road
1 9 . 4 6 | Electric Potential and Capacitance

Q.20 In the figure shown here, each capacitance C1 in Q.26 A capacitor of capacitance C1=1.0µF withstands
the network is 3 µF and each capacitance C2 is 2µF the maximum voltage V1=6.0 kV while a capacitor of
capacitance C2 = 2.0µF withstands maximum voltage
(i) Compute the equivalent capacitance of the network
V2 = 4.0 kV. What voltage will the system of these two
between the points a and b.
capacitors withstand, if they are connected in series?
(ii) Calculate charge on each capacitors nearest to a and
b when Vab =900volt.
Q.27 Two identical parallel plate air capacitors are
(iii) With 900 volt across a and b, compute Vcd connected in one case in parallel and in the other in
series. In each case the plates of one capacitor are
C1 C1 C C1
a brought closer together by a distance a and the plates
of the other are moved apart by the same distance a.
C2 C2 C1
How will the total capacitance of each system change
b as result of such manipulation ?
C1 C1 D C1

Q.28 A 3 mega ohm resistor and 1 µF capacitor are


Q.21 If you have serval 2.0 µF capacitors, each capable connected in a single loop circuit with a source of
of withstantding 200 volts without breakdown, constant 4 volt. At one second after the connection is
how would you assemble a combination having an made what are the rates at which;
equivalent capacitance of; (i) The charge on the capacitor is increasing
(a) 0.40 µF or of (ii) Energy is being stored in the capacitor
(b) 1.2 µF , each capable of withstanding 100 volts. (iii) Joule heat is appearing in the resistor
(iv) Energy is being delived by the source
Q.22 Six 1 µF capacitors are so arranged that their
equivalent capacitance is 0.70 µF . If a potential
Q.29 A capacitor of capacity 1 µF is connected in closed
difference of 600 volt is applied to the combination,
series circuit with a resistance of 107 ohms, an open key
what charge will appear on each capacitor?
and a cell of 2V with negligible internal resistance:

Q.23 Two condensers are in parallel and the energy (i) When the key is switched on at time t=0, find;
of the combination is 103 J, when the difference of (a) The time constant for the circuit.
potential between their terminals is 2 volts. With the
(b) The charge on the capacitor at steady state.
same two condensers in series, the energy is 1.6x102 J
for the same differences of potential across the series (c) Time taken to deposit charge equaling half that at
combination. What are their capacities? steady state.
(ii) If after fully charging the capacitor, the cell is shorted
Q.24 A capacitor of capacitance C1=1.0 µF charged by zero resistance at time t=0, find the charge on the
upto a voltage V = 110V is connected in a parallel to capacitor at t = 50s.
the terminals of a circuit consisting of two uncharged
capacitor connected in series and possessing the
Q.30 An electric dipole, when held at 30o with respect
capacitance C2 = 2.0 µF and C3 = 3.0 µF . What charge
to a uniform electric field of 30 × 10 4 NC−1 experiences
will flow through the connecting wires?
a torque of 27 × 1026 N m. Calculate the dipole moment
of the dipole.
Q.25 Two capacitors A and B are connected in series
across a 100V supply and it is observed that the potential
Q.31 A regular hexagon of side 10 cm has a charge 5 µ
difference across them are 60V and 40V. A capacitor
C at each of its vertices. Calculate the potential at the
of 2 µF capacitance is now connected in parallel with
center of the hexagon.
A and the potential difference across with A and the
potential difference acorss B rise to 90V. Determine the
capacitance of A and B. Q.32 What is the work done in moving a 2 µ C point
charge from corner A to B of a square ABCD, when a 10
µ C charge exists at the center of the square?
P hysi cs | 19.47

Q.33 The potential at a point 0.1 m from an isolated Q.2 In the circuit shown, a potential difference of 60V
point charge is +100 volt. Find the nature and is applied across AB. The potential difference between
magnitude of the point charge. the point the point M and N is

A 2C
Q.34 Two charges equal to +20 µ C and -10 µ C are M
placed at points 6 cm apart. Find the value of the
60V
potential at a point distant 4 cm on the right bisector C C
of the line joining the two charges.
B N
2C
Q.35 A system has two charges qA = 2.5 × 10‒9 C and qB
= ‒2.5 × 10‒7C located at points A: (0, 0,-15) cm and B: (A) 10V (B) 15V (C) 20V (D) 30V
(0, 0, +15) cm, respectively. What are the total charge
and electric dipole moment of the system? Q.3 In the circuit shown in figure, the ratio of charges
on 5 µF and 4 µF capacitor:
Q.36 A test charge ‘q’ is moved without acceleration
2F
from A to C along the path from A to B and then from 3F
B to C in electric field E as shown in the figure. (i) 5F
Calculate the potential difference between A and C. (ii)
At which point (of the two) is the electric potential more
and why? 4F
(2, 3)
B E 6V
(A) 4/5 (B) 3/5 (C) 3/8 (D) ½

(2, 0) (6, 0) Q.4 From a supply of identical capacitors rated 8 µF ,


C A 250V the minimum number of capacitros required to
form a composite 16 µF , 1000V is:

Q.37 An electric dipole is held in a uniform electric field. (A) 2 (B) 4 (C) 16 (D) 32
(i) Show that the net force acting on it is zero.
Q.5 In the circuit shown, the energy stored in 1 µF
(ii) The dipole is aligned parallel to the field. capacitor is
3F 5F
Find the work done in rotating it through an angle of
180°.
1F

4F
Exercise 2
24V

Single Correct Choice Type


(A) 40 µJ (B) 64 µJ (C) 32 µJ (D) None

Q.1 A capacitor of capacitance C is charged to a potential


Q.6 If charge on left plane of the 5 µF capacitor in
difference V from a cell and then disconnected from
the circuit segment shown in the figure is -20 µC , the
it. A charge +Q is now given to its positive plate. The
charge on the right plate of 3 µF capacitor is
potential difference across the capacitor is now
3F
Q 2F
(A) V (B) V+ 5F
C

Q Q
(C) V+ (D) V- , if V< CV 4F
2C C
(A) +8.57 µC (B) -8.57 µC
(C) +11.42 µC (D) -11.42 µC
1 9 . 4 8 | Electric Potential and Capacitance

Q.7 What is equivalent capacitance of the system of Previous Years’ Questions


capacitors between A & B.
A Q.1 Seven capacitors each capacitance 2 µF are
connected in a configuration to obtain an effective
10
C C C capacitance µF . Which of the following combinations
11
B will achieve the desired result?  (1990)
(A) 7.6C (B) 1.6C (C) C (D) none

Q.8 Three capacitors 2 µF , 3 µF and 5 µF can withstand


(A) (B)
voltages to 3V, 2V and IV respectively. Their series
combination can withstand a maximum voltage equal
to
(A) 5Volts (B) (31/6) Volts
(C) (26/5) Volts (D) None (A) (B)
(A) (B)
(C) (D)
Q.9 A parallel plate capacitor has an electric field
of 105V/m between the plates. If the charge on the
capacitor plate is 1 µC , then the force on each capacitor
plate is
(A) 0.1 N (B)
(A)
0.05 N (C) 0.02 N (D) 0.01 N (B)
(C) (D)
(C) (D)

Q.10 A capacitor is connected to a battery. The force


of attraction between the plates when the separation
between them is halved.
(A) Remains the same   (B) Becomes eight times
(C) (D)
(C) Becomes four times   (D) Becomes two times

d
Q.11 A parallel plate capacitor is
make as shown in figure. Find Q.2 A parallel combination of 0.1MΩ resistor and
capacirance and equivalent dielectric k1 k2
a 10 µF capacitor is connected across a 1.5V source
constant. of negligible resistance. The time required for the
capacitor to get charged upto 0.75V is approximately
d/2 d/2
(in second)  (1997)
2 ε0 A(k1k 2 ) 4 ε0 A(k1k 2 )
(A) (B) (A) Infinite (B) loge2 (C) log102 (D) Zero
k1 + k 2 k1 + k 2
Q.3 For the circuit shown, which of the following
5 ε0 A(k1k 2 ) 3 ε0 A(k1k 2 )
(C) (D) statement is true? (1999)
k1k 2 k1 + k 2
V1 = 30 V V2 = 20 V
+ - + -

Q.12 A capacitor stores 60 µC charge when connected S1


C1= 2p F
S3
C2= 2p F
S2
across a battery. When the gap between the plates is
filled with a dielectric, a charge of 120 µC flows through
the battery. The dielectric constant of the material (A) With S1 closed, V1=15V, V2=20V
inserted is :
(B) With S3 closed,V1= V2 = 25V
(A) 1 (B) 2 (C) 3 (D) None
(C) With S1 and S2 closed, V1 = V2 = 0
(D) With S3 closed, V1 = 30 V, V2 = 20V
P hysi cs | 19.49

Q.4 Consider the situation shown in the figure. The Q.8 4 charges are placed each at distance ‘a’ from
capacitor A has a charge q on it whereas B is uncharged. origin. The dipole moment of configuration is (1983)
The charge appearing on the capacitor B after a long y
time after the switch is closed is (2001)
3q
q
+ -
+ - x
+ - -2q -2q
+ - S
+ - q
+ -
A B
(A) Zero (B) q/2 (C) q (D) 2q (A) 2 q a ˆj (B) 3 q a ˆj
(C) 2 a q [iˆ + ˆj] (D) None
Q.5 A parallel plate capacitor C with plates of unit are
and separation d is filled with a liquid of dielectric Q.9 n small drops of same size are charged to V volts
d each. If they coalesce to form a single large drop, then
constant K=2. The level of liquid is initially. Suppose
3 its potential will be (1985)
the liquid level decreases at a constant speed V, the (A) V/n (B) Vn (C) Vn1/3 (D) Vn2/3
time constant as a function of time t is (2008)
Q.10 A hollow metal sphere of radius 5 cm is charged
such that the potential on its surface is 10 V. The
potential at the center of the sphere is (1998)
C
(A) 0 V
d R
d (B) 10 V
3
(C) Same as at point 5 cm away from the surface outside
sphere.
(D) Same as a point 25 cm away from the surface.

(A)
6ε0R
(B)
(15d + 9vt ) ε0R Q.11 If the electric potential of the inner metal sphere
5d + 3vt 2d2 − 3dvt − 9v 2 t2 is 10 volt & that of the outer shell is 5 volt, then the
potential at the center will be: (1999)

(C)
6ε0R
(D)
(15d − 9vt ) ε0R (A) 10 volt (B) 5 volt (C) 15 volt (D) 0
5d − 3vt 2
2d + 3dvt − 9v t 2 2

Q.12 Three concentric metallic spherical shell A, B


Q.6 When a negative charge is released and moves in and C or radii a, b and c (a<b<c) have surface charge
electric field, it moves towards a position of  (1982) densities - σ , + σ , and - σ respectively. The potential
of shell A is : (1986)
(A) Lower electric potential and lower potential energy
(A) ( σ ε0 ) [a+b-c] (B) ( σ ε0 ) [a-b+c]
(B) Lower electric potential and higher potential energy
(C) ( σ ε0 ) [b-a-c] (D) None
(C) Higher electric potential and lower potential energy
(D) Higher electric potential and higher potential energy Q.13 A hollow metal sphere of radius 5 cm is charged
such that the potential on its surface is 10 V. The
Q.7 An infinite non - conducting sheet of charge has potential at the centre of the sphere is (1983)
a surface charge density of 10–7 C/m2. The separation (A) Zero
between two equipotential surfaces near the sheet
(B) 10 V
whose potential differ by 5V is (1983)
(C) Same as at a point 5 cm away from the surface
(A) 0.88 cm (B) 0.88 mm (C) 0.88 m (D) 5 × 10‒7 m
(D) Same as at a point 25 cm away from the surface
1 9 . 5 0 | Electric Potential and Capacitance

Q.14 A solid conducting sphere having a charge Q is Then the potential at the origin due to the above
surrounded by an uncharged concentric conducting system of charges is  (1998)
hollow spherical shell. Let the potential difference q
between the surface of the solid sphere and that of the (A) Zero (B) 8πε x ln2
0 0
outer surface of the hollow shell be V. If the shell is now
given a change of -3Q, the new potential difference q ln(2)
between the same two surfaces is  (1989) (C) Infinite (D)
4 πε0 x0
(A) V (B) 2 V (C) 4 V (D) -2 V
Q.19 Two identical metal plates are given positive
Q.15 Two identical thin rings, each of radius R, are charges Q1 and Q2 (< Q1 ) respectively. If they are
coaxially placed a distance R apart. If Q1 and Q2 are now brought close together to form a parallel plate
respectively the charges uniformly spread on the two capacitor with capacitance C, the potential difference
rings, the work done in moving a charge q from the between them is  (1999)
center of one ring to that of the other is (1992)
(A) (Q1 + Q2 ) / 2C (B) (Q1 + Q2 ) / C
(A) Zero
(C) (Q1 − Q2 ) / C (D) (Q1 − Q2 ) / 2C
(B) q(Q1 − Q2 )( 2 − 1) / 2(4 πε0R)

(C) q 2(Q1 + Q2 ) / (4 πε0R) Q.20 A parallel plate capacitor of area A, plate


separation d and capacitance C is filled with three
(D) q(Q1 / Q2 )( 2 + 1) 2(4 πε0R)
different dielectric materials having dielectric constants
K1 ,K 2 and K 3 as shown, If a single dielectric material
Q.16 Two point charges +q and –q are held fixed at (-d, is to be used to have the same capacitance C in this
0) and (d, 0) respectively of a x-y co-ordinate system. capacitor then its dielectric constant K is given by
Then  (1995)  (2000)
A/2 A/2
(A) The electric field E at all point on the x-axis has the
d
same direction. K1 K2 2
d
(B) Work has to be done in bringing a test charge from K3
∞ to the origin
(C) Electric field at all point on y-axis is along x-axis
(D) The dipole moment is 2qd along the x-axis 1 1 1 1 1 1 1
(A) = + + (B)
= +
K K1 K 2 2K 3 K K1 + K 2 2K 3
Q.17 A parallel plate capacitor of capacitance C is 1 K1K 2 K1K 3 K K
connected to a battery and is charged to a potential (C)
= + 2K 3 =(D) K + 2 3
K K1 + K 2 K1 + K 3 K 2 + K 3
difference V. Another capacitor of capacitance 2C is
similarly charged to a potential difference 2V. The
charging battery is now disconnected and the capacitors Q.21 A uniform electric field pointing in positive
are connected in parallel to each other in such a way x-direction exists in a region. Let A be origin, B be he
that the positive terminal of one is connected to the point on the x-axis at x=+1 cm and C be the point on
negative terminal of the other. The final energy of the y-axis at y=+1 cm. Then the potentials at the points
configuration is  (1995) A, B and C satisfy  (2001)
(A) VA < VB (B) VA > VB
3 25 2 9
(A) Zero (B) CV 2 (C) CV (D) CV 2 (C) VA < VC (D) VA > VC
2 6 2

Q.18 A charge +q is fixed at each of the points Q.22 Two equal point charges are fixed at x=-a and
=x x= 0, x 3x=
0 , x 5x 0 .....∞ on the x-axis x=+a on the x-axis. Another point charge Q is placed at
and a charge –q is fixed at each of the points the origin. The change in the electrical potential energy
=x 2x= 0, x 4x=0, x 6x0 .....∞ . Here, x0 is a positive of Q, when it is displaced by a small distance x along
constant. Take the electric potential at a point due to a the x-axis, is approximately proportional to (2002)
charge Q at a distance r from it to be Q/ 4 πε0r .
(A) x (B) x2 (C) x3 (D) 1/x
P hysi cs | 19.51

Q.23 Two identical capacitors, have the same y


capacitance C. One of them is charged to potential
V1 and the other to V2 . Likely charged plates are B
then connected. Then, the decrease in energy of the
combined system is (2002) C
x
60o O
1 1
(A) C(V12 − V22 ) (B) C(V12 + V22 )
4 4
A
1 1
(C) C(V1 − V2 )2 (D) C(V1 + V2 )2
4 4
q
(A) The electric field at point O is directed
2
Q.24 A long, hollow conducting cylinder is kept coaxially along the negative x-axis 8 πε 0R
inside another long, hollow conducting cylinder of
larger radius. Both the cylinders are initially electrically (B) The potential energy of the system is zero
neutral.  (2007)
(C) The magnitude of the force between the charges at
(A) A potential difference appears between the two
q2
cylinders when a charge density is given to the inner C and B is
cylinder 54 πε0R 2
q
(B) A potential difference appears between the two (D) The potential at point O is
cylinders when a charge density is given to the outer
12πε0R
cylinder
Q.27 A parallel plate capacitor C with plates of unit
(C) No potential difference appears between the two area and separation d is filled with a liquid of dielectric
cylinders when a uniform line charge is kept along the
d
axis of the cylinders constant K = 2. The level of liquid is initially. Suppose
3
(D) No potential difference appears between the two the liquid level decreases at a constant speed v, the
cylinders when same charge density is given to both time constant as a function of time t is  (2008)
the cylinders

Q.25 Positive and negative point charges of equal


 a  −a  C
magnitude are kept at 0,0,  and  0,0,  ,
 2  2  R
d d
respectively. The work done by the electric field when
3
another positive point charge in moved from (-a, 0, 0)
to (0, a, 0) is  (2007)
(A) Positive
(B) Negative
(C) Zero 6ε0R (15d + 9vt)ε0R
(A) (B)
(D) Depends on the path connecting the initial and final 5d + 3vt 2d2 − 3dvt − 9v 2 t2
position
6ε0R (15d − 9vt)ε0R
(C) (D)
q q 2q 5d − 3vt 2d2 + 3dvt − 9v 2 t2
Q.26 Consider a system of three charges , and −
3 3 3
placed at points A, B and C respectively, as shown in the
figure. Take O to the center of the circle of radius R and
angle CAB= 60o  (2008)
1 9 . 5 2 | Electric Potential and Capacitance

Q.28 A 2µF capacitor is charged as shown in the figure. Q.33 A uniformly charged solid sphere of radius R
The percentage of its stored energy dissipated after the has potential V0 (measured with respect to ∞ ) on
switch S is turned to potion 2 is (2011)
its surface. For this sphere the equipotential surfaces
1 2 3V0 5 V0 3 V0 V
with potentials , , and 0 have radius
2 4 4 4
R1 ,R 2 ,R3 and R 4 respectively. Then  (2015)

2F 8F (A) R1 ≠ 0 and (R 2 − R1 ) > (R 4 − R 3 )


(B) R1 = 0 and R 2 < (R 4 − R 3 )
(C) 2R < R 4

(A) 0% (B) 20% (C) 75% (D) 80% (D) R1 = 0 and R 2 > (R 4 − R 3 )

Q.29 Two capacitors C1 and C2 are charged to 120 V Q.34 In the given circuit, charge Q2 on the 2 µ F
and 200 V respectively. It isfound that by connecting capacitor changes as C is varied from 1 µ F to 3 µ F . Q2
them together the potential on each one can be as a function of ‘C’ is given properly by : (figures are
madezero. Then:  (2013) drawn schematically and are not to scale) (2015)

(A) 3C1 = 5C2 (B) 3C1 + 5C2 =


0
(C) 9C1 = 4 C2 (D) 5C1 = 3C2 1F
C
Q.30 A charge Q is uniformly distributed over a long 2F
rod AB oflength L as shown in the figure. The electric
potential at the point O lying at a distance L from the
end A is:  (2013) E

Charge Charge

O A B
(A) Q2 (B) Q2
L L
C C
1F 3F 1F 3F
3Q Q
(A) (B)
4 π ε0 L 4 π ε0 L ln 2 Charge Charge
Q ln 2 Q
(C) (D) (C) Q2 (D) Q2
4 π ε0 L 8 π ε0 L
C C
1F 3F 1F 3F
Q.31 A parallel plate capacitor is made of two circular
plates separated by a distance of 5 mm and with a
dielectric of dielectric constant 2.2 between them. When
the electric field in the dielectric is 3 × 104 V/m, the charge Q.35 A combination of capacitors is set up as shown in
density of the positive plate will be close to:  (2014) the figure. The magnitude of the electric field, due to
a point charge Q (having a charge equal to the sum of
(A) 3 × 10 4 C / m2 (B) 6 × 10 4 C / m2 the charges on the 4 µF and 9 µF capacitors), at a point
(C) 6 × 10−7 C / m2 (D) 3 × 10−7 C / m2 distant 30 m from it, would equal:  (2016)

 (A) 360 N/C (B) 420 N/C


Q.32 Assume that an electric field E = 30 x2 ˆi exists in
(C) 480 N/C (D) 240 N/C
space. Then the potential difference VA − V0 , where V0
is the potential at the origin and VA the potential at
x = 2 m is: (2014)
(A) – 80 V (B) 80 V
(C) 120 V (D) None of these
P hysi cs | 19.53

JEE Advanced/Boards

Exercise 1 Q.5 Find the charge on the capacitor C=1 µF in the


circuit shown in the figure.
Q.1 A parallel plate capacitor has plate area 100 cm2 and
place separation 1 cm. A glass plate (k = 6) of thickness C 1F
6 mm and an ebonite plate (k = 4) are inserted to fill the
gap between the plates. Find new capacitance. 6F 4F
8F

Q.2 In the given network (See figure) if Potential 2F 2F 12F
difference between p and q is 2V and C2=3C1. Then find
difference between a & b.
Q.6 The figure shows a circuit consisting of four capacitors.
+ -
Find the effective capacitance between X and Y

b 1F 1F
a C2 1F
C1 X Y
p 2F
q
C1 C2
Q.7 Five identical capacitor plates, each of area A are
Q.3 Find field produced in the circuit shown in figure on
arranged such that adjacent plates are at a distance‘d’
closing the switch S.
apart, the plates are connected to a source of emf V as
2F
shown in figure. The charge on plate 1 is ------ and that
+20C -20C on plate 4 is ----
S
+50C -50C
1 2 3 4 5 -
5F V
+

Q.4 In the following circuit (See figure), the resultant


capacitance between A and B is 1 µF . Find equivalent
capacitance of the circuit. Q.8 Find the capacitance of the system shown in figure.
Plate area = A
1F 1F 10V
d k=1 k=2
1F 1F
1F k=3 k=4
1F
1F d

1F 1F 1F


1F 1F C=1F 1F
Q.9 Figure shows three concentric conducting spherical
shells with inner and outer shells earthed and the
1F 1F 1F 1F 1F
middle shell is given a charge q. find the electrostatic
energy of the system stored in the region I and II.

3.5r

q
2.5r I
II
III
1 9 . 5 4 | Electric Potential and Capacitance

Q.10 Find the ratio between the erergy stored in 5µF (a) Charge on capacitor as a function of time.
capacitor to the 4 µF capacitor in the given circuit in
(b) Energy of the capacitor at t=20s.
stedy state.
5 5F Q.15 In the circuit shown in figure the capacitance of
each capacitor is equal to C and resistance R. One of
the capacitor was charged to a voltage V and then at
the moment t=0 was shorted by means of the switch
4F 4
S. find: R

2V
C C
Q.11 A solid conducting sphere of radius 10cm is S
enclosed by a thin metallic shell of radius 20cm. a W
charge q=20 µC is given to the inner sphere. Find
the heat generated in the process, the inner sphere is (a) The current in the circuit as a function of time t.
connected to the shell by a conducting wire. (b) The amount of generated heat.

Q.12 In the circuit shown here, At the steady state, the Q.16 The two identical parallel plates are given charges
charge on the capacitor is ------------. as shown in figure. If the plate area of either face of each
R1 plate is A and separation between plates is d, then find
the amount of heat liberated after closing the switch.
C
E R3 +3q +q
R2

Q.13 For the arrangement shown in the figure, the key


is closed at t=0. C2 is initially uncharged while C1 has a
charge of 2 µC S

30 C2

2F Q.17 Five identical conducting plates 1, 2, 3, 4 & 5 are


30 60 fixed parallel to and equidistant from each other (see
1F C1 figure). Plates 2 & 5 are connected by a conductor while
1 & 3 are joined by another conductor. The junction of
1 & 3 and the plate 4 are connected to a source of
K 9V
constant e.m.f. V0 find:
(a) Find the current coming out of the battery just after 5
switch is closed. 4 -
3
(b) Find the charge on the capacitors in the steady state
condition. 2
1 +
Q.14 In the circuit shown in figure R1=R2=6R3= 300M Ω , (i) The effective capacity of the system between the
C=0.01 µF and E=10V. The switch is closed at t=0, find terminals of the source.
S
(ii) The charges on plates 3 & 5. Given
R1 R3 D = Distance between any 2 successive plate &
R2 A = Area of either face of each plate.

E C
P hysi cs | 19.55

Q.18 Find the charge flown through the switch from A dielectric of dielectric  S
to B when it is closed. constant (or relative
permittivity) 3. Find
V A C B C
3F 3F 10V 6F the ratio of the total
5V 3F electrostatic enegry
6F A B stored in both capacitors
10V 5V 6F before and after the introduction of the dielectric.

Q.23 A parallel plate capacitor has plates with area A &


Q.19 Three capacitors of 2 µF , 3 µF and 5 µF are separation d. A battey charges the plates to a potential
independently charged with batteries of emf’s 5V, difference of V0. The battery is then disconnected
20V and 10V respectively. After disconnecting from & a di-electric slab of constant K & thickness d is
the voltage source. These capacitors are connected introduced. Calculate the positive work done by the
as shown in figure with their positive polarity plates system (capacitor+slab) on the man who introduces
connected to A and negative polarity is earthed. Now the slab.
a battery of 20V and an uncharged capacitor of 4 µF
capacitance are connected to the junction A as shown Q.24 A parallel plate capacitor is filled by a di-electric
with a switch S. when switch is closed, find: whose relative permittivity varies with the applied
voltage according to the law= α V, where α =1 per
3F volt. The same (but containing no di-electric) capacitor
2F
charged to a voltage V=156 volt is connected in parallel
A 20 V 4F to the first “non-linear” uncharged capacitor. Determine
5F the final voltage Vf across the capacitors.

(a) The potential of the junction A. Q.25 Two parallel plate capacitors A & B have the same
separation d=8.85+104m between the plates. The plate
(b) Final charge on all four capacitros. areas of A & B are 0.04 m2 & 0.02m2 respectively. A
slab of di-electric constant (relative permittivity) K=9
Q.20 In the circuit shown in figure, find the amount of has dimensions such that it can exactly fill the space
heat generated when switch s is closed. between the plates of capacitor B.
2F S
A B A B

5V 10V 3F

10V
4F
(i) The di-electric slab is placed inside A and A is then
Q.21 the connection shown in figure are established charged to a potential difference of 110volt. Calculate
with the switch S open. How much charge will flow the capacitance of A and the energy stored in it.
through the switch if it is closed? (ii) The battery is disconnectd & then the di-electric slab
is removed from A. find the work done by the external
1F 2F agency in removing the slab from A.

24V
S (iii) The same di-electric slab is now placed inside B,
filling it completely. The two capacitors A & B are then
2F 1F
connected. Calculate the energy stored in the system.

Q.26 Two square metallic plates of 1m side are kept 0.01m


Q.22 The figure shows two identical parallel plate apart, like a parallel plate capacitor, in air in such a way that
capacitors connected to a battery with the switch S one of their edges is perpendicular, to an oil surface in a
closed. The switch is now opened and the free space tank filled with an insulating oil. The plates are connected
between the plates of the capacitors is filled with a
1 9 . 5 6 | Electric Potential and Capacitance

to a battery of e.m.f. 500 volt. The plates are then lowered Q.32 A dipole is placed at origin of coordinate system
vertically into the oil at a speed of 0.001 m/s. calculate the as shown in figure, find the electric field at point P (0, y).
current drawn from the battery during the process. y
[Di-electric constant of oil= ε0 =8.85x10 C / N m ]
12 2 2 2
P (0,y)

Q.27 A 10 µF and 20 µF capacitor are connected to a 10V


x
cell in parallel for some time after which the capacitors P 45
o

are disconnected from the cell and reconnected at


t=0 with each other, in series, through wires of finite
resistance. The +ve plate of the first capacitor. Draw the Q.33 Two concentric rings of radii r and 2r are placed
graph which best describes the charge on the +ve plate with center at origin. Two charges +q each are fixed at
of the 20 µF capacitor with increasing time. the diametrically opposite points of the rings as shown
in figure. Smaller ring is now rotated by an angle 90c
Q.28 A capacitor of capacitance C0 is charged to a about Z-axis then it is again rotated by 90o about
potential V0 and then isolated. A small capacitor C is Y-axis. Find the work done by electrostatic force in each
then charged from C0, discharged and charged again, step. If finally larger ring is rotated by 90o about X-axis,
the process being repeated in times. The potential of the find the total work required to perform all three steps.
large capacitor has now fallen to V. Find the capacitance y
of the small capacitor. If V0=100volts, V=35volts, find +q
the value of n for C0=0.2 µF and C=0.01075 µF . Is it
possible to remove charge on C0 this way? +q

Q.29 In the figure shown initially switch is open for a x


long time. Now the switch is closed at t=0. Find the
charge on the rightmost capacitor as a function of time +q
given that it was initially unchanged.
z +q

R S
V R C C Q.34 A cone made of insulating material has a total
charge Q spread uniformly over its sloping surface.
Calculate the energy required to take a test charge q
from infinity to apex A of cone.
Q.30 Two capacitors A and B with capacitor are connected
as shown in figure with one wire from each capacitor
free. The upper plate of A is positive and that of B is A
negative. An uncharged 2 µF capacitor C with lead wires
falls on the free ends to complete the circuit. Calculate: AB=L
B
C 2F
The slant length is L.
+ -
3F 2F
A 100V B 180V Q.35 A non-conducting disc of radius a and uniform
positive surface charge density σ is placed on the
(i) The final charges on the three capacitors (ii) The ground, with its axis vertical. A particle of mass m &
amount of electrostatic energy stored in the system positive charge q is dropped, along the axis of the disc,
before and after the completion of the circuit. from a height H with zero initial velocity. The particle
q 4 ε0 g
has = .
m σ
Q.31 Three charges 0.1 coulomb each are placed on
the comers of an equilateral triangle of side 1 m. If the (a) Find the value of H if the particle just reaches the
energy is supplied to this system at the rate of 1 kW, disc.
how much time would be required to move one of the (b) Sketch the potential energy of the particle as a
charges onto the midpoint of the line joining the other function of its height and find its equilibrium position.
two?
P hysi cs | 19.57

Exercise 2 Q.6 Four metallic plates are arranged as shown in the


figure. If the distance between each plate is d, The
Single Correct Choice Type capacitance of the given system between points A and
B is (Given d<<A)

Q.1 Two capacitor having capacitance 8 µF and 16 d


µF have breaking voltage 20V and 80V. They are d
combined in series. The maximum charge they can A
d
B
store individually in the combination is:
(A) 160 µC (B) 200 µC
ε0 A 2ε0 A 3ε0 A 4 ε0 A
(C) 1280 µC (D) None of these (A)   (B)   (C)   (D)
d d d d

Q.2 Three plate A, B and C each of area Q.7 Find the equivalent capacitance across A and B
0.1m2 are separated by 0.885mm from A B
each other as shown in the figure. A C 10V
d
10V battery is used to charge the d
A B
system. The energy stored in the system is d
(A) 1µ J (B) 10-1µ J (C) 10-2µ J (D) 10-3µ J
ε0 A 2ε0 A 3ε0 A 4 ε0 A
(A)   (B)   (C)   (D)
d d d d
Q.3 A capacitor of capacitance C is initially charge to a
potential difference of V. Now it is connected to battery
Q.8 The diagram shows four capacitors with
of 2V with opposite polarity. The ratio of heat generated
capacitances and break down voltage as mentioned.
to the final erergy stored in the capacitor will be
What should be the maximum value of the external emf
(A) 1.75 (B) 2.25 (C) 2.5 (D) ½ source such that no capacitor breaks down? [Hint: First
of all find out the break down voltage of each branch.
Q.4 Five conducting parallel plates having E After that compare them.]
area A and separation between them A B
123 45 3C; 1kV 2C; 2kV
being d, are placed as shown in the figure.
Plate number 2 and 4 are connected with 1 2
a wire and between point A and B, a cell
of emf E is connected. The charge flown 7C; 1kV 3C; 2kV
E
3 ε0 AE 2 ε0 AE 4ε0 AE ε0 AE
(A) (B) (C) (D)
4 d 3 d d 2d (A) 2.5kV    (B) 10/3kV   (C) 3kV   (D) 1kV

Q.5 The plates S and T of an uncharged parallel plate Q.9 A conducting body 1 has some initial charge Q, and
capacitor are connected acorss a battey. The battery is its capacitance is C. There are two other conducting
then disconnected in a system as shown in the figure. bodies, 2 and 3, having capacitances: C2=2C and C3→ ∞.
The system shown is in equilibrium. All the strings are Bodies 2 and 3 are initially uncharged. “Body 2 is touched
insulating and massless. The magnitude of charge on with body 1 and touched with body 3, and the removed.”
one of the capacitor plates is: [Area of plates = A] This process is repeated N times. Then, the charge on
k S T body 1 at the end must be
KCE
(A) Q/3n (B) Q/3n-1
K +1
m (C) Q/2n (D) None

4mgAε0
(A) 2mgAε0 (B) Q.10 In the adjoining figure, capacitor (1) and (2) have
k a capacitance ‘C’ each. When the dielectric of dielectric
2mgAε0 constant K is inserted between the plates of one of the
(C) mgAε0 (D) capacitor, the total charge flowing through battery is
k
1 9 . 5 8 | Electric Potential and Capacitance

Conductor Dielectric Q.13 Condenser A has a capacity of 15µF when it is


filled with a medium of dielectric constant 15. Another
condenser B has a capcity 1 µF with air between the
plates. Both are charged separately by a battery of
100V. After charging, both are connected in parallel
without the battery and the dielectric material being
removed. The common potential now is
x-0 x-0 x-2d x-3d x-4d x-5d C

(A) From B to C; n
n
KCE
(B) From C to B
K +1 n
S

(K − 1)CE 9V
(C) From B to C;
2(K + 1) (A) 400V (B) 800V

(K − 1)CE (C) 1200V (D) 1600V


(D) From C to B
2(K + 1)
Q.14 Two identical capacitors 1 and 2 are connected
in series to a battery as shown in figure. Capacitor
Q.11 The distance between plates of a parallel plate 2 contains a dielectric slab of dielectric constant
capacitor is 5d. Let the positively charged plate is at k as shown. Q1 and Q2 are the charges stored in the
X = 0 and negatively charged plate is at X = 5d. Two capacitors. Now the dielectric slab is removed and the
slabs one of conductor and other of a dielectric of corresponding charges are Q’1 and Q’2. Then
equal thickness d are inserted between the plates as
E C
shown in figure. B

Potential versus distance graph will look like: C C C


A
D
v v 1 2

Q1′ k +1 Q′ k + 1
(A) (B) (A) = (B) 2 =
Q1 k Q2 2

x x Q2′ k +1 Q′ k
(C) = (D) 1 =
v v Q2 2k Q1 2

Q.15 Four identical plates 1, 2, 3 and 4 are placed


(C) (D) parallel to each other at equal distance as shown in the
figure. Plates 1 and 4 are joined together and the space
x x between 2 and 3 is filled with a dielectric of dielectric
constant k=2. The capacitance of the system between 1
and 3 & 2 and 4 are C1 and C2 respectively.
Q.12 The distance between the plates of a charged
parallel plate capacitor is 5cm and electric field inside C1
The ratio is:
the plates is 200V cm-1. An uncharged metal bar of width C2
2cm is fully immersed into the capacitor. The length of 1
the metal bar is same as that of plate of capacitor. The
voltage across capacitor after the immersion of the bar 2
is 3
(A) Zero (B) 400V 4
(C) 600V (D) 100V 5 3 5
(A) (B) 1 (C) (D)
3 5 7
P hysi cs | 19.59

Q.16 A charged capacitor is allowed to discharge Q.20 A charged capacitor is allowed to discharge
through a resistance 2Ω by closing the switch S at through a resistor by closing the key at the instant t=0.
the instant t = 0. At time t = ln2 µs , the reading of the At the instant t=(ln 4) µS , the reading of the ammeter
ammeter falls half of its initial value. The resistance of falls half the initial value. The resistance of the ammeter
the ammeter is equal to is equal to
0.5F (A) 1 MΩ    (B) 1 Ω    (C) 2 Ω   (D) 2 MΩ
+ -
S
Multiple Correct Choice Type
A
2 Q.21 Two capacitors of 2 µF and 3 µF are charged to 150
(A) 0  (B) 2Ω   (C) 2KΩ   (D) 2MΩ volt and 120 volt respectively. The plates of capacitor
are connected as shown in the figure. A discharged
capacitor of capacity 1.5 µF falls to the free ends of the
Q.17 A capacitor C = 100 µF is connected to three wire. Then
resistor each of resistance 1k Ω and a battery of emf 1.5F
9V. The switch S has been closed for long time so as
to charge the capacitor. When switch S is opened, the
capacitor discharges with time constant +
2F
-
3F
150V 120V
C-0.5F - +
A
+ -
(A) Charge on the 1.5 µF capacitors is 180 µC

A (B) Charge on the 2 µ F capacitor is 120 µ C


2
(C) Positive charge flows through A from right to left.
(A) 33ms    (B) 5ms   (C) 3.3ms    (D) 50ms
(D) Positive charge flows thorugh A from left to right.

Q.18 In the circuit shown, when the key k is pressed


Q.22 In the circuit shown, each capacitor has a
at time t=0, which of the following statements about
capacitance C. The emf of the cell is E. If the switch S is
current I in the resistor AB is true.
closed C
K B
C C
2V 1000
1000 + -
E

(A) Positive charge will flow out of the positive terminal


(A) I = 2mA at all t of the cell.
(B) I oscillates between 1mA and 2mA (B) Positive charge will enter the positive terminal of
(C) I = 1mA at all t the cell
(D) At t = 0, I = 2mA and with time it goes to 1mA (C) The amount of charge flowing through the cell will
be CE.
Q.19 In the R-C circuit shown in the figure the total (D) The amount of charge flowing through the cell will
energy of 3.6x10-3 J is dissipated in the 10 Ω resistor be 4/3 CE.
when the switch S is closed. The initial charge on the
capacitor is
Q.23 A circuit shown in the figure consists of a battery of
S emf 10V and two capacitance C1 and C2 of capacitances
2F 10 1.0 µF and 2.0 µF respectively. Which of the options are
correct?
A B
C1 e C2
60
(A) 60 µC (B) 120 µC (C) 60 2µC (D) µC
2
1 9 . 6 0 | Electric Potential and Capacitance

(A) Charge on capacitor C1 is equal to charge on Q.27 A parallel plate air-core capacitor is connected
capacitor C2 across a source of constant potential difference. When a
dielectric plate is introduced between the two plates then:
(B) Voltage across capacitor C1 is 5V
(A) Some charge from the capacitor will flow back into
(C) Voltage across capacitor C2 is 10V
the source.
(D) Energy stored in capacitor C1 is two times the energy
(B) Some extra charge from the sourch will flow back
stored in capacitor C2.
into the capacitor.

Q.24 If Q is the charge on the plates of a capacitor of (C) The electric field intensity between the two plates
capacitance C, V the potential difference between the does not charge.
plates, A the area of each plate and d the distance (D) The electric field intensity between the two plates
between the plates, The force of attraction between the will decrease.
plates is

1  Q2  1  CV 2  Q.28 A parallel plate capacitor of plate area A and plate


(A)   (B)   separation d is charged to potential difference V and
2  ε0 A  2  d 
then the battery is disconnected. A slab of dielectric
constant K is then inserted between the plates of the
1  CV 2  1  Q2  capacitor so as to fill the space between the plates. If Q,
(C)   (D)  
2  Aε0  4  πε d2  E and W denote respectively, the magnitude of charge
 0 
on each plate, the electric field between the plates (after
the slab is inserted) and the work done on the system,
Q.25 Four capacitors and a battery are connected as
in question, in the process of inserting the slab, then
shown. The potential drop across the 7 µF capacitor is
6V. Then the; ε0 AV ε0KAV
(A) Q = (B) Q =
12F d d

V ε0 AV 2  1
E (C) E = (D)
= W 1 − 
7F Kd 2d  K
3.9F
3F Q.29 A parallel plate capacitor has a parallel slab
of copper inserted between and parallel to the two
(A) Potential difference across the 3 µF capacitor is 10V plates, without touching the plates. The capacity of the
capacitor after the introduction of the copper sheet is:
(B) Charge on the 3 µF capacitor is 42 µC
(A) minimum when the copper slab touches one of the
(C) Emf on the battery is 30V plates.
(D) Potential difference across the 12 µF capacitor is 10V (B) Maximum when the copper slab touches one of the
plates.
Q.26 The capacitance of a parallel plate capacitor is C
(C) Invariant for all positions of the slab between the
when the region between the plates has air. This region
plates.
is now filled with a dielectric slab of dielectric constant
k. the capacitor is connected to a cell of emf E, and the (D) Greater than that before introducing the slab.
slab is taken out
(A) Charge CE (k-1) flows through the cell Q.30 The plates of a parallel plate capacitor with no
dielectric are connected to a voltage source. Now a
(B) Energy E2C (k-1) is absorbed by the cell. dielectric of dielectric constant K is inserted to fill the
(C) The energy stored in the capacitor is reduced by whole space between the plates with voltage source
E2C (k-1) remaining connected to the capacitor.
1 (A) The energy stored in the capacitor will become
(D) The external agent has to do E2C(k − 1) amount of
work to take the slab out. 2 K-times
(B) The electric field inside the capacitor will decrease
to K-times
P hysi cs | 19.61

(C) The force of attraction between the plates will Assertion Reasoning Type
increase to K2 times.
(A) Statement-I is true, statement-II is true and
(D) The charge on the capacitor will increase to K-times statement-2 is correct explanation for statement-I.
(B) Statement-I is true, statement-II is true and
Q.31 A parallel-plate capacitor is connected to a cell. Its statement-II is not the correct explatnation for
positive plate A and its negative palte B have charges +Q statement-I.
and –Q respectively. A third plate C, identical to A and
(C) Statement-I is true, statement-II is false.
B, with charge+Q, is now introduced midway between
A and B, parallel to them, which of the following are (D) Statement-I is false, statement-II is true.
correct?
3Q Q.34 Statement-I: The electrostatic force between the
(A) The charge on the inner face of B is now −
2 plates of a charged isolated capacitor decreases when
dielectric fills whole space between plates.
(B) There is no change in the potential difference
between A and B. Statement-II: The electric field between the plates of a
(C) The potential difference between A and C is one- charged isolated capacitance decreases when dielectric
third of the potential difference between B and C. fills whole space between plates.

(D) The charge on the inner face of A is now Q/2. y y

(A) (B)
Q.32 The circuit shown in the figure consists of a battery
of emf ε =10V; a capacitor of capacitance C=1.0 µF and (d,0) (3d,0)
x x
(d,0) (3d,0)
three resistor of values R1=2 Ω , R2=2 Ω and R3=1 Ω .
Initially the capacitor is completely uncharged and the y
y
switch S is open. The switch S is closed at t=0.
R1 (C)
(D)
S x
(d,0) (3d,0) x
(d, 0) (3d,0)
 R2 R3

Q.35 Statement-I: If temperature is increased, the


(A) The current through resistor R3 at the moment the dielectric constant of a polar dielectrc decreases
switch closed is zero. whereas that of a non-polar dielectric does not change
significantly.
(B) The current through resistor R3 a long time after
the switch closed is 5A Statement-II: The magnitude of dipole moment of
(C) The ratio of current through R1 and R2 is always individual polar molecule decreases significantly with
constant. increase in temperature.

(D) The maximum charge on the capacitor during the


operation is 5 µC Q.36 Statement-I: The heat produced by a resistor in
any time t during the charging of a capacitor in a series
circuit is half the energy stored in the capacitor by that
Q.33 In the circuit shown in figure C1=C2=2 µF Then time.
charge stored in
Statement-II: Current in the circuit is equal to the rate
1 2 3
of increase in charge on the capacitor.
C1 C2

2 1 3

120V

(A) Capacitor C1 is zero (B) Capacitor C2 is zero


(C) Both capacitor is zero (D) Capacitor C1 is 40 µC
1 9 . 6 2 | Electric Potential and Capacitance

Comprehension Type Q.40 A long time after closing the switch


(A) Voltage drop acorss the capacitor is E
Paragraph 1: Two capacitors each having area A and
plate separation‘d’ are connected as shown in the E
(B) Current through the battery is
circuit. Each capacitor carries charge Q0/2. R1 + R 2
Q0 2
1  R 2E 
2 (C) Energy stores in the capacitor is C  
2  R1 + R 2 

(D) Current through the capacitor becomes zero.


Q0
2 Paragraph 3: The charge across the capacitor in two
different RC circuit 1 and 2 are plotted as shown in
The plates of one capacitor are slowly pulled apart by figure.
an external agent till the separation between them q
becomes 2d. The other capacitor is not disturbed. qmax 1
2
Q.37 The force applied by the external agent when the
separation between the plates is ‘X’ is given by O t

Q 02 d Q 02 d Q20 Q 02 d
(A) (B) (C) (D) Q.41 Choose the correct statement (S) related to the
3 Aε0 12 Aε0 2 Aε0 6 Aε0
two circuits.
(A) Both the capacitors are charged to the same charge.
Q.38 At this position the potential difference across the
capacitors is given by (B) The emf’s of cells in both the circuit are equal.

Q0 xd Q0 xd (C) The emf’s of the cell may be different.


(A) (B)
2Aε0 (x + d) Aε0 (x + d) (D) The emf’s E1 is more than E2

Q0 xd Q0 d
(C) (D) Q.42 Identify the correct statement (s) related to the R1,
4Aε0 (x + d) 2Aε0 x
R2, C1 and C2 of the two RC circuits.

(A) R1 > R2 if E1 =E2 (B) C1< C2 If E1=E2


Paragraph 2: In the circuit as shown in figure the switch
is closed at t = 0. R1 C2
(C) R1C1> R2C2 (D) <
R1 R2 C1

E C R2 Previous Years’ Questions


S
Q.1 A parallel plate capacitor of capacitance C is
Q.39 At the instant of closing the switch connected to a battery and is charged to a potential
(A) The battery delivers maximum current difference V. Another capacitor of capacitance 2C is
similarly charged to a potential difference 2V. The
(B) No current flows through C charging battery is now disconnected and the capacitors
(C) Voltage drop across R2 is zero are connected in parallel to each other in such a way
that the positive terminal of one is connected to the
(D) The current through the battery decreases with time
negative terminal of the other. The final energy of the
and finally becomes zero.
configuration is  (1995)

3 2 25 2 9 2
(A) Zero (B) CV (C) CV (D) CV
2 6 2
P hysi cs | 19.63

Q.2 Two identical metal plates are given positive charges Q.6 A parallel plate air capacitor is connected to a
Q1 and Q2(<Q1) respectively. If they are now brought battery. The quantities charge, voltage, electric field
close together to form a parallel plate capacitor with and energy associated with this capacitor are given by
capacitance C, the potential difference between them Q0, V0, E0 and U0 respectively. A dielectric slab is now
is  (1999) introduced to fill the space between the plates with the
battery still in connection. The corresponding quantities
(A) (Q1+Q2)/2C (B) (Q1+Q2)/C
now given by Q, V, E and U are related to the previous
(C) (Q1-Q2)/C (D) (Q1-Q2)/2C one as  (1985)
(A) Q>Q0 (B) V>V0
Q.3 A parallel plate capacitor of area A, plate separation
d and capacitance C is filled with three different (C) E>E0 (D) U> U0
dielectric materials having dielectric constants K1, K2
and K3 as shown. If a single dielectric material is to be Q.7 A parallel plate capacitor is charged and the
used to have the same capacitance C in this capacitor charging battery is then disconnected. If the plates
then its dielectric constant K is given by  (2000) of the capacitor are moved farther apart by means of
insulating handles  (1987)
A/2 A/2
(A) The charge on the capacitor increases
d
K1 K2 2 (B) The voltage across the plates increases
d (C) The capacitance increases
K3
(D) The electrostatic energy stored in the capacitor
increases.

A = Area of plates Q.8 A parallel plate capacitor of plate area A and plate
separation d is charged to potential difference V and
1 1 1 1 1 1 1 then the battery is disconnected. A slab of dielectric
(A) = + + (B)
= + constant K is then inserted between the plates of the
K K1 K 2 2K 3 K K1 + K 2 2K 3
capacitor so as to fill the space between the plates. If Q,
E and W denote respectively, the magnitude of charge
1 K1K 2 K1K 3 K K on each plate, the electric field between the plates (after
(C)
= + 2K 3 (D)
= K + 2 3
K K1 + K 2 K1 + K 3 K 2 + K 3 the slab in inserted), and work done on the system, in
question, in the process of inserting the slab, then
Q.4 Two identical capacitors, have the same capacitance  (1991)
ε0 AV ε0KAV
C. One to them is charged to potential V1 and the other (A) Q = (B) Q =
to V2. Likely charged plates are then connected. Then, d d
the decrease in energy of the combined system is
V ε0 AV 2  1
 (2002) (c) E = (D)
= W 1 − 
Kd 2d  K 
1
( )
(A) C V1 − V2 (B) C V1 + V2
4
2 2 1
4
2
( 2
)
Q.9 A dielectric slab of thickness d is inserted in a
1 1
C ( V1 − V2 ) (D) C ( V1 + V2 )
2 2
(C) parallel plate capacitor whose negative plate is at X=0
4 4 and positive plate is at X=3d. the slab is equidistant
from the plates. The capacitor is given some charge. As
1 2 X goes from 0 to 3d  (1998)
Q.5 A 2 µF capacitor is charged as 
shown in the figure. The percentage
(A) The magnitude of the electric field remains the same
of its stored energy dissipated after
the switch S is turned to position 2 is 2F 8F (B) The direction of the electric field remains the same
 (2011) (C) The electric potential increases continuously
(A) 0% (B) 20%  (D) The electric potential increases at first, then
(C) 75% (D) 80% decreases and again increases
1 9 . 6 4 | Electric Potential and Capacitance

Q.10 Two point charges Q and –Q/4 are separated by a (C) The direction of electric field at all points on the axis
distance x. Then (2001) will be along the axis.
Q x -Q/4 (D) If the ring is placed inside a uniform external electric
field then net torque and force acting on the ring would
(A) Potential is zero at a point on the axis which is x/3 be zero.
on the right side of the charge –Q/4.
Q.14 An electric dipole is placed at the center of a
(B) Potential is zero at a point on the axis which is x/5
sphere. Mark the correct answer (2003)
on the left side of the charge –Q/4.
(A) The flux of the electric field through the sphere is zero
(C) Electric field is zero at a point on the axis which is at
a distance x on the right side of the charge –Q/4. (B) The electric field is zero at every point of the sphere.
(D) There exists two points on the axis where electric (C) The electric potential is zero everywhere on the
field is zero. sphere.
(D) The electric potential is zero on a circle on the surface.
Q.11 An electric charge 10−8 C is placed at the point
(4m, 7m, 2m). At the point (1m, 3m, 2m), the electric
Q.15 For the situation shown in the figure below
 (2000)
(assume r>> length of dipole) mark out the correct
(A) Potential will be 18 V statement (s) (2002)
(B) Field has no Y-component p
Q (small dipole)
(C) Field will be along Z-axis r
(D) Potential will be 1.8 V (A) Force acting on the dipole is zero
pQ
Q.12 Potential at a point A is 3 volt and at a point B is (B) Force acting on the dipole is approximately
7 volt, an electron is moving towards A from B. (2004) and is acting upward 4 πε0r 3

(A) It must have some K.E. at B to reach A pQ


(C) Torque acting on the dipole is in clockwise
(B) It need not have any K.E at B to reach A direction 4 πε0r 2
pQ
(C) to reach A it must have more than or equal to 4 eV (D) Torque acting on the dipole is in anti-
K. E at B. clockwise direction. 4 πε0r 2

(D) When it will reach A, it will have K.E. more than or at


least equal to 4eV if it was released from rest at B. Q.16 For the situation shown in the figure below, mark
out the correct statement (s) (2006)
Q.13 The figure shows a non - conducting ring which
has positive and negative charge non uniformly B
distributed on it such that the total charge is zero. q
Which of the following statements is true? (2007) d R

+ - Hollow neutral conductor


+ -
+ - q
+ - (A) Potential of the conductor is .
4 πε0 (d + R)
+
+ - -
+ O q
(B) Potential of the conductor is .
-
+ - axis 4 πε0 d
+ -
+
(C) Potential of the conductor can’t be determined as
-
nature of distribution of induced charges is not known
(A) The potential at all the points on the axis will be zero.
(D) Potential at point B due to induced charges is
(B) The electric field at all the points on the axis will be −qR
zero. 4 πε0 (d + R)d
P hysi cs | 19.65

Paragraph for Question No. 17 to 19 Q.18 The charge appearing on right side of plate 4 is
 (2009)
Four metallic plates are placed as shown in the figure.
Plate 2 is given a charge Q whereas all other plates are (A) Zero (B) Q/4 (C) -3Q/4 (D)+Q/2
uncharged. Plates 1 and 4 are jointed together. The
area of each plate is same. Q.19 The potential difference between plates 1 and 2 is
1 2 3 4  (2009)
Q
3 Qd Qd 3 Qd 3Qd
(A) (B) (C) (D)
2 ε0 A ε0 A 4 ε0 A ε0 A

(A) If statement-I is true, statement-II is true; statement


II is the correct explanation for statement-I.
d 2d d (B) If statement-I is true, statement-II is true;
statement- II is not a correct explanation for statement-I.
(C) If statement-I is true; statement-II is false.
Q.17 The charge appearing on the right on the right
side of plate 3 is (2009) (D) If statement-I is false; statement-II is true.
(A) Zero (B)+Q/4 (C) -3Q/4 (D)Q/2
Q.20 Statement-I: For practical purposes, the earth is
used as a reference at zero potential in electrical circuits.
Statement-II: The electrical potential of a sphere of
radius R with charge Q uniformly distributed on the
Q
surface is given by  (2008)
4 π ∈0 R

Q.21 Six point charges, each of the same magnitude q, are arranged in different manners as shown in column
II. In each case, a point M and line PQ passing through M (potential at infinity is zero) due to the given charge
distribution when it is at rest. Now, the whole system is set into rotation with a constant angular velocity about the
line PQ. Let B be the magnetic field at M and µ be the magnetic moment of the system in this condition. Assume
each rotating charge to be equivalent to a steady current.  (2009)

Column I Column II
(A) E = 0 (p) + - Q Charges are at the comers of a regular hexagon. M is at the centre
of the hexagon. PQ is perpendicular to the hexagon.
- +
M
P+ -
(B) V ≠ 0 (q) P Charges are on a line perpendicular to PQ at equal intervals. M is
the mid-point between the two innermost charges.
- + - + - +
M

(C) B = 0 (r) Q Charges are placed on two coplanar insulation rings at equal
+ - + intervals. M is the common centre of the rings. PQ is perpendicular
to the plane of the rings.
- M -

P +
1 9 . 6 6 | Electric Potential and Capacitance

Column I Column II
(D) µ ≠ 0 P Charges are placed at the corners of a rectangle of the sides a and
(s) + - 2a and at the mid points of the longer sides. M is at the centre of
the rectangle. PQ is parallel to the longer sides.
M
+ + - -
Q

(t) Charges are placed on two coplanar, identical insulating rings at


- + -
equal intervals. M is the mid points between the centres of the
P Q rings. PQ is perpendicular to the line joining the centres and
M coplanar to the rings.
- + -

Q.22 A parallel plate air capacitor is connected to a Q.25 A dielectric slab of thickness d is inserted in a
battery. The quantities charge, voltage, electric field parallel plate capacitor whose negative plate is at x=0
and energy associated with this capacitor are given and positive plate is at x=3d. The slab is equidistant
by Q0 , V0 ,E0 and U0 respectively. A dielectric slab is from the plates. The capacitor is given some charge. As
now introduced to fill the space between the plates x goes from 0 to 3d (1998)
with the battery still in connection. The corresponding
(A) The magnitude of the electric field remains the same
quantities now given by Q, V,Eand U are related to the
previous one as (1985) (B) The direction of the electric field remains the same

(A) Q > Q0 (B) V > V0 (C) E > E0 (D) U > U0 (C) The electric potential increases continuously
(D) The electric potential increases at first, then
Q.23 A parallel plate capacitor is charged and the decreases and again increases
charging battery is then disconnected. If the plates
of the capacitor are moved farther apart by means of Q.26 An elliptical cavity is carved within a perfect
insulating handles  (1987) conductor. A positive charge q is placed at the center of
(A) The charge on the capacitor increases. the cavity. The points A and B are on the cavity surface
as shown in the figure. Then  (1999)
(B) The voltage across the plates increases.
(C) The capacitance increases.
A
(D) The electrostatic energy stored in the capacitor
increases. q B

Q.24 A parallel plate capacitor of plate area A and plate


separation d is charged to potential difference V and
then the battery is disconnected. A slab of dielecric (A) Electric field near A in the cavity = electric field near
constant K is then inserted between the plates of the B in the cavity.
capacitor so as to fill the space between the plates
(B) Charge density at A= potential at B.
If Q, E and W denote respectively, the magnitude of
charge on each plate, the electric field between the (C) Potential at A=potential at B.
plates (after the slab is inserted), and work done on the (D) Total electric field flux through the surface of the
system, in question, in the process of inserting the slab, cavity is q/ ε0 .
then (1991)
ε0 AV ε0KAV
(A) Q = (B) Q =
d d

V ε0 AV 2  1
(C) E = (D)
= W 1 − 
Kd 2d  K 
P hysi cs | 19.67

Q.27 For spherical symmetrical charge distribution, Q.30 Six point charges are kept at the vertices of a
variation of electric potential with distance from center regular hexagon of side L and centre O, as shown in the
is given in diagram. Given that:  (2006) 1 q
figure. Given that K = , which of the following
q q 4 π ε0 L2
V= for r ≤ R 0 and V = for r ≥ R 0
4 πε0R 0 4 πε0r statement(s) is (are) correct ? (2012)

v F
F E
+q q
d
P
E3
r
r = R0
A S T D
O -2q
Then which option(s) are correct: +2q
E2
(A) Total charge within 2 R 0 is q
R
(B) Total electrostatic energy for r ≤ R 0 is zero
C
(C) At r = R 0 electric field is discontinuous. B
+q -q
(D) There will be no charge anywhere expect at r= R 0 .
(A) The electric field at O is 6K along OD.
(B) The potential at O is zero
Q.28 Which of the following statement(s) is/are correct?
 (2011) (C) The potential at all points on the line PR is same
(A) If the electric field due to a point charge varies as (D) The potential at all points on the line ST is same
−2.5 −2
r instead of r , then the Gauss’s law will still be valid
Q.31 In the circuit shown in the figure, there are two
(B) The Gauss’s law can be used to calculate the field
parallel plate capacitors each of capacitance C. The
distribution around an electric dipole
switch S1 is pressed first to fully charge the capacitor
(C) If the electric field between two point charges is C1 and then released. The switch S2 is then pressed to
zero somewhere, then the sign of the two charges is charge the capacitor C2 . After some time, S2 is released
the same and then S3 is pressed. After some time, (2013)
(D) The work done by the external force in moving a S1 S2 S3
unit positive charge from point A at potential VA to
point B at potential VB is (VB − VA )
C1 C2
2V0 V0
Q.29 In the given circuit, a charge of +80 µC is given
to the upper plate of the µF capacitor. Then in the
steady state, the charge on the upper plate of the 3µF
capacitor is (2012) (A) The charge on the upper plate of C1 is 2 C V0
(B) The charge on the upper plate of C1 is C V0
+80C
4F (C) The charge on the upper plate C1 is 0
(D) The charge on the upper plate of C1 is - C V0

2F 3F Q.32 A parallel plate capacitor has a dielectric slab of


dielectric constant K between its plates that covers
1/3 of the area of its plates, as shown in the figure.
The total capacitance of the capacitor is C while that
of the portion with dielectric in between is C1 . When
(A) +32 µC (B) +40 µC
the capacitor is charged, the plate area covered by
(C) +48 µC (D) +80 µC the dielectric gets charge Q1 and the rest of the area
gets charge Q2 . The electric field in the dielectric is E1
1 9 . 6 8 | Electric Potential and Capacitance

and that in the other portion is E2. Choose the correct d/2
option/options, ignoring edge effects  (2014)

E1 E1 1
(A) = 1 (B) = Q1 E1
E2 E2 K S/2

E2 2
Q1 3 C 2+K Q2
(C) = (D) =
Q2 K C1 K
+ S/2 -

Q.33 A parallel plate capacitor having plates of area 1


S and plate separation d, has capacitance C1 in air.
When two dielectrics of different relative permittivities d
( ε1 2 and=
= ε2 4 ) are introduced between the two
plates as shown in the figure, the capacitance becomes (A) 6/5   (B) 5/3   (C) 7/5    (D) 7/3
C
C2 . The ratio 2 is  (2015)
C1

PlancEssential Questions
JEE Main/Boards JEE Advanced/Boards

Exercise 1 Exercise 1
Q.1 Q.3 Q.4 Q.3 Q.5 Q.6

Q.11 Q.13 Q.14 Q.7 Q.14 Q.15

Q.18 Q.20 Q.29 Q.16 Q.19 Q.20

Exercise 2 Exercise 2
Q. 2 Q.3 Q.4 Q.2 Q.6 Q.12

Q.5 Q.6 Q.11 Q.14 Q.15 Q.16


Q.17 Q.18 Q.22
Q.33 Q.34
Previous Years’ Questions
Q.5 Q.23 Q.30
Previous Years’ Questions
Q.31 Q.32 Q.21 Q.27
P hysi cs | 19.69

Answer Key

(b) 3 parallel rows; each consisting of five;


JEE Main/Boards
2.0 µF capacitors in series
Exercise 1
Q.22 420 µC one one, 180 µC on two,
Q.1 1.29 µF
60 µC on remaining 3 capacitors
Q.2 5x103 V
Q.23 4x102 F,102 F
Q.3 4 µF
Q.24 60 µC
Q.4 1.39 µF
Q.25 0.16µF,0.24µF
Q.5 2.67 x10−2 J
Q.26 Vmax < 9 kV
−7
Q.6 2.21x10 J
Q.27 (i) In parallel → new capacitance greater than the
Q.7 7.2F initial one (ii) In series → unchanged
C 24
Q.8 = Q.28 (i) 0.9 µC / s ; (ii) 1.09 × 10−6 J / s ;
C' 25
Q.9 C = 2C0 (iii) 2.73x10−6 J / s ; (iv) 3.82 × 10−6 J / s

Q.10 11.1% energy is increased Q.29 (i) (a) 10s ; (b) 2 µC ; (c) 6.94s

Q.11 6x103 V/ m, ∆ E =−12.15 x10−8 J (ii)


= q 1.348 × 10−8 C
∆E =30.4X10−8 J Q.30 1.8 ×1024 Cm
Q.12 18 µC
Q.31 2.7 × 106 V
φA C1 + φB C2 + φD C3
Q.13 φ0 = Q.32 0
C1 + C2 + C3
Q.33 1.11 × 10−9 C positive
E E
Q.14 (i) (ii)
R1 R1 + R 3 Q.34 1.8 × 106 V
Q.15 4.5g Q.35 Total charge is zero. Dipole moment =7.5 × 10‒8m
along z-axis.
Q.16 Both will travel the same distance
Q.36 (i) dV=4E, (ii) Vc > VA
Q.17 260volts, 6x10-4C charge flows from 3 µF to 2 µF
Q.37 (i) 0
Q.18 12 µC , 3 µC , 9 µC , 6 µC

Q.19 (i) 2.04V; (ii) 6.12 µC

Q.21 (a) Five 2 µC capacitors in series;

Exercise 2
Q.1 C Q.2 D Q.3 C Q.4 D Q.5 C Q.6 A

Q.7 B Q.8 B Q.9 B Q.10 C Q.11 A Q.12 C


1 9 . 7 0 | Electric Potential and Capacitance

Previous Years’ Questions


Q.1 A Q.2 D Q.3 D Q.4 A Q.5 A Q.6 C
Q.7 B Q.8 A Q.9 D Q.10 B Q.11 A Q.12 C
Q.13 B Q.14 A Q.15 B Q.16 C Q.17 B Q.18 D
Q.19 D Q.20 D Q.21 B Q.22 B Q.23 C Q.24 A
Q.25 C Q.26 C Q.27 A Q.28 D Q.29 A Q.30 C
Q.31 C Q.32 D Q.33 B C Q.34 A Q.35 B

JEE Advanced/Boards
V0 1
Q.15 (a) I = e−2t/Rc ; (b) CV 2
Exercise 1 R 4 0

Q.1 44.25pF 1 q2d


Q.16
Q.2 30V 2 ε0 A
Q.3 0
5  ε0 A 
32 Q.17 (i)  ;
Q.4 µF 3  d 
23

Q.5 10 µC 4  ε0 AV  2  ε0 AV 
(ii) Q3 =   ; Q5 =  

3 d   3  d 
8
Q.6 µF Q.18 69 mC
3
Aε0 V 100
Q.7 Q.19 (a) Volts
d 7

25ε0 A (b) 28.56mC, 42.84mC,


Q.8
24 d 71.4mC, 22.88mC

Q.20 150 µJ
2
Q.9 2k(q − x) Q.21 16 µC
35r
Q.10 0.8
3
Q.22
Q.11 9J 5
1  1
 E  Q.23 W =Aε0 V02  1 −  '
Q.12 C  R 2  K
 R + R  3
 1 3 
Q.24 12Volt
7
Q.13 (a) A Q.25 (i) 0.2 × 10−8 F,1.2 × 10−5 J;
50
(ii) 4.84 × 10−5 J
(b) Q1 = 0
9µC, Q2 =
(iii) 1.1 × 10−5 J

Q.14 (a) 0.05 1 − e− t/2  µC ;


 

(b) 0.125 µJ
P hysi cs | 19.71

Q.26 4.425 × 10‒5 Amp Q.31 1.8 × 105 sec



Q.27 12V 2 kP  
Q.32 ( − i − 2 j)
q(C) 2y 3
200 2C 4
 8 4  Kq2
Q.33 Wfirst step =
 −  , Wsecondstep =
0, Wtotal =
0
6 3 5 r
O t
Qq
C Q.34
2π ∈0 L
 V 
1/n  4a
C C0  0
Q.28=  1  0.01078µ=
−= F,n 20,No Q.35 (a) H = ,
 V  3
   a
(b)U=mg 2 h2 + a2 − h equilibrium at h=
  3
Q.29 =q
CV
2
(
1 − e− t/RC )
Q.30 Q A =
90µC,QB =
150µC,QC =
210µC,Ui
= 47.4mJ,U
= f 18mJ

Exercise 2
Single Correct Choice Type

Q.1 A Q.2 B Q.3 B Q.4 B Q.5 A Q.6 B


Q.7 B Q.8 A Q.9 A Q.10 D Q.11 B Q.12 C
Q.13 B Q.14 C Q.15 B Q.16 A Q.17 D Q.18 D
Q.19 B Q.20 C

Multiple Correct Choice Type

Q.21 A, B, C Q.22 A, D Q.23 A, D Q.24 A, B Q.25 B, C, D Q.26 A, B, D


Q.27 B, C Q.28 A, C, D Q.29 C, D Q.30 A, C, D Q.31 A, B, C, D Q.32 A, B, C, D
Q.33 B,D

Assertion Reasoning Type

Q.34 D Q.35 C Q.36 D

Comprehension Type

Paragraph 1: Q.37 B Q.38 B Paragraph 2: Q.39 A, C Q.40 A, B, C


Paragraph 3: Q.41 A, C Q.42 D
1 9 . 7 2 | Electric Potential and Capacitance

Previous Years’ Questions


Q.3 D Q.4 C Q.5 D Q.6 A, D Q.7 B, D Q.8 A, C, D
Q.9 B, C Q.10 A, B, C Q.11 A Q.12 A, C Q.13 A Q.14 A, D
Q.15 B, C Q.16 A, D Q.17 B Q.18 B Q.19 C Q.20 B
Q.21 A → p, r, s; B → r, s; C → p, q, t; D → r, s Q.22 A, D Q.23 B, D Q.24 A, C, D
Q.25 B, C Q.26 C, D Q.27 A, B, C, D Q.28 C, D Q.29 C Q.30 A, B, C
Q.31 B, D Q.32 A, D Q.33 D

Solutions

JEE Main/Boards Sol 4:


4F
11F
Exercise 1 C D
CF
A G 6F 12F B
Sol 1: Parallel ⇒ (4.5 + 4.5) µ F = 9 µ F = C'2 E F

1 1 1 1 8F
Total ⇒ = + + 2F
Ctotal 3 3 9
Capacitance b/w C and D
3F C2 ' 3F −1
1 1 
= 4µ F +  +  µ F
1 7  6 12 
= ⇒ Ctotal = 9 / 7 µ F = 1.29 µF
Ctotal 9
= 4µ F + 3µ F
= 7µ F
Sol 2: We have
Let capacitance b/w E and F
Q =CV ⇒ 5 × 10−6 =C × V = 2 + 2 = 4µ F
−6
5 × 10 Net capa. b/w G and B
⇒ V= = 5000 V
−9
10 −1 −1
1 1  1
D =  +  µF + 1 +  µ F
C4 C3 8 4  7

Sol 3: A C2 B 8 7 64 + 21 85
=  +  µF = µF= µF
C1C5 3 8 24 24
E
Now from symmetry, we can argue that the charge at 1 24
Now 1= +
C2 must be zero and hence potential diff. across D and E. C 85

1 1 1 1 1 85
So (capacitance across ADB)−1 = + = + = ⇒ Cc= µ F= 1.39 µ F
C 4 C3 4 4 2 61

⇒ Capacitance across ADB = 2µ F


Similarly, Capacitance across AEB = 2µ F
So total capacitance = 2 + 2 = 4µ F
P hysi cs | 19.73

Sol 5: Now Q = 4 × 10−6 × 200 = 800HQ 1 1


Sol 8: (A) (Capacitance total ) = +
C1 C2
4F d d
-Q Q ⇒ +
400V 2 × k1ε0 A 2 × k 2 ε0 A

d 1 1


Capacitance −1 = . + 
-(Q-q)
2F
Q-q 2Q/3 2ε0 A  k1 k 2 
−1

 ⇒ CA
=
2ε0 A  1 1 
. + 
d  k1 k 2 
-q q
Q/3 (B) Capacitance Total = C1 + C2
2F

Now, an applying Kirchhoff’s law, k1ε0 (A / 2) k 2 .ε0 .(A / 2)


= +
Q −q q d d
− 0
=
4 µF 2 µF ε0 A
⇒ C=
B (k1 + k 2 ).
Q = 3q ⇒ q = Q / 3 2d
−1
1 1 1 1
Einitial = × C × V 2 = × 4 ×10−6 × (200)2 = 8 × 10−2 2ε0 A / d.  + 
2 2
So,= CA / CB =
 k1 k 2 
4Q 2 Q2 (k1 + k 2 ). ε0 A / 2d
Q 21 / 2C1 + Q 22 / 2C2 =
Efinal = +
9 × 2C1 2C2 × 9
CA 4 ⋅ k1k 2 4 × 2 × 3 24
 2
2 1  = = = =
= Q  +  CB (k + k ) 2
(5)2 25
 9C1 18C2  1 2

 2 1  64 × 10−2 ε0 A 1 1
(8 × 10−4 )2 
= +  = Sol 9: CA = 1
and CB −= +
 9 × 4 × 10−6 18 × 2 × 10−6  2×6 d C1 C2

= 5.33 × 10−2 J d d d 2ε A
= + = ⇒ CB = 0
4ε0 A 4ε0 A 2ε0 A d
⇒ Energy lost = 8 − 5.33 = 2.67 ×10−2 J

So CA / CB =1/ 2

ε0 A 8.854 × 10 −12 × 100 × 10−4


Sol 6:=
C = Sol 10: We have Q = CV
d 2 ×10−3
1
Now energy = × CV 2
8.854 × 10−11 2
= = 4.427 ×10−11 F
2 ε A ε A
V = Same, but C1 = 0 and C2 = 0
1 d 0.9d
So, E = × C × V2
2 So energy change
1 4.427
= × 4.427 × 10−11 × (100)2 = ×10−7 = 0.221µ J
2 2 1 ε A 1 ε A 1 ε A  1 
= − × 0 × V2 + × 0 × V2 = × 0 × V2 .  − 1
2 d 2 0.9d 2 d  0.9 
1 2
Sol 7: = CV = 100 × 103 × 3600 = Einitial [0.111]
2

200 × 103 × 3600 ⇒ 11.1 % energy is increased.


⇒ C= = 7.2 F
(104)2
ε0 A 9 × 10−12 × 100 × 10−4
Sol 11:=C =
d 2 × 10 −2
1 9 . 7 4 | Electric Potential and Capacitance

= 4.5 × 10‒12 F Sol 13: A


1 1
Change in energy = × C1 × V 2 − × C2 × V 2 q1 C1
2 2
1 2 O
= Einitial − Efinal = × V . C1 − C2  C2 q1 v q3C
2 0 3

B D
1 1 1
= × (300)2 . ε0 A  − 
2  d1 d2  We have q1 + q2 + q3 =
0

9 × 10 4 × 9 × 10−12 × 100 × 10−4  1 1 ⇒ C1 .(φ0 − φA ) + C2 .(φ0 − φB ) + C3 .(φ0 − φD )


= × − 
2 × 10−2 2 5
⇒ (C1 + C2 + C3 )φ0 = C1 φA + C2 φB + C3 φD
= 40.5 ×10−8 3 / 10 
C φ + C2 φB + C3 φD
−7 ⇒ φ0 =1 A
= 1.215 ×10 J C1 + C2 + C3
Q CV ε0 A 300
E
= = = ⋅V = = 60 × 102
ε0 A ε0 A d2 .ε0 A 5 × 10 −2 Sol 14: (i) Now, at just t = 0, charge at C1 = 0 and charge
at C2 = 0 so = V Q= / C 0 so potential diff. between
= 6 × 103 V / m AB = 0
Now in 2nd case , Q = cons tant i = E / R1 + R AB = E / R1
We have Q = CV (ii) After long time , no current through both capacitors
−12 −10
= 4.5 × 10 × 300 = 13.5 × 10 ⋅C E
E iR1 + iR3 ⇒ i =
⇒=
= Q = 1.35 × 10−9 ⋅ C R1 + R3

Now change in energy: Einitial − Efinal Q


Sol 15: E due to one plate =
2ε0 A
 Q 2 Q 2  Q 2  d1 d 
⇒  − =  . − 2 Q Q2
 2C 2C  2  ε A ε A  So force = Q. × =
 1 1  0 0  2ε0 A 2ε0 A
Q2
Q2 Q 2 .(d1 − d2 ) So=
T mg
=
= [d1 − d2 ] = 2ε0 A
2ε0 A 2ε0 A
C2 V 2
mg =
(1.35 × 10−9 )2 .(2 − 5) × 10−2 (1.35 × 10−9 )2 × ( −3) 2ε0 A
= =
2 × 9 × 10−12 × 10−2 2 × 9 × 10−12
C2 V 2 ε A.V 2 9 × 10−12 × 10−12 × (5 × 108 )2
m= = 0 =
−(1.35 × 10−9 )2 2ε0 A.g 2gd2 2 × 10 × (5 × 10−3 )2
= = −0.3 × 10−6 = −3 × 10−7 Joules
2 × 3 × 10−12
= 4.5 × 10−3 kg = 4.5 gm
12V 2
Sol 16: Let there, masses be me and mi
Sol 12: 1 1
4 Then Vd = me v e2 = × Mi × Vi2  …. (i)
2 2
6 Ee Ee
Now acceleratio of particles ⇒ = ae and = = ai
me Mi
Now no current will pass through the capacitor.
So, i = 12 / 8 = 3 / 2 A 1
So time to hit the plate ⇒ =
S ut + at2
2
Thus, potential diff. across capacitor
2d 2d.me
9V ⇒ Q = CV = 18µC.
= 6×3 / 2= ⇒ = t=
ae e Ee
P hysi cs | 19.75

2d 2d.Mi q1 q2 3
Similarly,=
ti = .. ⇒ = ⇒ q1 = q
ai Ee 3 2 2 2

, q1 780 µ C
q2 520 µ C =
=
Now distance = Ve × te
520 µ C
2d.me Potential difference = = 260 Volts
2Vd 2 µF
= ×
me Ee Thus the charge will flow from each positive terminal to
the negative terminal.
V
= 2d. (independent of mass)
Ee
Sol 18: No current will pass through, the upper portion,
Same distance can be found for the ion and hence the and voltage difference between the capacitors would
time is same. be same as ε .
−1 −1
 1 1   1 1
Sol 17: Q =
3 × 300 =
900 µ C. C total 
=
C +C
+  =  +  = 5µ F
 1 2 C3 + C 4   10 10 
And Q 2 =
2 × 200 =400 µ C.
(4)
Actual emf = ε − × 1 = ε − 1 = 3V
Q1 Q1-q (3 + 1)
+ -
So Q = CV = 5 µ F × 9 = 15µ C
q1 q3
Q2 Q2+q

Q1 − q (Q 2 + q)
So using Kirchhoff’s law, ⇒ − 0
=
3 2
q2 q1
2Q − 3Q 2
⇒ 2Q1 − 2q = 3Q 2 + 3q ⇒ q = 1
5

q1 Now, q1 + q2 = 15µ C  …(ii)


C1 q1 q2
and − 0
=
C1 C2
C1 q1 q2
⇒ = ⇒ q1 = 4q2  …. (i)
8 2

q1 + q2 = 900 − 400 = 500 µ C ⇒ q1 =


12 µ C, q2 =
3µ C

(from charge conservation) Also (q3 + q4 ) =


15
q1 q2 q1 3 q3 q4
Also − 0 ⇒ 2q1 = 3q2 ⇒
= = − 0
=
3 2 q2 2 c3 c4
⇒ q1 =
300µ C and q2 =
200 µ C
q3 q4 q3 3
= ⇒ =
q1 + q2 = 1300 µ C 6 4 q4 2

q1 ⇒ q3= 9µ C ⇒ q4= 6µ C

q1

q2
1 9 . 7 6 | Electric Potential and Capacitance

Sol 19: Diagram-1 Sol 20: (i)


2F C1 C1
4F q4 -q4 e c
q2 q2 a
5F q5 -q5
D
q1 -q1 C2 C2 C1
8F 8F
A G B
6F b
q6 -q6 C1 f C1 d
q3 -q3


3F q66F -q6

Diagram-2 C1
e q’’ c
a


2F q’’
q4 C2 CT ’
4F
2F
q4
5F 4F q5 b
-q’’C q’’ d
f
A 4F B
1
5F q5
4F


A B
3F 12F
e
a


3F 12F
Diagram-3 q’’
2.4F C2 CT’’

A 2.4F B b
f
5F


A 2.4F B
5F
−1
2.4CF =  1 + 1 
total  
 5 4.8  a
−1 −1
1 1  1 5  120
Ctotal
=  +  = 2.45µ F
 5 4.8   +  = = CT’’’
 5 24  49
−1
1 5  120 b
=
So,
 =Q+ CV
= = =
2.45 × 102.45µF µ C
= 24.5
 5 24  49
Now from diagram (3), capacitance
We have
= same ⇒ charge is equally divided. −1 −1
 1 1 1 2 1
24.5 CT =
 + +  = (6 / 7)µ C
 +  =
⇒ q=
2 q=
3 = 12.25 µ C 3 3 2 3 2
2
Now from diagram (2) 6 20
Now CT ' = + 2= µC
q4 + q5 =
q2 7 7
q4 q5  7 2
−1
 61   60 
−1
And − 0 ⇒ 2q4 − q5
= Now CT " = +  =  =  µC
2 4
 20 3   60   61 
12.25 × 2
So q4 = 12.25/3µC = 4.08µC and=
q5 = 8.166µ C
3 60 182
⇒ CT '"= + 2= µC
4.08µ C 61 61
(i) So voltage diff. =
= q / c = 2.04 V
2µ F
−1 −1
 61 1 1   61 2 
12.25µ C 6.12µC CTotal = + +  = 1 C
+  =µ
(ii) Charge = =  182 3 3   182 3 
2
[q3 =
12.25µ Cand2q6 =
q3 ]
P hysi cs | 19.77

(ii) Q = CV = 1× 900
= 900µC 7 7
VAC= × VCB ⇒ VAC ⇒ .VCB
3 3
(iii)
600 3
600V ⇒ VAC= 7 × = 420 V and V=
CB 600 × = 180 V
C 500V 10 10
900
1 C × 420 = 420 µ C
⇒ Q AC =CV =µ
600 C 2 300 C
And QBC= 180 × 1= 180 µ C
0 d 400V
300V And in each 3 , q will be equally distributed
q= 420 − 2 × 180 ( on each)
Q 900µC
=
1 1
900 Sol 23: We have × C1 × (2)2 + × C2 .(2)2 = 10 × 102
v
⇒ ∆= = 300V 2 2
3
300 ⇒ C1 + C2 =
500
Now between e and c = = 100V
3
Q2 Q2 1 1
So, between c and=
d 500 − 400V = 100V And 160
= + 320 Q 2  +
⇒= 
2C1 2C2 C
 1 C 2

Sol 21: (a) a b 1 1 V2
= V 2 × Ceq2  + =
All in series  C1 C2   1 1 
 + 
(b) 2  C1 C2 
V ⋅ C1 ⋅ C2
320 =
C1 + C2
a b C1 ⋅ C2
⇒ 80
= ⇒ C1 ⋅ C
= 2 40000  …(i)
500
Cnet =1.2F ⇒ On solving, 400 , C2 = 100
C1 =

Sol 24:
q2 -q2
q1 -q1

Cnet
= 1.2 µ F 2F 3F

Sol 22: By trial and error q3 -q3


110C -110C

1F

We have q3 + q1 = 110µC
A 1 C B
and q2 − q1 =
0
q3 q1 q2 q1 q2
 q1 and
⇒ q2 = − − 0 ⇒ q3 =
= +
A 1 C Ctot B 1 2 3 2 3
5 ⋅ q1 6 × 110
⇒ 110 − q1= ⇒ q1= = 60 ⇒ q1 = 60µC
A 1 C Ctot B 6 11

VAC + VCB =
600 ….. (i) So, q=
2 60µC and q5= 50µC

So, 110 − 50= 60µC charge passes through wire.
And 1× VAC = Ctot × VCB
1 9 . 7 8 | Electric Potential and Capacitance

40V (unchanged)
Sol 25: 100V 0
CA CB Ceq (parallel)= C1 + C2

100V CA 90 0  1 1  ε0 A.(2d)
= ε0 A.  + = 2
CB  d + a d − a  (d − a2 )

2µC  1 1  2ε A
Without any a difference , Ceq =
ε0 A ×  +  = 0
We have CA × 60 = CB × 40 ( q charge is same) d d d

Now 1 − a2 / d2 < 1 so the capacitance increases in case


CA 2 of parallel.
⇒ =
CB 3

Similarly Sol 28:


1F
CA + 2 9×3 25
⇒ = 9 ⇒ CA + 2= × CA = 2 = C q -q
CB 2 2 A
6
i 3 x 10 
4
= CA = CA 0.16µ F
⇒ =
25

CA 2
Similarly , = CB 0.24 µ F
⇒= 4V
CB 3
(i) We have , iR + q / c =
4
Vmax V 0
Sol 26: dq q 4
⇒ iCR + q =4C ⇒ + =
C1 C2 dt RC R
For Vmax .(inseries) , Q = same for both the capacitance (in q,t t
4 t/RC
∫ d q.e
t/RC 
R ∫0
series ) ⇒ = e .dt

0,0
So , C1 (Vmax − V)= C2 (V − 0)
4
q.e
= t/RC
=.(RC) et/RC − 1
R  
⇒ 1× (Vmax − V) = 2.(V − 0)
Vmax
⇒ Vmax − V =
2V ⇒ V = =
⇒ q 4C 1 − e− t/RC 
3  
Now V < 4kV dq  1  − t/RC
Now = 4C.   .e
Vmax dt  RC 
⇒ < 4 ⇒ Vmax < 12 kV
3
4
= × e−1/3 = 0.96 µ C / s
And Vmax − V < 6kV 3 × 10 6

2Vmax Q2
⇒ < 6kV ⇒ Vmax < 9 kV (ii) E= x ×
3 2c
Lower limit is 9 kV dE 1 dq q dq
⇒ = × 2 × q. = .
C1.C2 dt 2C dt C dt
Sol 27: Ceq (Series) =
(C1 + C2 ) 4C 1 − e−1/3 
=   . 4 × e−1/3 = 1.09 × 10−6 J / s
 1
−1
d+a d−a
−1
 2d 
−1
ε0 A C 3 × 106
1 
=  +  =

+  =   =
 C1 C2   ε0 A ε0 A   ε0 A  2d
 dq 
2
2
(iii) P = i R =   .R
Which is same as without any difference a.  dt 
= (0.96 µ C)2 .3 × 106
P hysi cs | 19.79

= (0.96)2 × 3 µ J = 2.73 µ J / s Sol 31: 5C

(iv) Energy = Energy in Resistance + Energy in capacitance 5C 5C


−6
= (2.73 + 1.09) × 10 J/s
= 3.82 × 10−6 J / s 10cm
cm
10 5C
Sol 29: 5C

5C
kq
Potential at center due to one charge =
r
9 × 109 × 5 × 10−6
2V = = 45 × 104 V
−1
10
(i)
Potential at center due to all the charges
(a) t = RC = 107 × 10–6 = 10 sec
(b) Q = CV = 2 × 1 µC = 2µC = 6 × Vq = 2.7 × 106 V

(c) Now , as in Ques. 29 above, q= 2 ×µ C [1 − e− t/10 ]


1 Sol 32:
⇒ = 1 − e− t/10
2 D C

⇒ et/10 = 2
10C
⇒ t = 10ln2 = 6.94 sec

(ii) A B
2C

We have q = q0 e− t/RC (k)(10µC) 2k


VA = = (10 µC)
a a
Now q= 2 × e−50/10 = 2.e−5 = 0.0135 µ C 2

(k)(10µC) 2k
Sol 30: VB = = (10 µC)
 a  a
 
-1

 2
NC

Work done is moving charge of 2µC = q(VB - VA) = 2mC


4
10

o
30
(0) = 0
x
30

+ 2
E=

Sol 33:
T = (Eq) (2  ) sin 30° q
T = E sin 30°. (q.2l) r = 0.1m
⇒ Dipole moment (D) v = 100 volts
26
T 27 × 10 kq 100 × 0.1
= q.2 l = = ⇒ = 100 ⇒ q =
Esin30° 30 × 10 4 × 1/ 2 r k
= 1.8 × 1024 cm = 1.1 × 10-9 C (positive)
1 9 . 8 0 | Electric Potential and Capacitance

Sol 34: ∴ Net force = Eq – Eq = 0 (Assuming length of dipole


is small)
4cm 
(ii) Ui = - P.E = -PE
P 
20c
6cm -10c Uf = P.E = PE
3cm 1cm Uf – Ui = 2PE = work done is rotating dipole

7cm
Exercise 2
q1 q2 q q 
Vp = K +k = k  1 + 2  Single Correct Choice Type
r1 r2  r1 r2 
q1 = −10µc , r1 = 1 cm Sol 1: (C)
q2 = 20µc , r2 = 7 cm CV + Q
-CV
Put the values in the equation for Vp, d
Vp = – 1.8 × 106 V

Sol 35:
-7 C
0
x1
5)

.5
,1

-2  σ1 CV   CV + Q CV  Q CV
,0

 +  = +  = +
(0

 02ε 2Aε0   2ε 0 A 2Aε0  2Aε 0 Aε0


-7 C
0 Q d d
x1
5)

.5 work done = E × d = + CV ×
,1

-2 (per unit charge) 2 Aε0 Aε0


,0
(0

Q 1 Q
Total charge of system = + C × V'× = V +
2C C 2C
= -2.5 × 10-7 C + 2.5 × 10-7 C = 0
−1
Dipole moment of system 1 1 1 
Sol 2: (D) Ceq =C  + +  =C + C / 2 =3C / 2
= -2.5 × 10 × (-15 – 15) × 10
-7 -2  C 2C 2C 
Q =×
60 3C / 2 =
90C
=-7.5 × 10-8 Cm (along –ve z-axis)
q1 q2
Now = = ; q1 2q2 and =
q1 + q2 90C
Sol 36: The motion perpendicular to electric field won’t C C/2
account in work
⇒ q2 =
30C. So potential difference
∴ Work done in moving charge from A to B is = Q=
/ C 30V
E(6 – 2) = 4E
∴ Potential difference between A and B = 4E also Sol 3: (C)
q3
VB = VC (since no parallel movement along E)
q2
⇒ VC – VA = 4E and VC > VA
q4
Sol 37: (i) q1
+ Eq
4F
6 Q
Eq -
P hysi cs | 19.81

1 1
−1
 10 
−1 q1 q2
Ceq =4 +  +  =4 +   =6.1µC. = ⇒ q1 = 2q2
4 2
3 7  21 
144µF
⇒ q2 = = 48µC
So qtot = q1 + q2 = (6 / 6.1)µC 3

Out of this again, this is divided in 5µF and 1µV


3F 1
-q
1
q q1 q2
= ⇒ q1 = 5q2
2F 5 1
5F 48
⇒ q6 = =µ8 C
-q q 6
Q2 8 × 8
so energy= = 32µJ
2 2
-q q =
4F 2C 2 × 1
q1 q2 q1 4
= ⇒ =
4 2.1 q2 2.1 Sol 6: (A) Charge on right should be positive and
q1 q2 q1 3
4 6 2.1 6 q1 + q2 = 20µC and = ⇒ =
q1 = × µC and q2 = × µC 3 4 q2 4
6.1 6.1 6.1 6.1
3 60
and also q3 + q4 =
q2 ⇒ q1 = × 20 = =8.57µC
7 7
q3 q4
and =
2 5 Sol 7: (B)
5 5 2.1 6
⇒ q4 = × q2 = × × µC
7 7 6.1 6.1
5 2.1 6
q4 7 × 6.1 × 6.1 3 A  A B

= =
q1 4×6 8 B C


6.1× 6.1 C

3C/2
Sol 4: (D) 4 times in series. Let each be x, then A B

x
= 16µF ⇒ x = 4µF
4 C
−1
Which is possible when 8 are connected in parallel  20 1  3C 8C
So eq ⇒  +  +C= +C=
⇒ 8 × 4 = 32  3C C  5 5

Sol 5: (C) −1
 1 1 1  5 1
Sol 8: (B) Ceq =  + +  − 1=  + 
6F  2 5 3  6 5
3F 6F −1
 31  30 30
 =  = = µF q C=
eq .Vmax .V ;
31 max
4F  30  31

24V
q 30
−1 V=
1 = .V < 3V ⇒ Vmax < 6.2 V
1 1 C 31× 2 max
 +  =2 and 2 + 4 =6µF
3 6
q 30 1
V2 == × .V < 2V ⇒ Vmax < 6.2 V
So charge = Q = 6µF × 24 = 144µC C 31 3 max

That much charge is divided in 4µF and 2µF (above eq. 30 1


V2 =× .Vmax < 1 Vmax < 31/ 6
capacitance) 31 3
1 9 . 8 2 | Electric Potential and Capacitance

Sol 9: (B)
σ
=105 V / m ⇒
σ
=0.5 × 105 V / m Previous Years’ Questions
ε0 2ε0

σ C1C2
Now, force= × Q= 0.5 × 105 × 1µC= 0.05 N Sol 1: (A) In series, C =
2ε0 C1 + C2

σ Q2
Sol 10: (C) Force = × Q=
2ε0 2Aε0

so Force ∝ Q 2
1F
Vmax 3V 2V 1V
O
2 3 5 10F
(1) (2) (3) (10)(1) 10
Cnet = = µF
Now, when d is halved ⇒ C is doubled; Q=CV ⇒ Q is 10 + 1 11
doubled.
So Force ∝ Q 2 ⇒ F becomes four times. Sol 2: (D) Since, the capacitor plates are directly
connected to the battery, it will take no time in charging

d/2 d/2  k + k2  C
Sol 11: (A) deff = + = d  1 
k1 k2  2k1k 2 

d 2k1k 2 R
∴ k eff = =
deff k1 + k 2
V
A 2 ∈0 A(k1k 2 )
Also, C =
∈0 =
deff k1 + k 2
Sol 3: (D) When S3 is closed, due to attraction with
opposite charge, no flow of charge takes place through
Sol 12: (C)
S3. Therefore, potential difference across capacitor
plates remains unchanged or V1 = 30V and V2 = 20V

Alternate solution
Charges on the capacitors are
q1 = (30)(2) = 60pC and
Capacitance increases = kC0
q2 = (20)(3) = 60pC or q1 = q2 = q(say)
Now,
The situation is similar as the two capacitors in series
qi =Ci V =60µC and qf =Cf V =180µC(60 + 120) are first charged with a battery of emf 50 V and then
⇒ Cf = kCi ⇒ k = Ci . V = 180µ C ⇒ k = 3 disconnected.

q q
+ - + -
2pF 2pF q=60pC q=60pC
+ - + -
V1=30V V1=20V

50V
∴ When S3 is closed, V1 = 30V and V2 = 20 V.
P hysi cs | 19.83

Sol 4: (A) Due to attraction with positive charge, the Dipole moment
negative charge on capacitor A will not flow through
the switch S. 4q
a/2
Sol 5: (A) After time t, thickness of liquid will
-4q
d 
remain  – vt 
3  a
P = 4q   ˆj = 2qa ˆj
Now, time constant as function of time: 2

ε0 (1)R kq VR
tc = CR = Sol 9: (D) Potential of each drop = V = ⇒q=
 d  d / 3 – vt R k
 d – – vt  + VR
 3  2 If n drops coalse, Q = nq = n
k

  Radius R’ = n1/3 R
 ε0 A 
 Applying C =  nkVR
 t  The potential of new drop = = Vn2/3
 d–t+  kn1/3R
 k 
6ε R Sol 10: (B) The potential at center is same as that of
=0
5d + 3Vt the surface.
∴ V = 10 Volts
Sol 6: (C) Option C is correct because electron gets
attracted towards positive charge (moves against Sol 11: (A)
electric field) larger the potential, lower the potential
energy for electron.

U = V(-q) b
10V

5V
a
Sol 7: (B) σ = 10-7 C/m2
Given, V = 5 volts

σ At center the potential will be same as the potential of


but V = .r the sphere enclosing it first
ε0
If it is any other potential, there will exist a electric field
ε0 (V) 8.854 × 10 −12
× 10 inside sphere contradicting gauss law.
⇒r= =
−7
σ 10
= 8.8 × 10-4 m = 0.88 mm Sol 12: (C)

Sol 8: (A) -
3q b
+
a
-2q -2q c -

(Center of charge) of negative charges is origin Potential at a due to charge of a = - (σ / ∈0 ) (a)

(center of charge) of positive charges is Potential at b due to charge of b = (σ / ∈0 ) (b)

3q(a) − q(a) q Potential at C due to charge of c = −(σ / ∈0 ) (c)


= σ
4q 2 ∴ Potential at a = (b – a – c)
∈0
1 9 . 8 4 | Electric Potential and Capacitance

Sol 13: (B) Electric potential at any point inside a hollow →


The electrical field E at all points on the x-axis will not
metallic sphere is constant. Therefore, if potential at
have the same direction.
surface is 10 V, potential at centre will also be 10 V.
For – d ≤ x ≤ d, electric field is along positive x-axis
while for all other points it is along negative x-axis. The
Sol 14: (A) In such situation potential difference →
depends only on the charge on inner sphere. Since, electric field E at all points on the y-axis will be parallel
charge on inner sphere is unchanged. Therefore, to the x-axis (i.e., î )
potential difference V will remain unchanged. The electrical potential at the origin due to both the
charges is zero, hence no work is done in bringing a
test charge from infinity of the origin.
1 Q1 1 Q2
Sol 15: (B) VC = VQ = VQ = +
1 1 2 4πε0 R 4πε0 R 2 Dipole moment is directed from the –q charge to the
+q charge (i.e.,– î direction)
1  Q2 
=  Q1 + 
4 πε0R  2 Sol 17: (B) The diagrammatic representation of given
problem is shown in figure.
1  Q1 
Similarly VC =  Q 2 +  The net charge shared between the two capacitors is
2 4 πε0R  2
Q’ = Q2 – Q1 = 4CV – CV = 3CV
Q1 Q2
+ - Q1=C1V1=CV
R 2 + -
R + -
+ -
+ -
C1 R C1

+ -
+ -
+ -
∴ DV = VC – VC + - Q2=C2V2=(2C)(2V)=4CV
1 2 + -
1  1 
= (Q1 – Q 2 ) – (Q1 – Q 2 ) The two capacitors will have the same potential, say V’
4 πε0R  2 
The net capacitance of the parallel combination of the
Q1 – Q 2 two capacitors will be
= ( 2 –1)
2(4 πε0R) C’ = C1 + C2 = C + 2C = 3C
The potential difference across the capacitors will be
W = qDV = q(Q1 – Q2)( 2 – 1)/ 2 (4pe0R)
Q' 3CV
V’ = = =V
C' 3C
Sol 16: (C) The diagrammatic representation of the
given question is shown in figure. The electrostatic energy of the capacitors will be
 1 1 3
Eq U’ = C’V’2 = (3C)V2 = CV2
2 2 2

E
 Sol 18: (D) Potential at origin will be given by
y
E-q
q 1 1 1 1 
V=  – + – + ....... 
4πε0  x0 2x0 3x0 4x0 
x
q 1  1 1 1  q
q -q = . 1– + – +.......  = ln(2)
4πε0 x0  2 3 4  4 πε 0 x0
(-d, 0) (d, 0)
P hysi cs | 19.85

→ → → Sol 21: (B) Potential decreases in the direction of


Sol 19: (D) Electric field within the plates=
E E Q1 + E Q2
electric field. Dotted lines are equipotential lines.
Q1 Q2 Q1 – Q 2
E = E1 – E2 = – ,E= y
2Aε0 2Aε0 2Aε0

∴ Potential difference between the plates



E C

x
A B
+Q1 +Q2
E2 E1 E

∴ VA = VC and VA > VB
1 2
 Q – Q2  Q1 – Q 2 Q1 – Q 2 q Q
VA – VB = Ed =  1 Sol 22: (B) q
 2Aε  d =  Aε  = 2C
 0 
2  0  x=–a x=0 x=+a
 d 
Initial Position

ε0 A
Sol 20: (D) Applying C= , we have q Q q
t t
d – t1 – t2 + 1 + 2 x=–a
K1 K 2 x=x x=a
ε0 (A / 2) Final position
d/2 d/2
d–d/2–d/2+ + 2KQq K.q.q
K1 K3 Ui = +
q 2a
ε0 (A / 2) Kε0 A
+ =  1 1  K.q.q
d/2 d/2 d and Uf = KQq 
d–d/2–d/2+ + +  +
K2 K3  a + x a – x  2a

Solving this equation, we get 1


Here, K =
4πε0
2KQqx2
A/2 A/2 A DU = Uf – Ut or |DU| =
d/2 d/2 a3
K1 K2
+  K d for x << a ∴ DU ∝ x2
K3 d/2 K3 d/2
Sol 23: (C) DU = decrease in potential energy
= Ui – Uf
In parallel 2
1 1  V + V2 
= C( V12 + V22 ) – (2C)  1 
K1K 3 K 2K 3 2 2  2 
K= +
K1 + K 3 K 2 + K3 1
= C(V1 – V2)2
4

Sol 24: (A) There will be an electric field between two


cylinders (using Gauss theorem). This electric field will
produce a potential difference,
1 9 . 8 6 | Electric Potential and Capacitance

Sol 25: (C) A = (–a, 0, 0), B = (0, a, 0) Sol 29: (A) 120C1 = 200C2
y
6C1 = 10C2
3C1 = 5C2
B -Q

Sol 30: (C)


x
A O
Q
z
x dx
Point charges is moved from A to B
VA = VB = 0 ∴ W = 0 x =2L k Q Q ln 2
=V ∫x=
=L x  L 
 
dx
4 π ε0 L
3
Sol 26: (C) Distance BC= AB sin 60°=(2R) = 3R
2
Sol 31: (C) By formula of electric field between the
1 (q / 3)(2q / 3) q 2 σ
∴ |FeBC|= = plates of a capacitor E =
4πε0 ( 3R) 2
54 πε0R 2 K ε0

⇒ σ = E K ε0 = 3 × 10 4 × 2.2 × 8.85 × 10 −12


Sol 27: (A) After time t, thickeness of liquid will =6.6 × 8.85 ×10−8
d  = 5.841× 10−7
remain  – vt 
 3 
≅ 6 × 10−7 C / m2
Now, time constant as function of time:

Sol 32: (A) E = 30 x2 ˆi
ε0 (1)R
tc = CR = dV = − ∫ E.dx
 d  d / 3 – vt
 d – – vt  +
 3  2 VA 2

∫ dV = − ∫ 30 x2 dx
  V0 0
 ε0 A  6ε0R
 ApplyingC =  = VA − V0 =
−80 volt
 t  5d + 3Vt
 d–t+ 
 k 
Sol 33: (B, C) The potential at the centre
Sol 28: (D) q1 = C1V = 2V = q 2
R 4 π r dr
Q 3 kQ 3 1
4 3 ∫0
= k = = =V ;k
This charge will remain constant after switch is shifted r 2 R 2 0 4 π ε0
πR
from position 1 to position 2. 3
1 q2 q2 q2
Ui = = = ∴ R1 = 0
2 Ci 2×2 4
kQ
1 q2 q2 q2 Potential at surface, V0 =
Uf = = = R
2 Cf 2 × 10 20
5V R
q2 Potential at R 2 = 0
⇒ R2 =
∴ Energy dissipated = Ui – Uf = 4 2
5
k Q 3 kQ 4R
 q2  Potential at R3 , = ⇒ R=
3
This energy dissipated  =  is 80% of the initial R3 4 R 3
 5 
 q 2  
stored energy  =  k Q kQ
 4  Similarly at R 4 , = ⇒ R 4 = 4R
  R4 4R
P hysi cs | 19.87

Sol 34: (A) Let the potential at P be V, Now q on=


C2 8C
= 2 24 C2
Then, C (E – V) = 1× V + 2 × V (we take C in µ F ) So total charge = (24 + 6)C1 = 30C1

V Same charge would be there at capacitor b/w a and b


1F 30 C1
So , ∆=
V = 30V
C P C1
2F
(50µ C )
2
(20 µ C)2
Sol 3: Einitial = +
2 × 2µ F 2 × 5µ F
E
CE 2500
Or, V = = 100 µJ + µJ = 350 µJ
3+C 10
2CE q1 q2 2
∴ Q2 = Now = ⇒ q1 = × 70 µ C = 20 µ C
3+C 2 5 7
q1
Sol 35: (B) Charge on 9 µF capacitor = 18 µC
Charge on 4 µF capacitor = 24 µC 2F
∴Q = 24 + 18 = 42 µ C

KQ 9 × 109 × 42 × 10−6 5F


=
∴ = 420 N / C q2
r2 (30 )
2

And thus , q=
2 50 µ C

Einitial = same (thus there would be no heat produced)


JEE Advanced/Boards
Sol 4:
Exercise 1 1F
C
A
6 mm 4 mm 8F
Sol 1: deff = + = 2 mm 4F 4F
6 4 4F
A
∴C =∈0 1F
deff

8.85 × 10−12 × 100 × 10−4


 8/3F 8F
= 44.25 pF
2 × 10−3
C
A
Sol 2: 8/3F 8/9F
+ - B
C
b A
a C2
C1 32/9F
q B
C1 C2
1 9
Now qpq =×
2 C2 =
6C1 Now, 1
= +
C 32
So voltage difference across b and p
32 − 9 1 32
= 6C1 / C1 = 6V ⇒ = C
⇒= µF
32 C 23
1 9 . 8 8 | Electric Potential and Capacitance

Sol 5: ∆ V =
10 V  4ε A  ε0 AV
4Q = (4C)V =  0  V ⇒ Q =
So, Q = CV = 10 × 1 µ C  d  d
(The ends of C = 1µ F are connected to the terminals of ε0 AV
the battery) So, the charge on plate 1 is and that on plate 4
d
2ε AV
is − 0
d
Sol 6: Suppose a charge q is given to the system.
q1 -q1 q4 -q4 C1.C2
Sol 8: Capacitance (left side):
C1 + C2
-q2 1
x 1 1 2 y 1× (A / 2).ε0 3 × (A / 2).ε0
q2 . 3Aε0 3Aε0
2 = d d = =
(A / 2).ε0 8d 8d
q3 -q3 4.
d
Then, q1 + q2 + q3 =q …… (i)
 Capacitance: (right side)
And q4 + q3 = q  ….. (ii)
2 × (A / 2).ε0 4 × (A / 2).ε0
Also , applying kirchoffs law in loop (i) .
d d 4 Aε0 2Aε0
−q1 q2 = = =
+ 0 ⇒ q1 = q2 …(iii)
= (A / 2).ε0 6 d 3d
6.
1 1 d
q3 q2 q4
And − − 0
= 3Aε0 2Aε0
2 2 1 So C=
total +
8d 3d
q3 2(q2 + q4 )
⇒=
 9 + 16  Aε0 25 Aε0
Now, 2q1 + q3 =
q (form (i)) =Ctotal  = 
 24  d 24 d
(q − q3 )
⇒ q=
2 q=
1
2
Sol 9:
 (q − q3 ) 
⇒ q3 2. 
= + q − q3 
 2  II
⇒ 4q3 =q − q3 + 2q − 2q3
I q2
⇒ q3 = 3q / 4 -x
p
1 3q
Now, ∆=
V 3q / 4 × = V
2 8 x
8 q-x
So Capacitance = q / ∆ V = µF -q+x
3

Sol 7: The Circuit can be converted to all branches have Now V at outer must = 0
the same capacitance and are connected in parallel.
k.( − x) kq k[( −q + x) + q2 ]
1 2 ⇒ + + 0
=
Q Q 3.5r 3.5r 3.5r

⇒ kq2 = 0 ⇒ q2 = 0
3 2
(A,1,3,5) (2,4,B) And now potential at inner cell = 0
Q Q
k.( −q + x) k(q − x) k.x kx
3 4 + + − 0
=
3.5r 2.5r 2.5r r
Q Q
5 4 k( −q + x) kq kx
⇒ + − 0
=
Q Q 3.5r 2.5r r
P hysi cs | 19.89

(x − q) q 20
1
+ −x =0 So Energy =∫ ε0 .E2 .(4 πr 2 ).dr
3.5 2.5 2
10

2(x − q) 2q
+ − x=0 ε0 dr
7 5 = . 4 π.k 2q2 20
10 ∫ r2
2
⇒ 10(x − q) + 14 q − 35 x =
0
⇒ 4q = 25x kq2  1 1  5kq2
= . −  =
2  0.1 0.2  2
⇒ x = 4q / 25
kx 5 × 9 × 109 × (20)2 × 10−12
So in region (I), E = =
r2 2
2.5r =9J
1
Energy = ∫ ε .E2 .(4 πr 2 ).dr
2 0 E
r Sol 12: i =
R1 + R3
2.5
1 2 1 2
= 2 ×ε0 × 4 π.kx ∫ r2 r .dr
So, ∆ v= R 3 .i=
R 3E
r
R1 + R 3
2
kx  2  3k 2 CR 3E
= 1 −  = 10r .x where x = 4q/25 Now, Q = C × ( ∆ V) =
2r  5  R1 + R 3
Similarly in region II
−kx k(q) k( −x + q) Sol 13: (A)
E
= + =
2 2
r r r2
2F
3.5r
1
⇒ Energy = ∫ ε0E2 (4 πr 2 )dr 30
2.5r
2 9V 2V
30 60 +
k(q − x)2  2 2  2k(q − x)2 1F
=  −  = -
2r 5 7  35r
O
-2V
Sol 10:=i 2 / 4 + =
5 2/9 -
or
2 10 +
So ∆ V for 4µ F =iR = 5 × =V
9 9
2 8 2µC
And ∆ V for 5 µ F = iR = 4 × = v (a) ∆V across 1µ=
F = 2V
9 9 1µ F
1 / 2 × 5 × (8 / 9)2 9−2
So Energy ratio = Now so current =
1 / 2 × 4 × (10 / 9)2 −1
 1 1 
30 +  + 
5 64 4  60 30 
= × = =0.8
4 100 5 7 7
= = = A 0.14 A
30 + 20 50
Sol 11: Now after connection all the 
charge will transfer to be outer shall. 30C (b) Current = 0; So, qC = 9 × 1 = 9µ C
1
Now, the heat generated would be And qC =( ∆V) × C2 =0 × C 2 =0
same as then energy of the region 100m 2

between the sphere and the shell 20m


(Electric field elsewhere is same)

kq
So, E =
r2
1 9 . 9 0 | Electric Potential and Capacitance

Sol 14: Q 2 (0.05)2 .[1 − e− t/2 ]2 × (µ c)2


(b) Energy = =
R1 i i1 2C 2 × 0.01(µ F)

5 × 0.05  2
= 1 − e−10 
R2 R2  2  
E + qC  0.125 µJ
-
q
⇒ E − iR1 − i1 R 3 − =0  ….. (i) Sol 15:
C
0 
And E − iR1 − i2R 2 = ….. (ii) q1
i= i1 + i2 …… (iii) (a)
 q2
q
So i2R 2 − i1R 3 − =0
C
q dq1 dq2
(i − i1 ) R 2 − i1R3 − 0
= =
=i =
C dt dt
q
iR 2 − i1 (R 2 + R3 ) − 0
= q1 q2
c And − iRC =
C C
R  q
⇒ iR1.  2 = i1 (R 2 + R 3 ) + and q1 + q2 =
CV
 R1  C

 q R q ⇒ q1 − iRC = CV − q1
⇒  E − i1R3 −  . 2 = i1(R 2 + R3 ) +
 C  R1 C dq1
⇒ 2q1 + .(RC) =
CV
dt
R  R .R  q  R 
⇒ E. 2= i1  R 2 + R3 + 3 2  + 1 +  2q1 dq1
R1  R1  C  R1  ⇒ + V /R
=
C dt
q
⇒ E.R 2= i1(R1.R 2 + R1.R 3 + R 3 .R 2 ) + .[R .R ] q,t t
C 1 2 ⇒ ∫
q0,0
(
d q1.e2t/RC = ) V
∫ R .e
0
2t/RC
.dt
2 dq q(R1 + R 2 )
⇒ = +
∑ R1.R 2 dt c.( ∑ R1.R 2 ) 2t/RC V (RC)  2t/RC 
⇒ q1.e= − q0 . . e −1
R 2  
t.(R1+R2 )  t.(R1+R2 ) 
 E.R 2  t c( ∑ R .R ) dt q,t  c.( ∑ R +R )  VC 
 ∑ R .R  ∫ e = ∫ d q.e
⇒  1 2 1 2
  ⇒ q=
1 q0 .e
−2t/RC
+ . 1 − e−2t/RC 
 1 2  0 0,0   2  

dq1  −2  −2/RC VC  2  −2t/RC
 t(R1+R2 )  t(R1+R2 ) = So, q0.  .e +   .e
E.R 2 C( ∑ R1.R 2 )  C( ∑ R1.R2 )  C( ∑ R1.R2 ) dt  RC  2  RC 
⇒ . e −1 = q.e
∑(R1.R 2 ) (R1 + R 2 )  
  −V V
= × 2.e−2t/RC + .e−2t/RC
R R
ECR 2  −t(R1 + R 2 )  dq
⇒ 1 − e =q V
(R1 + R 2 )  C( ∑ R1.R 2 )  = i=
− 1=+ .−2t/RC
dt R
(b) Heat generated = Einitial − Efinal
 − t(300 +300)×2 
 10 × 0.01× 300   0.01×106 [300×300 +300×50 + 300×50×186 ] 
q   . 1 − e   1  CV 2 1 
 300 + 300   1
  = × CV0 2 −  ×  × × 2 
2 2  2  C 
 
0.1 
= . 1 − e− t/2  = 0.05 1 − e− t/2  µC CV02 CV02 CV02
2    
= − =
2 4 4
P hysi cs | 19.91

3q q q So capacitance = Q / ∆V
Sol 16: E = − =
2Aε0 2Aε0 Aε0 = q / 3q / 5c
qd q 5C 5Aε0
⇒ Voltage = E × d= = = =
Aε0 C 3 3d
1
So energy initial = × C × V2 (ii) Charge on plate 3 ⇒ q4
2
2
q (from 2 and 3)
2q4 + q2 =
1 q q2 q2d
= × C ×  = = ⇒ q4 = − q / 10
2 C 2C 2Aε0
Now, we have ∆ v =
3q / 5C , So v 0 = 3q / 5C
and Efinal = 0 (both have q = 2q)
5V0 C
So Einitial = Efinal + heat ⇒ =q
3
q2d 5V0 Aε0
⇒ = Heat ⇒ .
=q
2Aε0 3d
Aε0 .V0
Sol 17: So, q4
= ⇒ on plate 3
6d
1
And charge on plate 5 = q1
2
-
3 Now, q1 + q2 =
q
4
5
⇒ q1 = 2q / 5
6
+
7 2 5V0 .Aε0 2 V0 Aε0
8 = × =
5 3d 3 d

Sol 18:

1 2 3 4 5 6 7
Qo Q2 Qo-Q2
3F
3F 10V 6F
5V
Q1-Qo 3F
-q1 7 q
2 8
Q1 A B
q1 1 -q3
q3 6F 5V 6F


q4 5 Qo
3 -q4 6 Qo 10V
A B
q2 4
-q2 Set up potential drop equations to get the values of Q0,
Q1, and Q2
Let q charge be given to the system. Now, close the switch ‘AB’ and draw a similar charge
distribution diagram.
q 
q1 + q2 = …. (i)
This will give you the new charges across all the plates.
q 
q3 + q4 + q2 = ….. (ii)
q3 q4 By using appropriate arithmetic, you can easily get to
− =0 ⇒ q3 =q4 …. (iii) the charge flown across the switch AB.
1 1 
q1 − q2 + q4 =
0
q2 
q1 + q4 = ….. (iv)
So on solving,
⇒ q2 = 3q / 5
So ∆ V =
3q / 5C (C = capacitance of each capacitors)
1 9 . 9 2 | Electric Potential and Capacitance

Sol 19: Only change would be there In q3

60C q3 = CV = 3µ F × 10 = 30µ F
-10C 10C So energy or heat generated = work done by battery -
energy stored in capacitor
A 20 V 4F
1
50C = 10 × 30 − × 3 × (10)2 = 300 − 150 = 150 µ J
2
−1 −1
 1 1 
Sol 21: Ceq = 1 +  +  + 1
3F   2 2 
V 2 V-20 2 2 4
1 = + = µF
20 V 4F 3 3 3
2F 3
4
5F So , Q =Ceq .V = × 24 =32 µ C
3
This will be divided equally, as the capacitor are
Charge is conserved
equal ⇒ Q = 16 µ C
(V − 0) × 3 + (V − 0) × 2 + (V − 0) × 5 + (V − 20) × 4 = 10 + 60 + 50
= 10 + 60 + 50 1
1 2
2
⇒ 14V − 80 =
120 24V 1
1 FF S 2
2 FF
+
+ 24V S
⇒ VA = 200 / 14 = 100 / 7 --
2'
2' FF 1
1 FF
300
(b) q1= 2 × (100 / 7) , q2 =3 × V = µC 4
4 3
3
7
5 × 100 300 (a)
(a)
And=
q3 = µC
7 7
-Q
-Q11 Q
Q11 Q
Q22 -Q
-Q22
 100 
= q4 = (VA − 20) × 4 µ F =  − 20  × 4µ F 1 2
 7  1
1FF 2FF
2
24V 1 S 2
+ 24V
+ S
−160 -- 0
= µ F = 22.88
7
0 2
2 FF V
V00 1
1FF
4
4 3
3
Q22 Q Q22
Sol 20: Q -Q
-Q11 Q
Q11
5
(b)
(b)

2F
10V 3F When the switch S is closed as shown in the figure
(b), then the charges stored across positive plates
5V

q2 capacitors 1 and be Q1 and Q2. The same charges will


0 - + (V+10) be distributed across negative plates of capacitor 3 and
4F 4. Let the potential at the negative terminal is zero, then
Now, q1 + q2 =
0 the charges across capacirots 1, 2, 3 and 4 will be

0 (Cons. of charge)
⇒ (V − 5)2 + (V + 10)4 = Q=
1 ( 24 − Vo ) × 1µF …(i)
⇒ 6V + 30 =
0 Q 2= ( 24 − Vo ) × 2µF  …(ii)

⇒ V = −5 Q 3= V0 × 1µF …(iii)

So q1 =−
( 5 − 5)2 =− 20 µ C Q 4 = V0 × 2µF …(iv)

And q2 = ( −5 + 10) × 4 = 20 µ C Thus from (i) and (iii) V0 = 12V and thus the charges be
n
After the switch is closed, the above eq s would s + Q1 =
Q3 =
12µF and Q 2 =
Q4 =
24µF
111 be valid , and hence q1 and q2 would be same.
P hysi cs | 19.93

Thus when switch is zero, 12µF charge is passed through Sol 24:
the switch. q1 C

Sol 22: Before opening the switch potential difference VF 0


across both the capacitors is V, as they are in parallel. q2
Hence, energy stored in them is,
VC
1 2
U= U= CV
A B
2 So, =
Q 156 × C
Thus, q1 + q2 =
156C
∴ UTotal =CV 2 =Ui
…(i)
⇒ CV + αCV 2 =156C
After opening the switch, potential difference across it
⇒ V 2 + V − 156 =0
is V and its capacity is 3C
⇒ (V − 12)( V + 13) =
0
1 2 3 2
∴ U
=A (3C)V
= CV ⇒ V = 12 Volts
2 2

In case of capacitor B, charge stored in it is q = CV and ε0 (A/ 2) K ε0 (A/ 2)


Sol 25:
= (i) C +
its capacity is also 3C. d d

8.85×10-12
q2 CV 2 = [0.02+ 0.18]
Therefore,
= UB = 8.85×10-4
2(3C) 6
= 0.2 ×10-8 F
2 2
3CV CV 1 2
∴ UTotal = + Energy = CV= 1.2 × 10−5 J
2 6 2
(ii) Q = CV
10 2 5CV 2
= = CV Uf  …(ii) ε0 A
6 3 Find the new capacitance, C' =
Ui 3 d
From eqs. (i) and (ii) U = 5 Q 2
Q 2
f Work done = – = 4.84 × 10−5 J
2C' 2C
Aε0 Aε0 AV0
Sol 23: C = , So Q = CV = × V0 = .ε 0
d d d
kc
AV0 ε0 Q2 Q2 (iii) C’ 2
So, Q = , Einitial = =
d 2C0 2C0
1 1 Aε
= × C0 V02 = . 0 × V02 Find the new charge distribution, and proceed as (i) & (ii)
2 2 d
Energy of system = 1.1×10-5 J
ε0 AV02
Einitial =
2d
2 Sol 26:
 Aε0  2
  V0
Q2 = d  V02 Aε0
E=
final = .
2Cf  Aε0  2k d (1-x)
2.k.  
 d 
x
Efinal + work done = Einitial

Aε0 V02 1 Aε V 2 Aε0 V02 Now area of air = 1× (1 − x)m2 = (1 − x)m2


 1
⇒ . + W = 0 0= ⇒ W  1− 
2d k 2d 2d  k = area of air = xm2
1 9 . 9 4 | Electric Potential and Capacitance

So both are in parallel connection. q1 q2


⇒ + − iR =
0
(1 − x).ε0 11× x.ε0 C 2C
Capacitance = +
d d
q1  q + 100  qd1
ε0 10x ε0 ε0 ⇒ +  1 + .R =
0
= + = 1 + 10x  C  2C  dt
d d d 
Now, Q = CV 3q1 50 dq1
⇒ ++ + .R = 0
dC dC 2CR RC dt
⇒ = i= .(V)
dt dt
ε d dq1 3
= V. 0 [1 + 10x] ⇒ + .q =
− 50 / RC
d dt dt 2CR 1
V.ε0 dx 500 × 10 × 0.001 q1t d
= × 10 × = −50
∫ d(q1.et/20R /3 ) = . et/2RC/3 .dt
RC ∫q
d dt 0.01× 8.85 × 10−12 ⇒
100,0
−10 −5
= 5 × 8.85 × 10 = 4.425 × 10 Amp.
−50 2CR  t/2CR /3−1 
⇒ × . e
Sol 27: We have Ceq= 30 µ F RC 3  

−100  (t/2RC/3) 
10F ⇒ q1et/2CR
= /3
− 100 e −1
3  

100 
q1 100.e− t/RC/3 −
⇒= . 1 − e− t/2RC/3 
3  

200 400 − t/2RC/3


⇒ q2 = q 1 + 100 µ C = + .e
3 3
20F

+ - 200
Charge
10V (C) 3

So, q = 30 × 10 = 300 µC
t
Thus, q on 20 µ=
F 200 µC
Sol 28: Q = C0V0
And q on 20 µ=
F 200 µC
Now,
Now,
q1 -q1 q1 q2

C0 C

-q2 q2
q1 q2 q1 q q C
So, − =0 ⇒ =2 ⇒ 1= 0
C0 C C0 C q2 C
Now, q2 − q1 = 200 − 100 = 100 µCNow,
C0
−dq1 dq2 So, q1
= (q1 + q2 ) (After first)
i= = − (C0 + C)
dt dt
q1 q2 C02 V0
And, + − iR =
0 = (After First)
10 20 (C0 + C)
P hysi cs | 19.95

After 2nd V 2dq1 2q1


⇒= + (from i and ii)
C0 C0 2R dt C
q1 = . .(C V )
(C0 + C) (C0 + C) 0 0
V t t/RC q1,t t/RC
 C0  2 ⇒
2R 0 ∫e = 0,0 ∫ d.(q1.e )
s= .C0 V0
 C + C 
 0  VC  t/RC 
⇒ . e. −1 =q1.et/RC
 C0  2  
Now after n ⇒  .C V = q1
 C + C  0 0
 0  VC 

= q1 1 − e− t/RC 
n 2  
q  C0 
voltage 1 V=
So= 0
 C + C 
35
C0  0  Sol 30:
n
 C0 
⇒   = 0.35
 C0 + C 
300C -360C
n 300C -360C
 0.2 
=   = 0.35 -300C 360C
 0.2 + 0.01075  -300C 360C

⇒ (0.094899)n = 0.35
qq1 1 -q-q
1
1
ln(0.35)
⇒ n= 
22 FF
ln(0.94899) -q33
-q -q-q
2 2
⇒ n = 20 33FF 22FF
q33 q2q2

Sol 29:
S −q3 q1 q2
i R
i1 (i) Now Kirchhoff’s law : − + 0
=
3 2 2
i3 q2 q1 = 3q
=2 3q1 + 2q3 …. (i)
V R 
C
i2 C and charge conservation
⇒ q1 − q3 = 300 µ C
i = i1 + i2 + i3 q2 − q3 = 60 µ C
dq1 dq
= i1= , 2 i2 = 3 60 − q3 =
 3 300 + q3  + 2q3
dt dt
q1 q2 = 180 − 3q3 = 900 + 3q 3 + 2q3 = −720 =
8q3
And − = 0 ⇒ q1 + q2  …. (i)
C C = q3 =
− 90 µ C
dq1 dq2
⇒ = = i= i And thus, =
q2 150 µ C
dt dt 1 2
And =
q1 210 µ C
d2
⇒ = i3R ⇒ q2 = i3RC 2
C 1 Q2 1 Q2
(ii) Einitial = × + ×
q2 2 C1 2 C2
⇒ i3 =  …. (ii)
RC 1 (300)2 1 (360)2
q = × + ×
V − iR − 1 = 0 2 3µ F 2 2µ F
C
= (300 × 50 + 360 × 90) µ J = 0.0474 J
(Kirchoff’s Law in bigger loop)

V q q (210)2 (90)2 (150)2


⇒ =i + 1 =i1 + i2 + i3 + 1 Efinal = + +
R RC RC 2×2 2×3 2×2
= (5625 + 1350 + 11025) µ J =0.018 J
1 9 . 9 6 | Electric Potential and Capacitance

Sol 31: 
  P   1  ˆ
E2 + 2k    3  ( − j)
+q  2 y 

e    kP
E = E1 + E2 = ( −ˆi − 2ˆj)
2y 3
a a
Sol 33:

+q +q I. +q
a
r
Initial +q
2
kq
Ui = ×3 r
a
a/2 a/2 Or
eq eq eq +q
r
Final
kq2 +q
kq2
Uf = + ×2
a a II. +q
 
2 r
2v
Work done = Uf – Ui

5kq2 3kq2 2kq2 r


= - =
a a a +q O r +q
Work done = Power × Time

2kq2 2r
⇒t=
ap +q
9 −2
2 × 9 × 10 × 10
= = 18×104 sec
1× 103
kq2 kq2 kq2 kq2
UI = ×2+ + ×2+
r 2r 3r 4r
Sol 32:
kq2 kq2 kq2
y UII = + + ×4
4r 2r 5r
P (0,y)
kq2  8 4 
Wfirst step = UII – UI =  − 
r 3 5
o x
45
 III.
P

E1 o,y (0,y) 


E2 2v

P r
 +
2 
P r
2

  P  1 2r
E1 = k   3 ( −ˆi)
 2  y 0
P hysi cs | 19.97

IV. Sol 35:


Eq
r q
mg
2r 2r
r H

r
UIII = UII ∴ wsecondstep = 0
dr
⇒ wthird = UIV – UIII – UII – UI - Wfirst step

∴ kldone
∴ Total work total =w first + wsec ond + w third =0 Initial electric potential = V
Q
kdq
Sol 34: Q = ∫ dv = ∫ (H2 + r2 )1/2
0

a
kσ2πr,dr  q 
⇒V= ∫ (H2 + r2 )1/2  where σ = 2 
A 0  πa 
L a
 (H2 + r 2 )1/2 
B ⇒ V = πσk  
Q  1/ 2  0
σ = surface charge density = take an elemental
πRL
part
dx
(
⇒ V = 2psk (H2 + a2 )1/2 − H = 2 )
Final electric potential Vf = 2psk (a)
 [substitute H = 0]
r
 By equation conservation,
x A mg H + 2psk ((H2 + a2)1/2 – H) q = (2pska)q

 q   q 
⇒ gH + σ   ((H2 + a2)1/2 –H) = (σ . a)  
dp 2 ε
 0  m  m2 ε0 

The potential due to this part at A is ⇒ H + 2 (H2 + a2)1/2 – H) = 2s


kdq kdq 2
dV = =  H H2
2 2  ⇒  a +  = H2 +a2 ⇒ 3 = aH
R +x  2 4
k
⇒ dV = . σ . 2pr d  4a
 ⇒H=
3
⇒ dV = kσ . 2psin θ d 
U = mg  2 H2 + a2 − H 
V
 
⇒V= L dU
∫ dV = kσ 2π sinθ ∫ d
=0⇒
2H
-1=0
0 dH H2 + a2
1 Q R a
= 2π L L ⇒H= (equilibrium)
4π ε0 πRL 3

Q
⇒V= U
2n ε0L

Energy required to bring the charge from infinity to


open = Uf – Ui a
H=
qQ 3
= 2π ε L
0
1 9 . 9 8 | Electric Potential and Capacitance

Exercise 2
⇒ 4.5CV 2 =
Heat generated
2
Single Correct Choice Type =ratio 4.5CV
= / 2CV 2 2.25

Sol 1: (A) Vmax 16/3 Sol 4: (B)


O oA
−1 d
1 1  16
Ceq =
 +  =µC then let vmax = the max (2, 4)
 8 16  3  oA
d (5)
voltage across the capacitors oA
d
16 (A, 1) oA (3, B)
So charge =Q= Cv max
= × v max
3 d
Now v =16 × v × 1  Q  
1 max
3 16  C  
E
v
= max < 80 ⇒ v max < 240V  2ε0 A   ε0 A 
3   
16 1  d   d 
and v 2 = × v max × Net capacitance =
3 8 3ε0 A
2 d
= v < 20 ⇒ v max < 30V
3 max 2 ε0 A
=
so vmax = 30V . 3 d
2 ε A 2 ε0 A
16 × 30 Charge flown =  0  E = E
Thus =
Q = 160µC 3 d  3 d
3  

Sol 5: (A) Force between plates = mg


Sol 2: (B) Ceq = 2C where C= capacitance of each plate.
σ σ2 A Q2
1 And A
× σ= = mg ⇒ = mg
So energy = × Ceq × V 2 2ε0 2ε0 2Aε0
2
(0.1)(8.85 × 10−12 ) ⇒ 2mgAε0 =Q
= CV 2 = × 102 = 10−1 µJ
−3
0.885 × 10
Sol 6: (B) Now across point c and d ∆V = 0
Sol 3: (B)

CV -CV CV 2CV q1 C


2V q2
Initial Final 2C

⇒ The capacitance is 0 for that


Now, initial energy + work done by battery
⇒ Ceq = C1 + C2 = 2Aε0 / d
= Final energy + heat generated

1
∴ × C × V 2 + 2V.(3CV)
2
1
= × C × (2V)2 + Heat generated
2
1 2
⇒ CV + 6CV 2 = 2CV 2 + Heat generated
2
P hysi cs | 19.99

Sol 7: (B) 2.1 Vmax .C 10


V=
2 = 0.3Vmax < 1 KV ⇒ Vmax < KV
3C 2C 7C 3

q1 -q1 q1 -q1 2.1Vmax .C 20


V'=
2 = 0.7Vmax < 2 KV ⇒ Vmax < KV
v2 v2 3C 7
= 2.5 KV

2
q2 -q2 q2 -q2 Sol 9: (A) q 1F
v2 v2

oA oA
d d

A B 1
q 1F
Let there be a charge Q on body 1.
oA
q1 q2 q1 1
d So = ⇒ =
2ε0 A 1 2 q2 2
Net Capacitance =
d and q1 + q2 = Q ⇒ q1 = Q / 3 where q = initial charge
−1 −1
 1 1   1 1  Q/ 3n
⇒q=
Sol 8: (A) Ceq =  +  + + 
 3C 2C   7C 3C 
EC
6C 21C 33C Sol 10: (D) Total charge now= E.C / 2 =
= + = =3.3C 2
5 10 10
Now after dielectric is inserted,
Q max = Vmax × Ceq = Vmax (3.3C)
kC.C kC kCE
Ceff
= = so
= Q net
q1 q2 q 4 (kC+ C) (k + 1) (k + 1)
Now
= = ; 1
6 / 5C 21/ 10C q2 7 KCE CE (K − 1)CE
so Q flowed = − = from B to A
4 k +1 2 2(k + 1)
Now q1 = × Vmax × (3.3C) =1.2 Vmax C
11
as capacitance increases.
7
q2 = × Vmax × (3.3C) =2.1Vmax C
11 Sol 11: (B) Voltage would be highest at x = 0. And
E = constant ⇒ V ∝ x so it would decrease linearly,
from x = 0 to x-d, will remain constant from x = d to
x = 2d.
c B

A
d
Sol 12: (C) 1000V 0V

1.2 Vmax .C ∆V = E × ∆d = 200 × 5 = 1000V.


V
=1 = 0.4Vmax < 1 KV ⇒ Vmax < 2.5 KV
3C
Now the electric field would be same so
1.2 Vmax .C 10
V'=
1 = 0.6Vmax < 2 KV ⇒ Vmax < KV ∆V = E × ∆d = 200 × 3 = 600V.
2C 3
1 9 . 1 0 0 | Electric Potential and Capacitance

Sol 13: (B) Q1 =


1500µC and Q 2 =
100µC q02 RC
⇒ ×R × × 1= 3.6 × 10−3 × 2
q1 q2 (RC) 2 2
q1 + q2 = 1600µC and = ⇒ q1 = q2 = 800µC
1 1 ⇒ q02 = 7.2 × 10 −3 × 2 × 10 −6
∆V= q / c= 800µC / 1µC= 800V
=14.4 × 10−9 =144 × 10−10
E.kC ⇒ q0 = 12 × 10 −5
Sol 14: (C) Q
= 1 Q=
2 Ceq=
.E
(k + 1)
dq  1
EC Q 2 ' k + 1 Sol 20: (C) q = q0 .e− t/ τ ⇒ = q0 .  −  .e− t/ τ ;
and Q ; dt  τ
= 1' Q=
2' Ceq
= .E =
2 Q2 2k
q
− 0 .e − t / τ
i=
τ
Sol 15: (B) From the symmetry, we can say that the
capacitance would be same. τ = RC = (r + 2)0.5 × 10−6 sec

1
and thus ×i i|t ln 4µs.
=
1.5F 2 =|t 0 =

300 C -360 C
1
-300 C +360 C τ = 2µs = (r + 2) × × 10−6 ⇒ r = 2Ω
2
1  q0   q0  − t/ τ
⇒ ×  −  =  −  .e = et/ τ = 2
2  τ   τ 
1.5F ⇒ ln 2 ⇒ ln 4 / τ
Sol 16: (A) q= q0 .e
q3− t/ τ -qwhere
3 τ (2 + r)0.5µ F
=
dq q0 − t/ τ 1.5F
⇒ =.e Multiple Correct Choice Type
dt z
300 -q -q2 -360 C

C1
-300 qC 2F ln2 × µs 3F +360 C Sol 21: (A, B, C) Now from charge conservation
e− t/ =
τ
1/ 2 1⇒ ln2
= t/τ⇒ = qln2
2
z q1 + q2 = 60µC ... (i); q3 − q1 = 300µC ... (ii)
r == 00)
⇒ τ = 1µs = (2 + r) × 1/ 2 ⇒ (r
⇒ q2 + q3 = 360µC
1.5F q1 q3 q2
q3 -q3 + − = 0 ; 3q1 + 4q
=3 2q2 ... (iii)
2 1.5 3

3(q3 − 300) + 4 q3 = 2(360 − q3 )


-q1 -q2
 q1
2F
q2
3F 7q3 − 900 = 720 − 2q3 ⇒ 9q3 = 1620
1620
⇒ q3 = = 180µC
9
and q1 =
−120 µC and q2 =
180µC
1
Sol 17: (D) R e q × C = × 100µF × 103 −1
2  1 1  2C 2CE
Sol 22: (A, D) Ceff =
 +  so charge =
= 0.5 × 105 × 10−6 = 0.05sec  C 2C  3 3

Final Ceff =
2C ⇒ charge =
2CE
Sol 18: (D) As at t = 0 the capacitor is assumed as a wire.
2CE − 2CE / 3 =
4CE / 3

dq q
Sol 19: (B)
= q q0 .e− t/RC ⇒= 0 .re−= t/RC
i Sol 23: (A, D)
dt RC
q0 1F v+10 2F
⇒=i .e− t/RC Now ∫ i2R.dt= 3.6 × 10−3 5 v O
−6
2 × 10 × 10
C1 C2
P hysi cs | 19.101

From charge conservation Q C = Q C 30


1 2 ∆V across 12 µF = 4 × = 10V
12
(5 − V) × C1 = (v + 10) × C2 ; 5 − V = (V + 10)2
CE/3 CE/3
−15 =
3V ⇒ V =−5V 2CE/3 + -

1 1
EC = × 1× (10)2 = 50µ J ; EC = × 2 × (5)2 = 25µ J  + - o
1 2 2 2 CE/3 CE/3

⇒ 2EC =
EC + - + -
1 1
E E
σ
Sol 24: (A, B) We have E = (by one plate)
2ε0
Sol 26: (A, B, D) Charge = KCV – CV=(K-1)CV
σ Q2
So force = . Q= Energy absorbed= (K − 1)CV 2
2ε0 2Aε0
1 (K − 1)
C2 V 2 d CV 2 Energy
energy = × K × C × V2 = .CV 2
Q = CV ⇒ F= × = 2 2
2Aε0 d 2d
1 1
Now × K × C × V 2 + work = × C × V 2 = (K − 1)CV 2
2 2
Sol 25: (B, C, D)
1
Ceq ⇒ Work
work =(K − 1)CV 2
12F 2

q1 Sol 27: (B, C) V = E d , so E=same.


7F same same
q2
3.9F
q1 Aε0 Aε0 V
3F Sol 28: (A, C, D) C = and Q = CV =
d d

Q CV V
Now Vnet = = = = E×d ⇒ E =V / Kd
KCV KCV C KC K
+ - + -
And energy initial + work done=energy final

 Q2 Q2
+ work done =
2C 2KC

V V Q2 1  CV 2  1 
Work
work =  − 1 =− 1 − 
2C K  2  K
−1
1 1
Ceq ' = 3.9 +  +  = 3.9 + 2.1 = 6µF Putting the value of "C" from the first line, we get
3 7
−1 ε0 AV 2  1 
1 1  =W 1 − 
Ceq =3.9 +  +  =4µF so q
= 4EµC. 2d  K 
 6 12 
−1
q1 q2 2.1  1 1 
q1 + q2 = 4E and = ⇒ q1 = × 4E Sol 29: (C, D) C=  + 
2.1 3.9 6 eq
 C1 C2 
= 0.7 × 2E = 1.4E −1
 x d−x−t
0.7 × 2EµC =  + 
so ∆V across 7 µF ⇒ =
q1/ 7 µF = 6  Aε Aε0 
7µF × 10  0
−1
⇒E=30V d−t  Aε0 
= =
⇒ q1 = 0.7 × 2 × 30 = 42µC.  Aε   
 0  d−t
q
∆V across 3 µ=
F = 14V ;
3 µF
1 9 . 1 0 2 | Electric Potential and Capacitance

Sol 30: (A, C, D) ∆V= iR= 5A × 1Ω= 5V.


CV KCV
So Q = CV = 1µF × 5 = 5µF

Sol 33: (B, D)


120 120
=i = = 40A
R e q  1 1 −1
 + 
2 2
V V
Now this I will be divided equally,=
so i1 20A,i
= 2 20A
Q=C’V=KCV
Now ( ∆ V2 ) =
0.... (B)
1 2 1 Q C = 20 × 2µC = 40µC.....(D)
Einitial = CV Efinal = × K × CV 2 = K × Einitial 1
2 2
As V =
E × d ,so
soEE=
=same
same Assertion Reasoning Type
same same

Q2 C2 V 2 Sol 34: (D) Statement-I is false electrostatic force


=F = K2
2Aε0 2AV0 remains the same.

Sol 31: (A, B, C, D) Sol 35: (C) Statement-II is false.


Q -Q q Q-q -2Q+q
2Q-q
Sol 36: (D) Q = (ε − iR)C ⇒ i = (ε − Q / C).1/ R
A B  -(Q-q)
Work done by battery = ε.Q = ε(ε − iR)
Q-q
Q 2 (ε − iR)2 C
And energy in capacitor = =
2C 2
d d/2 d/2
(ε − iR)2 C
= + iR.(ε − iR).C
2
i1 100 60

⇒ Statement-I is false.
120 C1 C2 V

Comprehension Type
i2 80 60

Sol 37, 38: (B, B)


Now the charge on outer surfaces should be same C1
q = Q −q ⇒ q = Q /2
So (A) =
−2Q + Q / 2 =
−3Q / 2
Charge on A=Q/2 and charge on B = 3Q/2 C2
So as E ∝ Q and V ∝ E ⇒ V ∝ Q and hence C is
correct.
C1.d q1 q2 Q 0 − q2 x
C2 = =⇒ =
Sol 32: (A, B, C, D) (A) at t=0 consider it as wire. x C1 C2 q2 d

10 10 10  
(B) = i= = = 5A Q0  Q 
Re q −1 2 x
 1 1 ⇒ 1+
=  0 
⇒ q2 =
1+  +  q2 d  1+ x 
2 2  
 d
(C) From kirchoff 's law
i1 R2
i1R1 = i2R 2 ⇒ = = cons tant
i2 R1
P hysi cs | 19.103

Now force Previous Years’ Questions


Sol 1: The diagrammatic representation of given
problem is shown in figure.

+ - Q1=C1V1=CV
+ -
+ -
+ -
+ -
t
d
+ -
x d-(x-t) + -
+ -
Q2 q22 Q 02 + - Q2=C2V2=(2C)(2V)=4CV
= = = + -
2Aε0 2Aε0 (1 + x/ d)2 2Aε
0
The net charge shared between the two capacitors is
Q 02 2d
1 Q’ = Q2 – Q1 = 4CV – CV = 3CV
And work done =
2Aε0 ∫ (1 + x/ d)2
d The two capacitors will have the same potential, say V’
2 2
Q0 d Q0 d The net capacitance of the parallel combination of the
= ×=
2Aε0 6 12Aε0 two capacitors will be
C’ = C1 + C2 = C + 2C = 3C
And potential difference = q1/C
The potential difference across the capacitors will be
Q0 Q 0 (x/ d) Q 0 x
q1 =
Q0 − = = Q' 3CV
1+ x / d 1+ x / d x + d V’ = = =V
C' 3C
Q 0 xd The electrostatic energy of the capacitors will be
potential q1 / C1 =
(x + d) A ε0 1 1 3
U’ = C’V’2 = (3C)V2 = CV2
2 2 2
Sol 39: (A, C) i=E/R1 → → →
Sol 2: Electric field within the plates=
E E Q1 + E Q2
E
Sol 40: (A, B, (B)
C) Current = 
R1 + R 2 E
E × R2
(A) Voltage
voltage = ;
(R1 + R 2 )
2
1 1  ER 2  +Q1 +Q2
(C) × CV 2 = × C × 
2 2

 (R1 + R 2 )   E2 E1

Sol 41: (A, C) (A) qmax = same.


1 2
(B) is wrong option as q = CV Now, C can be different
Q1 Q2 Q1 – Q 2
E = E1 – E2 = – ,E=
(D) Not necessarily, depends on R1 and R 2 also. 2Aε0 2Aε0 2Aε0

∴ Potential difference between the plates


Sol.42: (D) τ1 < τ2 ; R1C1 < R 2C2 ;
Now E1C1 = E2C2 (charge same)  Q – Q2  Q1 – Q 2 Q1 – Q 2
VA – VB = Ed =  1
 2Aε  d =  Aε  =
2C
R1 C2  0 
2  0 
⇒ C1 = C2 ⇒ R1 < R 2 ; <
R2 C1  d 
1 9 . 1 0 4 | Electric Potential and Capacitance

ε0 A ∴ V = V0, C > C0
Sol 3: (D) Applying C = ,
t1 t2 Q = CV∴ Q > Q0
d – t1 – t2 + +
K1 K 2
1 2
U= CV ∴ U > U0
we have 2
V
E= but V and d both are unchanged
A/2 A/2 A d
K1 d/2 K2 d/2 Therefore, E = E0
+  K d
K3 d/2 K3 d/2
Sol 7: (B, D) Charging battery is removed. Therefore,
q = constant Distance between the plates is increased.
Therefore, C decreases.
In parallel
q
ε0 (A / 2) Now, V = , q is constant and C is decreasing
C
d/2 d/2 Therefore, V should increase.
d–d/2–d/2+ +
K1 K3
1 q2
U= again q is constant and c is decreasing
ε0 (A / 2) Kε0 A 2 C
+ = Therefore U should increase.
d/2 d/2 d
d–d/2–d/2+ +
K2 K3
Sol 8: (A, C, D) Battery is revoved. Therefore, charge
K1K 3 K 2K 3
Solving this equation, we get K = + stored in the plates will remain constant
K1 + K 3 K 2 + K3
ε0 A
Q =CV = V
Sol 4: (C) DU = decrease in potential energy = Ui – Uf d
2 Q = constant.
1 1  V + V2 
= C( V12 + V22 ) – (2C)  1  Now, dielectric slab is inserted. Therefore, C will
2 2  2  increase. New capacity will be
1 ε0KA
= C(V1 – V2)2 C’ = KC =
4 d
Q V
Sol 5: (D) q1 = C1V = 2V = q V’ = =
C' K
This charge will remain constant after switch is shifted V' V
from position 1 to position 2. And new electric field E = =
d K.d
1 q2 q2 q2 Potential energy stored in the capacitor,
Ui = = =
2 Ci 2×2 4 1 ε AV 2
1 q2 q2 q2 Initially, Ui = 2 CV2 = 0
Uf = = = 2d
2 Cf 2 × 10 20 2
1 1  Kε A   V  ε0 AV 2
q2 Finally, Uf = C’V’2 =  0    =
∴ Energy dissipated = Ui – Uf = 2 2  d   K  2Kd
5
 q2  Work done on the system will be
This energy dissipated  =  is 80% of the initial
 5 
 q2   
ε0 AV 2  1 
stored energy  =  |DU| = 1– 
 4  2d  K 
 

Sol 6: (A, D) When dielectric slab is introduced capacity Sol 9: (B, C) The magnitude and direction of electric field
gets increased while potential difference remains at different points are shown in figure. The direction of
unchanged. the electric field remains the same. Hence, option (b) is
correct. Similarly, electric lines always flow from higher
P hysi cs | 19.105

to lower potential, therefore, electric potential increase if (d < n)


continuously as we move from x = 0 to x = 3d. kQ kQ 4x
⇒ - =0 ⇒ =d
q1 q1 d 4(x − d) 5

- + - + Electric field at P is zero


- +
- + d
- +
- + - +
+Q -Q/4
f
- +
- + E-Q/4 EQ
x
- +
- + - + kQ kQ
- E0/K + ⇒ =
2
E0 E0 d 4(d − x)2
x=0 x=e x=2e x=3d
±d
⇒d–x=
Therefore, option (c) is also correct. The variation of 2
electric field (E) and potential (V) with x will be as 3d d
follows ⇒x= or x =
2 2
E 2x
⇒d= or d = 2x
3
(since d > x)
d 2d 3d
x 
Sol 11: (A) r = (iˆ + 3ˆj + 2k)
ˆ - (4iˆ + 7ˆj + 2k)
ˆ

= −3iˆ − 4ˆj + 0kˆ



V |r | =5
k×q 9 × 109 × 10−8
B V= = = 18 v
A r 5

Field is parallel to r and thus has no 3-component

V0
x Sol 12: (A, C) It must have any K.E. at B to reach A
O d 2d 3d
Since
OA||BC and (Slope)OA > (Slope)AB UA – UB = -e (VA – VB) = +4eV
Because E0–d = E2d–3d (K.E.)f – (k.E.)i = UB – UA = -4eV
And E0–d > Ed–2d
Sol 13: (A)
Sol 10: (A, B, C) + -
+ -
+ -
PQ n P -Q/4 + -
+
+ - -
+ -
d + -
Potential at P is zero +
+ -
-

If (d>x)
By symmetry potential due to negative part =(potential
kQ kQ due to particle part). (Also every small charge is
⇒ - = 0 ⇒ d = 4d – 4x
d 4(d − x) equidistant from axis)

3d 4x ∴ Potential at all potential axis is zero


⇒x= =d=
4 3
1 9 . 1 0 6 | Electric Potential and Capacitance

Direction of field is perpendicular to axis and towards (It is independent of presence of conductor because
negative side induced charges provides zero potential at center)
There will be a torque when placed in uniform field Potential B = Potential at C
(Since electric field inside conduction is zero)
Sol 14: (A, D) Electric field due to dipole exists inside
⇒ V = constant
sphere only a circle on the sphere has zero potential
which is equidistant from poles of dipole ∴ Potential at B due induced charges
= Potential at B – potential at B due to q
kq kq −kqR
- = - =
+ qenclosed =0 4 π ∈0 (d + R) 4 π ∈0 d 4 π ∈0 (R + d)d
 Electric flux = 0
Sol 17: (B) Let the distribution of charges be
Zero potential
Sol 15: (B, C) (1) (2) (3) (4)
+q y x Q-x -Q+x y
r+ F

r -x -Q+x Q-x
Q 2

r+ d 2d d
-q
2F. 2
FR = (= 2F sin θ) =
2
a + 2
r + 2
2 Since the potential difference between plates (1) and
(4) is zero.
 kQ  kQ(P)
 2 .q  = − x.d Q − x(2d) (Q − x)d
r +  2
 (r + 2 )3/2
2 ⇒ + + =0
A ∈0 A ∈0 A ∈0
1 pQ 3Q
⇒ FR = . 3 (r >>  ) ⇒x= ∴ Option B
4π ε0 r 4
F.r ⇒ Charge on right side of plate 3 = Q – x = Q/4
Torque on dipole = (2  )
(r + 2 )1/2
2

= (=2 F cos q  ) Sol 18: (B) Also Charge is conserved on plate 1 and 4

r.(2 )  l Q  ⇒ y + (-x) + Q – x + y = 0 + 0
⇒T=  . .q 

 4 π ε0 r + 
2 2 1/2 2 2 Q
(r +  )  ⇒y= ∴ Option B
4
1 pQ Q
⇒T= (r >>  ) ∴ Charge on right side of plate 4 =
4π ε0 r 2 2

Sol 19: (C) Potential difference between (i) and (ii)


Sol 16: (A, D)
x ( d) 3Qd
= =
A ∈0 4A ∈0

q R B R C
Sol 20: (B) Obviously, both statements are correct. But,
Statement-II is not a correct explanation of statement-I.
kq
Potential at center of sphee =
4 π ∈0 (d + R) Sol 21: Refer theory on Superposition of electric field &
Ampere’s Loop Law.
P hysi cs | 19.107

Sol 22: (A, D) Q = KQ0 V → Constant Sol 30: (A, B, C)


+q -q
1 1 
=U =
2
(KC ) V2 =2
CV 2  k K U0
 
E3
E = E0 / k
+2q -2q
E1
O
1
Sol 23: (B, D) C ∝ E2
d

+q -q
Sol 24: (A, C, D) Q – remains constant.
2kq
∈0 A k ∈0 A E=
2 E=
3
C changes from to L2
d d 4kq
Hence, other variable also change. E1 =
L2
  
Sol 25: (B, C) Eall = E1 + E2 + E3

-Q 6kq 1 q
+Q = 6×
= 6K (Along OD)
=
L2 4 π ε0 L2

Vat O = 0

Vat line PR is zero because this line is equatorial axis for


three dipole.

x=0 d x=3d Sol 31: (B, D) After switch S1 is closed, C1 is charged


by 2CV0 when switch S2 is closed, C1 and C2 both
have upper plate charge CV0 .
When S3 is closed, the upper plate of C2 becomes
Q2  charged by −CV0 and lower plate by +CV0 .
Eoutside = 
A ∈0 
 → So, ( Q ) is wrong
Q2  Sol 32: (A, D) As E = V/d E1 / E2 = 1 (both parts have
Einside =
K A ∈0  common potential difference)

(B) is correct as the direction of field remain the same. Assume C0 be the capacitance without dielectric for
whole capacitor.
(C) As we are going in the opposite direction of electric
C0 2C0
field, potential would rise. k + C
=
3 3
C 2+k
Sol 26: (C, D) Using Gauss’s Law =
C1 k
  qin
Net =Flux ∫= E.ds
∈0 Q1 k
= .
It electric field is same at all the points of a surface, it is Q2 2
knows as equipotential surface.

Sol 27: (A, B, C, D) Refer Theory.

Sol 28: (C, D) Refer Theory.

Sol 29: (C) 2 µ F & 3 µ F are in parallel combination.


1 9 . 1 0 8 | Electric Potential and Capacitance

Sol 33: (D)

+80
-80
80
q2 =
2
x 80 = 32 q1 = 3 x 80 = 48
5 5
+32 +48
-32 -48

S
4 ε0
C10
= = 2 4 ε0 S
d/2 d

2 ε0 S ε0 S
=C20 = , C30
d d

1 1 1 d  1
= + = 1 + 
C'10 C10 C10 2 ε0S  2

4 ε0 S
⇒ C'10 =
3d

7 ε0 S
C2 =C30 + C'10 =
3d

C2 7
=
C1 3

C10 C20

C30

You might also like